'I'D ANDREESCD
DORIN ANDRICA
360 Problems for Mathematical Contests
TITU ANDREESCU
DORIN ANDRICA
360 Problems
for Mathematical Contests
GIL Publishing House
© GIL Publishing House
ISBN 973-9417-12-4
360 Problems for Mathematical Contests
Authors: Titu Andreescu, Dorin Andrica
Copyright © 2003 by Gil. All rights reserved.
GIL Publishing House
P.o. Box 44, Post Office 3, 4700, Zalau, Romania, tel. (+40) 260/616314 fax. (+40) 260/616414 e-mail:
[email protected]
www.gil.ro
IMPRIMERIA
�
ARTA IV GRAFICA I ��LIBRIS
Calea $erbanVodti 133,S.4,Cod 70517,BUCURE$TI Tel.: 336 29 11 Fax: 337 07 35
Contents
FOREWORD 3 FROM THE AUTHORS . . . . . . . . . . . . . . . . . . . . . . . . . . . . . . . . . . . . . . . . . . . . . . . . . . . . . . . . . 5 Chapter 1 . ALGEBRA . . . . . . . . . . . . . . . . . . . . . . . . . . . . . . . . . . . . . . . . . . . . . . . . . . . . . . . . . 7 Problems . . . . . . . . . . . . . . . . . . . . . . . . . . . . . . . . . . . . . . . . . . . . . . . . . . . . . . . . . . . . . . . . . . . . 9 Solutions . . . . . . . . . . . . . . . . . . . . . . . . . . . . . . . . . . . . . . . . . . . . . . . . . . . . . . . . . . . . . . . . . . . 17 Chapter 2. NUMBER THEORY . . . . . . . . . . . . . . . . . . . . . . . . . . . . . . . . . . . . . . . . . . . . . . . 47 Problems . . . . . . . . . . . . . . . . . . . . . . . . . . . . . . . . . . . . . . . . . . . . . . . . . . . . . . . . . . . . . . . . . . . 49 Solutions . . . . . . . . . . . . . . . . . . . . . . . . . . . . . . . . . . . . . . . . . . . . . . . . . . . . . . . . . . . . . . . . . . . 57 Chapter 3. GEOMETRY . . . . . . . . . . . . . . . . . . . . . . . . . . . . . . . . . . . . . . . . . . . . . . . . . . . . . . 85 Problems . . . . . . . . . . . . . . . . . . . . . . . . . . . . . . . . . . . . . . . . . . . . . . . . . . . . . . . . . . . . . . . . . . . 87 Solutions . . . . . . . . . . . . . . . . . . . . . . . . . . . . . . . . . . . . . . . . . . . . . . . . . . . . . . . . . . . . . . . . . . . 95 Chapter 4. TRIGONOMETRY . . . . . . . . . . . . . . . . . . . . . . . . . . . . . . . . . . . . . . . . . . . . . . . 137 Problems . . . . . . . . . . . . . . . . . . . . . . . . . . . . . . . . . . . . . . . . . . . . . . . . . . . . . . . . . . . . . . . . . . 139 Solutions . . . . . . . . . . . . . . . . . . . . . . . . . . . . . . . . . . . . . . . . . . . . . . . . . . . . . . . . . . . . . . . . . . 147 Chapter 5. MATHEMATICAL ANALYSIS . . . . . . . . . . . . . . . . . . . . . . . . . . . . . . . . . . . 179 Problems . . . . . . . . . . . . . . . . . . . . . . . . . . . . . . . . . . . . . . . . . . . . . . . . . . . . . . . . . . . . . . . . . . 181 Solutions . . . . . . . . . . . . . . . . . . . . . . . . . . . . . . . . . . . . . . . . . . . . . . . . . . . . . . . . . . . . . . . . . . 189 Chapter 6. COMPREHENSIVE PROBLEMS . . . . . . . . . . . . . . . . . . . . . . . . . . . . . . . . 221 Problems . . . . . . . . . . . . . . . . . . . . . . . . . . . . . . . . . . . . . . . . . . . . . . . . . . . . . . . . . . . . . . . . . . 223 Solutions . . . . . . . . . . . . . . . . . . . . . . . . . . . . . . . . . . . . . . . . . . . . . . . . . . . . . . . . . . . . . . . . . . 235 . . . . .
. . . . . . . . . . . . . . . . . . . . . . . . . . . . . . . . . . . . . .
. . . . . . . . . . . . . . . . . . . . . . . . .
FOREWORD
I take great pleasure in recommending to all readers - Romanians or from abroad the book of professors Titu Andreescu and Dorin Andrica. This book is the fruit of a prodigious activity of the two authors, well-known creators of mathematics questions for Olympiads and other mathematical contests. They have published innumerable original problems in various mathematical journals. The book is organized in six chapters: algebra, number theory, geometry, trigonometry, analysis and comprehensive problems. In addition, other fields of math ematics found their place in this book, for example, combinatorial problems can be found in the last chapter, and problems involving complex numbers are included in the trigonometry section. Moreover, in all chapters of this book the serious reader can find numerous challenging inequality problems. All featured problems are interesting, with an increased level of difficulty; some of them are real gems that will give great satisfaction to any math lover attempting to solve or even extend them. Through their outstanding work as jury members of the National Mathematical Olympiad, the Balkan Mathematics Contest (BMO) , and the International Math ematical Olympiad (IMO) , the authors also supported the excellent results of the Romanian contestants in these competitions. A great effort was given in preparing lectures for summer and winter training camps and also for creating original problems to be used in selection tests to search for truly gifted mathematics students. To support the claim that the Romanian students selected to represent the country were really the ones to deserve such honor, we note that only two mathematicians of Romanian origin, both former IMO gold-medalists, were invited recently to give conferences at the International Mathematical Congress: Dan Voiculescu (Zurich, 1994) and Daniel Tataru (Beijing, 2002) . The Romanian mathematical community unanimously recog nized this outstanding activity of professors Titu Andreescu and Dorin Andrica. As a consequence, Titu Andreescu, at that time professor at Loga Academy in Timi§oara and having students on the team participating in the IMO, was appointed to serve as deputy leader of the national team. Nowadays, Titu ' s potential, as with other Ro manians in different fields, has been fully realized in the United States, leading the USA team in the IMO, coordinating the training and selection of team contestants and serving as member of several national and regional mathematical contest juries.
One more time, I strongly express my belief that the 360 mathematics problems featured in this book will reveal the beauty of mathematics to all students and it will be a guide to their teachers and professors. Professor loan Tomescu Department of Mathematics and Computer Science University of Bucharest Associate member of the Romanian Academy
FROM THE AUTHORS
This book is intended to help students preparing for all rounds of Mathematical Olympiads or any other significant mathematics contest. Teachers will also find this work useful in training young talented students. Our experience as contestants was a great asset in preparing this book. To this we added our vast personal experience from the other side of the " barricade" , as creators of problems and members of numerous contest committees. All the featured problems are supposed to be original. They are the fruit of our collaboration for the last 30 years with several elementary mathematics journals from all over the world. Many of these problems were used in contests throughout these years, from the first round to the international level. It is possible that some problems are already known, but this is not critical. The important thing is that an educated - to a certain extent - reader will find in this book problems that bring something new and will teach new ways of dealing with key mathematics concepts, a variety of methods, tactics, and strategies. The problems are divided in chapters, although this division is not firm, for some of the problems require background in several fields of mathematics. Besides the traditional fields: algebra, geometry, trigonometry and analysis, we devoted an entire chapter to number theory, because many contest problems require knowledge in this field. The comprehensive problems in the last chapter are also intended to help under graduate students participating in mathematics contests hone their problem solving skills. Students and teachers can find here ideas and questions that can be interesting topics for mathematics circles. Due to the difficulty level of the problems contained in this book, we deemed it appropriate to give a very clear and complete presentation of all solutions. In many cases, alternative solutions are provided. As a piece of advice to all readers, we suggest that they try to find their own solutions to the problems before reading the given ones. Many problems can be solved in multiple ways and pertain to interesting extensions.
This edition is significantly different from the 2002 Romanian edition. It features more recent problems, enhanced solutions, along with references for all published problems. We wish to extend our gratitude to everyone who influenced in one way or another the final version of this book. We will gladly receive any observation from the readers. The authors
Chapter 1 ALGEBRA
PROBLEMS
{Cl ' C2 , , cn } .
Let C be a set of characters We call word a string of at most m characters, m :::; that does not start nor end with How many words can be formed with the characters of the set C? 1.
n n,
. . •
Cl.
. . . , 5(n ) n x, y , x y,
The numbers 1, 2, are divided into two disjoint sets. Prove that these > sets contain at least pairs is also an such that the number element of the set which contains the pair. 2.
x-y [ b] 3. Let a l , a2 ,I. . . ,an be distinct numbers from the interval a, and let be a } { permutation of , 2, . . . , n [. b] [ b] Define the function f a, a, as follows: 1 ,n a { q i if x = �i' x = f( ) x otherwIse x, where f[h] Prove that there is a positive integer h such that f[h](X) a
:
-t
()
i
=
fafa ... af. htimes
'----v-----"
2x x,+y,yz2 areynonzero 2Z +numbers 2x x3withy3x + yZ3+ z = 0, then 2 + Z2 real x + y + y + Z + Z + x = -yz + -zx + xy-. b d b d Prove that Let a, , c, be complex numbers with a + + C + a3 + b3 + c3 + d3 = 3 (abc + bcd + cda + dab) . b3 3 b b 3 6.b Let a, , c be nonzero real numbers such that a + + c = 0 and a + + c = a5 + 5 + c5• Prove that 4.
Prove that if
---
5.
7. Let
=
O.
b d a, b, c,2d( be4 integers. Prove that a + + c + divides a + b4 + c4 + �) _ (a2 + b2 + c2 + d2 ) 2 + 8abcd.
10
1. ALGEBRA 8.
Solve in complex numbers the equation
(x + l )(x + 2)(x + 3)2 (X + 4)(x + 5) 360. =
9.
Solve in real numbers the equation -IX
+ VY + 2v'z=2 + .jU + -IV x + y + + + =
z
u
v.
10. Find the real solutions to the equation
( x + y ) 2 (X + 1 ) (y - 1 ) . =
1 1 . Solve the equation
� x + J4X + V16X + l·· + J4nx + 3
-..;x =
1.
1 2 . Solve the equation
b where a, ,
x + a + -Jx + b + -Jx +
-J c
c =
x+a+b-
-J
c,
are real parameters. Discuss the equation in terms of the values of the
parameters.
b be distinct positive real numbers. Find all pairs of positive real a and ( ) numbers x, y , solutions to the system of equations { xx24 - yy24 ax{/a-2 byb2 . 13. Let
=
_
14. Solve the equation
=
_
[ 25x4- 2 ] l3x3+ 4 ' =
[a] denotes the integer part of a real number a. 1 + V5 . 15. Prove that If a � -- -' then
where
2
11
1. 1. PROBLEMS
x, y, z are real numbers such that x S + y S + Z S f:. 0, then the 2xyz - (x + y + z) ratio xS+yS+ZS equals � if and only if x + y + z 16.
Prove that if
=
17.
Solve in real numbers the equation vx;:-=-r +
18.
1 9.
O.
2 .JX2 - 4 + . . . + n JX n - n2 "21 (xl + X2 + . . . + xn) .
1 -+x -y = 9
=
Find the real solutions to the system of equations 1 1
(
1
-
?Ix
+
1
-
�
)(
1+
1
-
?Ix
)(
1+
1
-
�
Solve in real numbers the system of equations
) - 18 -
y22 + u22 + v22 + w22 44x - 1 x2 + u2 + v2 + w2 = 4y 1 x2 + y2 + v 2 + w 2 4u - 1 x2 + y2 + u2 + w2 4 v - 1 x +y +u +v w- 1 20. Let aI, a2 , a s, a4 , a5 be real numbers such that al� + a2 + a s + a4 + a5 0 and � � � i l l l�max i
-
=
=
=
=
>
--
--
to zero. Prove that
a, b, c be real numbers such that abc 1. Prove that at most two of the numbers 2a - -1b ' 2b - -1 ' 2c - -1 c a are greater than 1. 23. Let
=
12
1. ALGEBRA b d 24. Let a, , be real numbers. Prove that 1 b b d ) 2 2 2 min (a - , - a- , - a < (0 1 ) k 25. Let aI, a2 , ' . . ,an be numbers in the interval , and let ;::: 2 be an integer Find the maximum value of the expression Ln V'ai (1 - ai+1 ) , i=l c,
c ,c
�
- 4'
m and n be positive integers. Prove that 1 m n xn - l x - >- -x m for any positive real number x. [�] ! !) ( ;::: 27. Prove that m n for all positive integers m and n. 26.
Let
---
28.
Prove that
for any integer
1 + -1 + -1 + ' ' ' + -1 > n � -n +2 -1 v'2 v'3 \Iii
n ;::: 2. 2 9. Prove that n ( 1 - 1 / vn) + 1 > 1 + �2 + �3 + . . . + .!.n > n (\In + 1 - 1 ) for any positive integer n . (0 1 ) na12a2 ' ." . .an Prove that 30. Let a I , a2 , . . . , an E , and let tn = a1 + a + + an Ln loga. tn ;::: (n - l) n. i= l 3n there is at least a perfect cube for any intege 31. Prove that between n and n ;::: 1 0. 32 . Compute the sum Sn = �n (- 1 ) k. k=l 33. Compute the sums: lk S T k ) ( ( l a) n = � b n = � (k l )(k 2)(k 3) + + 2) (�); ) + + + (�). .
"'"
1c(Ic+l) 2
13
1 1. . PROBLEMS
Show that for any positive integer n the number n n 1 1 .3+ . . . + 2 +
34.
: ) 22 n - 2
; ) 2n
C C is the sum of two consecutive perfect squares. 35.
Evaluate the sums:
36.
Prove that
12 (�) 32 (�) 52 (�) +
+
+...
C
n
2
: 1) 3n
( + 1)2
= n n
for all integers n � 3. Prove that
37.
for all positive integers n. Let
38.
Xn 22n + 1, =
for all positive integers n.
L2n [log k] k=l 2
=
(n - 2)2 n + n + 2
n =
1, 2, 3, . . . Prove that 1 2 2 - + - + - + . . . + -- < 1 3
2n -l Xn
2 Xl X2 X3
Let f : C -t C be a function such that f(z)f(iz) = Prove that f (z) + f ( -z) = for all z E C 39.
0
40.
f(a)
=
n-3
Consider a function f 1. Prove that if f(x)f(y) + f
:
(0, 00)
(�) f (�)
-t
=
�
Z2 for all z E
and a real number a
2f(xy) for all x, y E
then f is a constant function. 41.
Find with proof if the function t: � -t [- 1, 1], f(x)
42 .
For all i, j
IS(i, j ) l ·
=
1, n define S (i, j)
=
=
>
C.
0 such that
(0 , 00) , sin[x] is periodical.
Ln ki+i. Evaluate the determinant � = k= l
14
43.
Let
xii
=
{ abi 0
i where ai , bi are real numbers. � Evaluate the determinant 2 n 44. a)
1. ALGEBRA
if if if =
Compute the determinant
j i j, + j i 2n + 1 j 2n + 1 + IXii l · i
i
=
=
i
x y y x z
i
v
z
v
v z
x y y cdab x b b b d d b ) Prove that if the numbers a e , a e, , de a are divisible by a prime then at least one of the numbers a + b + e + d, a + b - e - d, a - b + e - d, a - b - e + d, v
z
p,
is divisible by p.
tl (x) x2 PI X qr t2 (x) x2
= = Consider the quadratic polynomials + + and + where + are real numbers. Prove that if polynomials and have zeros of the same nature, then the polynomial
45.
P2 X q� ,
PI , P2 , ql, q2
tl
t2
has real zeros.
a, b, e be realTnumbers withba 0bsuch that the quadratic polynomial (x) ax2 + ex + S + e S - 4abe has nonreal zeros. T ( ) ax2 + bx + e and T2 (x) Prove that exactly one of the polynomials I x b ax2 + ex + has only positive values. 46.
>
Let
=
=
47.
Consider the polynomials with complex coefficients
P (x) xn + alxn-l + . . . + an and Q(x) xn + blxn- l + . . . + bn i � ; having zeros Xl, X2 , . . . ,Xn and x , x , . . . ,x respectively. if al + a s + a5 + . . . and a2 + a4 + a6 + . . . are real numbers, then bl +Prove b2 + . that b . . + n is also a real number. =
=
15
1. 1 . BROBLEMS P (x) be a polynomial of degree n. If 48. Let k k P (k) -k0, 1, . . . , n for +1 P ) ( evaluate m , where m n. P( ) 49. Find all polynomials x with integral coefficients such that =
=
>
for all real numbers
x.
Consider the polynomials Pi, i that if the polynomial 50.
=
1, 2, . . . , n with degrees at least 1. Prove
P (x) P1 (Xn+1 ) + XP2 (xn+1 ) + . . . n- 1 ( n+1 ) + x pn x , is divisible by xn +xn - 1 + . . · + x + 1, then all polynomials Pi(x) , i 1, n, are divisible =
=
by x - 1. 51 .
Let P be a prime number and let
P (x) aoxn + a1 xn-1 + . . . + an ) . Prove that be a polynomial with integral coefficients such that anP =1= 0 (mod p ( ) 0 (mod p) for all if there are n + 1 integers 0'1 , 0'2 , , an+ 1 such that ar ) 1, 2, . . . , n + 1, then there exist i , j with i i- j such that a i aj (mod p . P ( ) P (xn ) 52. Determine all polynomials with real coefficients such that p n x =
==
• • •
r =
==
=
for all real numbers x, where n > 1 is a given integer. 53.
Let
P (x) aoxn + a1 xn-1 + . . . + an , an i- 0, =
be a polynomial with complex coefficients such that there is an integer
m with
P has at least a zero with the absolute value less than 1. P 54. Find all polynomials of degree n having only real zeros Xl, X2 , . . . , X n such that n P ( )1 n2I( ) P � x - Xi X X ' Prove that the polynomial
=
for all nonzero real numbers x. 55.
Consider the polynomial with real coefficients = + +...+
P (x) aoxn a1 xn-1
an ,
16
1. ALGEBRA
f:. and an
O.
Prove that if the equation P(x) equation
=
0 has all of its roots real and distinct, then the
x2 PII(x) + 3xP' (x) + P(x)
=
0
has the same property. Let R�? and Rf�) be the sets of polynomials with real coefficients having no multiple zeros and having multiple zeros of order n respectively. Prove that if P(x) E R �? and P(Q(x)) E R �? , then Q' (x) E R�]- 1 ) . 56.
57. Let P(x) be a polynomial with real coefficients of degree at least 2. Prove that if there is a real number a such that
P(a)plI(a)
>
(P' ( a)) 2 ,
then P has at least two nonreal zeros. 58.
Consider the equation aox n + a 1 Xn - 1 + . . . + an
=
0
with real coefficients ai . Prove that if the equation has all of its roots real, then (n - l)ar 2:: 2naOa2 . Is the reciprocal true? 59.
Solve the equation X4 - (2m + 1)x3 + (m - 1)x2 + (2m2 + l)x + m = 0,
where m is a real parameter. 60.
Solve the equation x2 n + a1 x2 n - 1 + . . . + a2 n _2X2 - 2nx + 1 = 0,
if all of its roots are real.
SOLUTIONS
Let Nk be the number of words having exactly k characters from the set C, = n - 1. The number that we seek is 1 :s; k :s; m. Clearly, + 2+...+ = Let Ak 2, . . . , k}, 1 :s; k :s; m. We need to find out the number of functions f : Ak ---+ A, k = 2, n with the properties 1.
N {I , 1
N1 N
f(l)
f:. a1
and f(k)
Nm·
f:. a1
For f(l) and f(k) there are n - 1 possibilities of choosing a character from , Cn and for f(i), 1 < i < k there are n such possibilities. Therefore the number C2 , of strings f(l)f(2) . . . f(k - l)f(k) is • . •
Nk
N1It+follows N2 + . .that N .+ m (Dorin Andrica)
=
=
(n - 1) 2 nk-2
(n - 1) + (n - 1) 2 nO + (n - 1) 2 n + . . . + (n - 1) 2 n - 2
1
m
=
2. Suppose, for the sake of contradiction, that there are two sets A and B such that Au B = 2, . . . , 5n }, An B = 0 and the sets contain together less than n pairs ( x, x > with the desired property. Let k be a given number, k = 1, n. If k and 2k are in the same set - A or B the same can be said about the difference 2k - k = k. The same argument is applied for 4k and 2k. Consider the case when k and 4k are elements of A and 2k is an element of B . If 3k is an element of A, then 4k - 3k = k E A, so let 3k E B. Now if 5k E A, then 5k - 4k = k E A and if 5k E B, then 5k - 3k = 2k E B; so among the numbers k, 2k, 3k, 4k, 5k there is at least a pair with the desired property. Because k = 1, 2, . . . , n, it follows that there are at least n pairs with the desired property. Revista Matematica Timi§oara (RMT), No. 2(1978) , pp. 75, Problem 3698)
y) , y,
{I ,
(Dorin Andrica,
3.
Note that (1)
17
18
1. ALGEBRA
and furthermore
for all integers 2:: Suppose that for all integers k 2:: we have Because there are n ! positive integers > such that
m l , m2 1 .
1
n l n2
(2)
I[k ](x) f=. x.
(3) [ ] I Let h n l - in2 0 and observe that for all k the functions k are injective, 1 , n are[ distinct. since numbers ai , From relation (3) we derive that h ] ( ) + n l 2 x l[n2 ](x) , x E [a, b] , or [ h ]) ( ) (10 l[n2 -1 ]) (x) , X E [a, b] . (10 l n2 + - l x I Because is injective, we obtain l[n2+h- l ] (x) l[n2 - 1 ] (x) , x E [a, b] and in the same manner l [h+l ](X) I(x) , x E [a, b] or [h]( ) [ b] l x, x E a, . X Alternative solution. Let Sn be the symmetric group of order n and Hn the cyclic subgroup generated by h] a. It is clear that Hn is a finite group and therefore there is [ integer h such that a is identical permutation. Notice that I[k ](x) { aq[kJ (i) if x �i' i 1 , n x otherwIse h]( ) [ I Then and the solution is complete. (Dorin Anxdrica,x Revista ( 978) , pp. 53, Matematica Timi§oara (RMT) , No. 2 1 =
=
>
=
=
=
=
=
=
=
=
=
Problem 3540) 4.
=
x y z 0, we obtain x + y -z, y + z -x, z + x -y, or, by squaring and rearranging, x2 + y2 Z2 2xy, y2 + Z2 x2 2yz, Z2 + x2 y2 - 2zx. The given equality is equivalent to Z3 Z2 - 2xy + x2 - 2yZ + y2 - 2zx = -x3 + -y3 + -, -z -x -y yz zx xy Because + +
=
=
_
---
=
=
=
_
=
19
1.2. SOLUTIONS
and consequently to
Xy + -yz + zx ) -X3 + -y3 + -Z3 . - (x +y + z) + 2 (z x Y yz zx xy The last equality is equivalent to 2 (X2y2 + y2z2 + Z2 X2 ) x4 + y4 + Z4. 0 we obtain (x + y + Z)2 0 or On the other side, from x + y + z x2 + y2 + Z2 - 2 (xy + yz + zx) . Squaring yields 4X + y4 + Z4 + 2 (X2 y2 + y2 z2 + Z2 X2 ) 4(X2 y2 + y2z2 + Z2 X2 ) + 8xyz (x + y + z) =
=
=
=
=
=
or
as desired.
( Titu A ndreescu, Revista Matematica Timi§oara (RMT) , No. 3 ( 1971) , pp. 25, 483; Gazeta Matematica (GM-B), No. 12 (1977) , pp. 501 , Problem 6090) Problem b d 5. We assume that numbers a, , c, are different from zero. Consider the equation xb 4 d- (2: a) x3 + (2: ab) xb2 - (2:dabc) x + abed 0 b d f:. 0 with roots a, , c, . Substituting x with a, , e and and simplifying by a, , c, , =
after summing up we obtain
Because
2: a
=
0, it follows that
b +a,c+thend = 0 , b b d 3bcd. Now d or + c - . It is left to prove that 3 + c3 + 3 b3 + c3 + d3 b3 + c3 - (b + c) 3 -3bc(b + c) 3bcd as desired. (Dorin Andrica, Revista Matematica Timi§oara (RMT) , No. 1-2 (1979) , pp. 47, 3803) Problem b 0 , we obtain 6. Because a + + c a3 + b3 + c3 3abc and a5 + b5 + c5 5abc(a2 + b2 + c2 + ab + bc + ca) If
one of the numbers is zero, say
=
=
=
=
=
=
=
=
20
1. ALGEBRA
The given relation becomes
3abc 5abc(a2 + b2 + c2 + ab + bc + ca) or 2a + b2 + c2 + ab + bc + ca 5'3 b since a, , c are nonzero numbers. follows that 1- [(a + b + C)2 + a2 + b2 + c2] 35 2 b 0 and, using again the relation a + + c = , we obtain a2 + b2 + c2 �5 ' as desired. ( Titu A ndreescu, Revista Matematica Timi§oara (RMT), No. 2 (1977) , pp. 59 , 30 16) =
=
It
= -
=
Problem 7.
Consider the equation
xb 4 -d (L: a) x3 + (L: ab) x2 b- (L: adbc) x + abed 0, with roots a, , e, . Substituting x with a, , e and , respectively, we obtain after summation that L: a4 + (L: ab) L: a2 + 4abed is divisible by L: a. Taking into account that =
we deduce that
L: a. 4Henceb4 4 d ) ( b 2 (ba + + e + 4 - a2 + 2 + e2 + d2 )2 + 8abed d, desired. + + e + is divisible by a (Dorin Andrica) is divisible by
as
8.
The equation is equivalent to
(x2 + 6x + 5)(x2 + 6x + 8)(x2 + 6x + 9) 360 . 2 6 Setting x + x y yields (y + 5)(y + 8)(y + 9) 360 , =
=
=
1.2. 21 or y3 +22y2 +157y = 0, with solutions Yl = 0, Y2 = - 11 +6i , = - 11 - 6i . 6 0 Turning back to the 6substitution, we obtain a first equation, X2 + x = , with 0 solutions = , X2 = -6 . 6i is equivalent to ( +3)2 = - 2 +6i. Setting 11 x = The equation X2 + + +3 = u+i v, u, v we obtain the system { U22uv-=v26= - 2 4 36 40 (2 (2 It follows that u +V2)2 = u -V2)2 + (2UV)2 = + = . Therefore 2 - v2 = -v'W { U2u2 +V2 2 = 2 v'W - 1, v2 = + 1, yielding the solutions and u = = - 3 ± V - 1 ± i V +1 where the signs + and -6 correspond.6i 1 The equation X2 + x = - 1 - can be solved in a similar way and it has the solutions 3 + J - 1 - i V +1 , = -3 - V.Jill - 1 +i V + = 3 ( 97 ) 6 ( Ti A d SOLUTIONS
Y3
Xl
x
E lR,
X
y'lO
V16
X 3 ,4
X5
y'lO
y'lO
V16
y'lO
X6
l.
tu1 55n )reescu, Revista Matematica Timi§oara (RMT) , No. 1 2 , pp. 2 , Problem 2 9. The equation is equivalent to ..;x+Y - + -2vz-=-2+u - +v - ..;v = 0, XVY
or
Vu
z
( - D2 + ( - D2 + (vz-=-2_1)2+ + (.ru-D\ (VV-D2 = 0 ..;xU, v are real numbers, ..;v it follows that vx
v'Y
Because x, y, Z,
= VY = Vu = = � and vz-=-2 = 1 . Hence 1 = 3. X = Y = U = v = 4' ( Titu Andreescu, Revista Matematica Timi§oara (RMT) , No. 2 (1974) , pp. 47, 00 0 ( 974) , pp. 560, Problem 14536) Problem 2 2 ; Gazeta Matematica ( GM-B ) , No. 1 1 Z
22
1. ALGEBRA
10.
Setting
or
X = x + 1 and Y = y - 1 yields (X + y)2 = Xy
0 �[X2 + y2 + (X + y)2] = . 2 0 , so the solution is x - 1 and y = 1 . Hence ( 977) , pp. 40, ( TiXtu =AYnd=reescu, Revista Matematidl Timi§oara (RMT) , No. 1 1 8 ) Problem 2 11 =
1 1.
The equation is equivalent to
� x + V4x + V16x + V/
Squaring the equation yields
..
+ v'4nx + 3 = ..jii + 1
V4X + V16X + V/ . . + v'4nx + 3 2..jii + 1 =
Squaring again implies
V16X + V/ + v'4nx + 3 = 4..jii + 1 Continuing this procedure yields 4nx + 3 = 4nx + 2 · 2n ..;x + 1 1' 4 2 and 2Ti. 2n Vx Hence x = . n 4-5 (1972) , pp. 43, ( tu Andreescu, Revista Matematidl Timi§oara ( RMT ) , No. 385 ) Problem 1 . .
=
12.
We distinguish two cases:
1) b = c. The equation becomes Jx + a + Jx + b = Jx + a, b so x;:=) b-f:.. 2 c. The equation Jx + b +is equivalent Jx + c = Jtox + a + b - c - Jx + a, b-c b-c or Jx + b - Jx + c - Jx + a + b - c+ Jx + a ' so Jx + b - Jx + c = Jx + a + b - c+ Jx + a.
(1 ) (2)
23 1 .2. SOLUTIONS () Summing up relations 1 and (2) we obtain v'x + b = v'x + a + b and then a = To conclude, we have found that b = then b (i ) the equation has the solution x = - . b ( ii ) If b f=. and a f=. there is no solution. ((iii and a = then x = -a is the only solution. Ti)tu Anf=.dreescu, ( 978) , pp. 26, Revista Matematica Timi§oara (RMT ) , No. 2 1 7 30 ) Problem 1 b 1 3. Because a and are distinct numbers, x and y are distinct as well. The second c,
c.
If
If
c,
c
c
c,
c,
equation could be written as
b. We have a a22 b22 = bb22y22 + 2b2y((x24 _ y24))3 + (X4 _ y4 ) 2 a x = x +X X _y .
and the system could be solved in terms of and
Subtracting the first equation from the second yields
which reduces to
3xy2 ) 3 + a = x b = y3 - x2 y a = x3 - xy2 ) . 3 + 3xy2 = x a = xb (x2 _3 y22 ) 3b = y (y2 _ X2 )b ( ) 3 a b ) ( = x y + y . a + = x + y a - = x - y 3. x + y = {!� a + bb x -b y =� a -b)/ (� b � b)/ � ( and (its solution is x = a + + a - 2 , y = 00 a +1 ) - a - 2. Andreescu, Tituunique Korean Mathematics Com petitions, 2 3x + 4 1 3 = y, Y E Z. It follows that 14. Let 3y - 4 13 x= b
b = y3 + 3x2 y
Solving the quadratic equation in yields ( and or ( and The second alternative is not possible because and cannot be both positive. It follows that and Hence The system now and becomes
24
1. ALGEBRA
[ � (3Y : 4) - 2 ] = Y,
and the equation is equivalent to
or
75Y - 126 [ [52] ] =[ Y] Using that for any real number a, a ::; a < a + 1 , we obtain 75y - 126 1 ::; 78 52 < y + , Y 6 <1 3 78 6 1 1 1 2 < 2 2 or ::; y , so 23 ::;y 23· 6 7 Note that y E Z, therefore y = 4or y = , thus 7 13 13 Xl = 1 and X2 = 1 ' are the solutions. A ndreescu, 3 ( 1972) , 25, ( Titudesired Revista Matematica Timi§oara ( RMT) , No. 1552) Problem 1 + V5 15. From a � - - we obtam. a2 - a - I � 0, or a � a1 + 1. We have 1[ + na2 ] = -1 + na - a, O::;a < 1 . a a Hence 1 + [1: na2 ] 1 + � :na - a a = = [( 1 + � - a) � + n] = n, n � O . pp.
[
That is because and
] [
]
(1 + -a1 - a) -a1 ::; (a - a) -a1 = 1 - -aa < 1 (1 + �a - a) �a �a2 O. >
>
( Titu Andreescu, Revista Matematica Timi§oara (RMT) , No. 2 (1978) , 45, 3479) Problem O 3 3 3 3 16. First we consider the case when X + y + z = . Then x + y + Z = xyz and 3 as desired. the ratio equals �, pp.
25
1.2. 2xyxz3-(x +y3++yZ+3 z) 2 SOLUTIONS
Conversely, if
-
then
"3'
and so
2(x3 +y3 +Z3 -3xyz)+3(x+y+z) O. Using the formula x3 +y3 +Z3 -3xyz (x+y+z)(x2 +y2 +Z2-xy-yz-zx), =
=
we obtain by factorization that and so
(x +y+z)[(x-y)2 +(y-Z)2 +(z-X)2 +3] O. Because is positive, it follows that x +y +z 0, desired. dreescu,factor ( TitutheAnsecond Revista Matematica Timi§oara RMT , No. 1(1 973), pp. 30, Problem 1513) =
(
17.
)
=
as
We write the equation as
Xl -2JXI -1+X2 -2· 2VX2 -22 +. .. +Xn -2nvxn -n2 0, or (JXI -1-1)2 + (VXi 2 -22 -2) 2 +... + (VXn -n2 n) 2 Because the numbers Xi, 1, n are real, it follows that ( Titu Andreescu, Revista Matematica Timi§oara RMT , No. 1(1977), pp. 14, Problem 2243) =
_
=
(
18.
=
O.
)
Using the identity
(a+b+c)3 a3 +b3 +c3 +3(a+b)(b+c c+a) we obtain (� + � +1) 3 �X + �Y +1+3 (� +�) (1+ _1_) (1+ _1 _) 9+1 +54 64 )(
=
{IX
Hence
flY
=
=
{IX
=
flY
{IX
flY
=
26
1. ALGEBRA
and so
-1 + -1 = 3 . fIX flY The system is now reduced to �� 9 x+y= 1 + 1 = 3, fIX flY
11
which is a symmetric system, having the solution
x = '8' y = 1
x = 1 , Y = '8'1
and
( Titu Andreescu, Revista Matematica Timi§oara (RMT), No. 4-5 ( 1972) , pp. 43, 386) Problem 1 19.
By summing up the equations of the system we obtain
It follows that
(4x2 - 4x + 1) + (4y2 - 4y + 1) + (4u2 - 4u + 1) + + (4v2 - 4v + 1) + (4w2 - 4w + 1) = O.
Due to the fact that
x,y,u, v,w are real numbers, we obtain 1 x = y = u = v = w = 2 8 ( 977 3 (Dorin Andrica, Gazeta Matematica (GM-B) , No. 1 ) , pp. 21 , Problem 16782) 20.
From the triangle inequality we deduce
+ . . . + laj-l - ajl ::; i) max la· - a"1 < J" - i , 1 <_ < (J' l Si
Hence
�
J-
S < S5
S < S5
So
4 5 a� - 2 � l �
i= l
5
� a"a" < � � J-
i , i=l i>Fi
50
1.2. SOLUTIONS and consequently 5
27
t, a� - (t, a;r � 50
5 5 r L L Note that ai 0 and so i=l a 10, as claimed. i=l ( Titu An dreescu, Romanian Mathematica Olympiad - second round 1979; Revista Matematica Timi§oara (RMT) , No. 1-2(1980), pp. 61, Problem 4094) ( b 4b 2 1 . The inequality a + ) 2 � a yields -a1 + 1b - a +4 b 1 1 4 1 1 and , similarly' -b + - - b ' -e + -a - e +4 a + e e Summing up these inequalities yields 1 1 1 1 1 1 2a + 2 b + 2e � a + b + b + e + e + a' as desired. (Dorin Andrica, Gazeta Matematica (GM-B), No. 8(1977), Problem 5966) =
::;
-
>
--
>
>
--
--
.
2 2 . Using the identities 5a + b5 + e5 = (a + b + e) 5 - 5 (a + b)(b + e)(e + a)(a2 + b2 + e2 + ab + be + 00)
and
we obtain
a53 + bb53 + e5 - (a + bb + e))53 = 5 (a2 + b2 + e2 + ab + be + ca) a + + - (a+ + e It suffices now to prove that b )2 b b 5 ( 2 b2 2 10 3 a + + e + a + e + 00) � 9 ( a + + e 3 (a2 + b2 + e2 + ab + be + ca) � 2 (a2 + b2 + e2 + 2ab + 2be + 2ca). -3
c3
or
The last inequality is equivalent to
which is clearly true.
a2 + b2 + e2 � ab + be + 00,
( Titu Andreescu, Revista Matematica Timi§oara (RMT), No. 1(1981) , pp. 49,
Problem 4295; Gazeta Matematica ( GM-B) , No. 6(1980), pp. 280, Problem 0-148; No. 11 (1982) , pp. 422, Problem 19450)
1. ALGEBRA
28
2a- �b, 2b- �,c 2c-�a are greater (1) (2a-D (2b-D (2C-D >1 and (2) (2a-D + (2b-D+ (2C-D >3. From the relation (1) and using abc 1 we obtain (3) 3>2(a+b+c)-G+�+D. On the other hand, relation (2) gives 2(a+b+c)-G+�+D >3
Assume by contradiction that all numbers than 1. Then 23.
=
which is a contradiction. The proof is complete. Revista Matematica Timi§oara (RMT) , No. 2(1986) , pp. 72, Problem 5982)
( Titu Andreescu,
24.
Assume by contradiction that all numbers are greater than 1/4. Then 1 1 1 1 �J2 4 4 4 4
a-b2 +b-c2 +c-a-+d-a2> - +- + - +0> G-ar + G-br + G-cr+ G-dr
hence
This is a contradiction so the claim holds. Revista Matematica Timi§oara (RMT) , No. 1(1985), pp. 59, Problem 5479)
( Titu Andreescu,
ai
Setting = sin2 expression becomes 25.
E
=
ai for i
=
1, n, where
aI, a2 , . . . , an are real numbers, the
n
\!sin2 cos2 ai a al· 2: i=1 ai +b n+l =
Using the AM-GM inequality yields
k bf ;::: b1 b2 -k1 2: i=1
bk, bi>0, i U . For b1 sinf ai, b2 cosf ai+l and b3 b4 bk {/2� we obtain k 1 . 2 (sm ai +cos2 ai+l + -2-2-) ;::: 2¥1 \I'sm2 ai cos2 ai+l· =
=
k
=
• • .
=
= ... =
=
k
1.2. SOLUTIONS
29
k 1 , 2, . . . , n yields (k ) l E �k (n + n - 2 ) 2 - 2_-/0 /0_ /ok2 1 � and so E < --2 2- n ·E2 n · 2 {14n Hence the maximum value of is ;; and it is reached if and only if 1 = a2 = . . . = an = 2"' al (Dorin A ndrica, Revista Matematidl Timi§oara (RMT) , No. 1 (1978) , pp. 63, Summing up these relations for =
>
Problem 3266) 26.
Because
x and
m
are positive, we have to prove that
x(xmn 1) - m(xn - 1) � 0, or (xn - l)[(xn ) m-I x + (xn ) m-2 x + . . . + x ] � 0. E( ) ( ) ( ) 2 Define E (x ) = x n m- I x + xn m - x + . . . + x and note that if x � 1 , then xnn �< 11 and E (x ) �< 0, so the inequality holds. In the other case, when x < 1 , we have x ( TiandA dx and again the inequality holds, as claimed. (1978) tu n reescu, Revista Matematica Timi§oara (RMT) , No. 2 , pp. 45, -
-m
-m
°
Problem 3480) 27.
For
m
:::;
can be written as We have
m >
n the inequality is clearly true,{O so1 considerI } n and define = + q with q E , , . . . ,n and the inequality ( !)P (pn + q)! � n .
p = [:]. This implies that
m
pn
(pn + q)! � (pn)! = ( !)P )( ) ( ) ( 1 1 l) ) = · 2 . . . n n + . . . 2 n ... ((P - n + 1 . . . (pn) � n , and (we TituareAdone. ndreescu, Revista Matematica Timi§oara (RMT) , No. 2 ( 1977) , pp. 61,
Problem 3034) 28.
We will use the inequality
Xml + xm2 + . . . + xmn - n_m1-_l (Xl + X2 + . . . + Xn ) m , which holds for all positive real numbers X I ,X2 , . . . ,Xn and all E ( >
m
-00 ,
] [1 ,00) .
0 U
1. ALGEBRA
30
.!. 2 = _ = Now set X l = 1 , X = 2, . . . , X n n and n . We obtain 1 1 1 1 [ n (n + 1) ]�2 = n - -' > -� l 1+v'2 + y'3 + · · · + \Iii -n+1 2 n as desired. ( Titu Andreescu, Revista Matematica Timi§oara (RMT) , No. 2(1974), pp. 52, m
1
n
Problem 2035) 29.
From the AM-GM inequality we deduce
i � IT � 1 = \in + 1, i=l or 1 n 1 \i 1 + - 2: -;- � n + 1, n i= l and so ( \i > n n + 1 - 1) ' 1 + ! + ! + . . . + .!. n 2 3 i+1 as desired. . · IS · stnct b ecause the numb ers -. · Ob serve that the mequa 1Ity nt i=l
.!.
i
�1 �
n
�
�
�
.
�
=
1 - are
,n,
distinct. In order to prove the first inequality we apply the AM-GM inequality in the form 1 2 1 + - + - + · · · + -1 2 3 > n \Iii
n
Therefore
or
n-1 n
if!n = _
_
.
n (1 - _\Iii1_ ) + 1 > 1 + �2 + . . . + .!..n
(Dorin Andrica, Revista Matematica Timi§oara (RMT) , No. 2(1977) , pp. 62,
Problem 3037)
al ,a2 , . . . ,an are positive, from the AM-GM inequality al a2 + . . . + an--- � -tlal a2 . . . an -n ) ( we deduce that tn ::; ala2 . . . an . Using that numbers ai are less than 1 we obtain ( ) n-1 logai tn � -- logai ala2 . . . an . n 30.
Because the numbers +
n
;;: l
1. 2. SOLUTIONS 31 Summing up these inequalities yields n n ) n- 1 loga; tn loga; (a1 a2 . . . an = 2: 2: n i=1 i= 1 ) ( ) ( n-1 -- [n + logal a2 + loga2 a1 + . . . + logal an + logan a1 + . . . + n )] ( + logan an - 1 + logan_l an · 1 Note that a + a > 2 for all a > 0, so 2] (n - l)n, n-1 � logai t n -[n + 2(n - 1) + 2(n - 2) + . . . + n i=1 as claimed. (Dorin Andrica, Revista Matematica Timi§oara (RMT) , No. 2(1977) , pp. 62, 2::
=
-
L.J
2::
=
Problem 3038)
31 . We begin with the following lemma. Lemma. >
Let a b be two positive integers such that 1. Then between numbers a and b �there- � is at>least perfect cube. Proof. Suppose, for the sake of contradiction, thata there is no perfect cube between that a and b. Then there is an integer c such b c3 � < a � (c + 1) 3 .
This means
c � � < � � c + 1, so which is false. 0 Now we can easily check that for n = 10, 11, 12, 13, 14, 15 the statement holds. If n 2:: 16, then 1 1 n > (2, 5) 3 = > �_ (1, 4 - 1) 3 ( 1) 3 ' or 1 aC y n > �_( Hence
{I3n - -rn > 1,
and using the above lemma the problem is solved. Revista Matematica Timi§oara RMT , No. 1-2(1990), pp . 59, Problem 4080)
( Titu Andreescu,
( )
32
1.
k(k2+
ALGEBRA
1) IS odd for 1 or Note that the number even for where is a positive integer. or We have the following cases: then i) if 32 .
k= 4p,
k=4p + 3 n= 4m,
k= 4p + k= 4p + 2
.
p
Sn= n k= m-lL..J (-4p - 1 - 4p - 2 + 4p+ 3 + 4p + 4) = 4m. n= 4m + 4m - (4m + = S = n n= 4m + 2, 4 (4 (4 (4m + 3) . m + m m + 2) S = = n n=4m + 3 4 (4 (4 (4 3) O Sn= m - m + - m + 2) + m + = . 4m n=4m 4m + n= 4 Sn= �( m + 3) n=4m+2 4m + 3. n = (D i A d i "".,. (-1)
L..
k=1
ii) if
and IS .
""
k(k+l) 2
p=O
1, then
1)
- 1.
then
iii) if
1)
iv) if
then
Hence
or 4303 n n) r ca,
{
1)
if if 1 if if Revista Matematidl Timi§oara (RMT) , No. 1(1981), pp. 50, -1
Problem 33.
for
a) Summing up the identities
(n +k 2) - (n +(n2 + 2k)(n)(n ++ k) (n)k k= 0 k=n 1 Sn= (n + 2)(n + (�(n;2) - (�:�) - (�:�)) = 3) n 2 + +2 n ( n (n + 2)(n + [2 +2 - (n + 2) - (n + 2)(n + to
1) 1-
yields
1)
1
1) b) Summing up the identities
-
1]
_
1) .
(n +k 3) - (n + 3(n +k)(3)(n +n 2+ -2)(nk)(n+ + k) (n)k k= 0 k= n n - (n + 3)(n + 2)(n + 1)
1-
-
for
to
yields
1'. _
1
1)
1 . 2.
SOLUTIONS
. (� ( ) ( )
(
33
) ( )) )
n+3 n+3 n+3 n+3 = 3 k n + 1 n + 2 n + k=O 1 2n 3 � (n2 + 3n + 2) = = 2 (n + 3) (n + 2) (n + 1) n+ 2 2 - (n + 3n + 2) - 2(n + 3) (n 2) (n + 1) . Revista Matematica Timi§oara (RMT) , No. 2(1975), pp. 43, _
_
(Dorin Andrica,
Problem 2116)
( 4
_
+
_
_
+
Let Sn be the number in the statement. It is not difficult to see that l Sn = 2 + v'3 + 2 - v'3 34.
� [(
t<+ (
H
t
].
The required property says: there exists k > 0 such that Sn = (k - 1) 2 + k2 , or, equivalently, 2k2 - 2k + 1 - Sn = The discriminant of this equation is Ll = 4(2Sn - 1), and, after usual computations, we obtain n+ l nH 2 ( 1 - v3) 2 ( 1 + v3) 2 �=
(
O.
;
)
Solving the equation, we find n+ l n+ l 2n 1 + ( 1 + v'.3) 2 + ( 1 - v'3 ) 2 k= 2n+2 Therefore, it is sufficient to prove that k is an integer. Let us denote is an integer for ( 1 + .J3) + ( 1 - .J3) where m is a positive integer. Clearly, [ all m. We will prove that 2 divides m = 1, 2, 3, . . . Moreover, the numbers satisfy the relation = 2 -1 + The property now follows by induction. Romanian IMO Selection Test, 1999)
+
m
Em
(Dorin Andrica,
35.
where
Em
Em
Differentiating the identity
sin nx = sinn X yields
m, ] � Em , Em Em 2Em-2•
((�) cotn-1 X - (�) cotn-3 x + (�) cotn-5 X )
n cos nx = n sinn - 1 x cos xP(cot x) - sinn X
-
� PI (cot X), sm X
• • •
34 For
1. ALGEBRA
x = � we obtain
7f
n cos n "4 Because
n ( P (l) P' ( )) ( ) 0. n -2 1 . = 2
nP (l) = n (�) - n (;) + n (�) - . . . and ' - 2P (1) = - 2 (n - 1) (�) + 2 (n - 3) (;) - 2 (n - 5) (�) + . . . , we have ' nP( 1) - 2P (1) = - [(n - 2) (�) - (n - 6)(;) + (n - 10) (�) - . . . J = = -n ((�) - (;) + (�) - . . . ) + 28n· To conclude, use that
hence
Sn = n (0.) n (cos n + sinn) 2 4 4 7f
7f
(Dorin Andrica, Revista Matematica Timi§oara (RMT) , No. 2 ( 1977) , pp. 3 00) Problem 2 36. Differentiating with respect to x the identities (x + l) n = (�) + (�) X + . . . + (�) xn and (x - l) n = (�) xn - ( � ) xn- 1 + . . . + (- 1) n (�) n 1 yields and
n (x _ 1) n- 1 = n (�) xn- 1 - (n - 1) (n � 1) xn-2 + . . . + ( _ 1) n- 1 (�). MUltiplying by x gives nx (x + 1) n-1 = (�) x + 2 (;) X2 + . . . + n (�) xn and ' nx (x _ 1 )n-1 = n (�) x" - (n - 1) (n � l) x"- + . . . + ( _ 1),,-1 (�) x.
Differentiating again we obtain
1.2. SOLUTIONS
35
n (x + l) n- l + n (n - l) x (x + 1) n-2 = (�) + 22 (�) x + . . . + n2 (�) xn - 1 n (x - l) n- l + n (n - l) x (x 1 ) n-2 = n2 (�) xn- 1 - (n - 1 ) 2 (n � 1) xn- 2 + + . . . + (_ W - l (�) Setting x = 1 yields 12 (�) + 22 � + . . . + n2 (�) = n (n + 1 )2n-2 -
and
()
Summing up the last two identities gives
as desired.
Sn = 12 (�) + 32 (�) + . . . = n (n + 1)2n- a,
(Dorin Andrica, Revista Matematica Timi§oara (RMT) , No. 1 (1978) , pp. 90, 3438) Problem 37.
Note that
2n k] n_ 1 2i+1_1 k] ] 2: [log2 = 2: 2: [log2 + [log2 2 n , k=lk 2 i=O k=2i i k] 2 and [log2 = for i < i+1 . Hence 2n � k] n-1 i 2i 2n] 2)2n 2 k2:=l g2 = 2: i=O . + [log2 = (n - + n + as claimed. (Dorin Andrica, Revista Matematica Timi§oara (RMT) , No. 2 ( 1981 ) , pp. 63, 4585; Gazeta Matematica (GM-B) , No. 2-3 (1982) , pp. 83, Problem 19113) Problem 22n - 1 for all positive integers n. Then 38. Let Yn = (22n )2 - 2 . 22n + 1 1 2 2 � _ __ 22n Yn Yn+1 ( 2n 1-) 1 22n +1 -2 1n (22n - 1)(22n+ 1 - 1 ) _- (22n _21 )(_22n +21 - 1 ) - (222n ) 2--1 1 _- 22n 1+ 1 -_ x1n and therefore 1 = -1 - -2 ::;
O
_
1. ALGEBRA
36
Xn Yn Yn+l Summing up these relations yields 2 2n-1 = 1 - -2n < 1 -X1l + -X22 + -X32 + . . . + -Xn YI Yn+l YI for all integers n, desired. (Dpositive ori4n135)Andrica, Revista Matematica Timi§oara (RMT) , No. 1-2 ( 1980) , pp. 67, Problem i 39. Substituting z with z in the relation J(z)J(iz) = Z2 yields J(iz )J( -z) _ Z2 . Summing up gives J(iz)(J(z) + J( -z)) = 0, J(i ) 0 J( ) J( ) o. so z = or z + J(-z)J(= J(z) = 0 if and only if z = From the relation that zJ(i i)z) =0 Z2 we deduce O. Hence 0 J( ) J( ) = 0 and, if z = 0, then + -:f. and so if z -z , then z z -:f. J(z) + J( -z) = 2J(0) = O. Clearly, J(z) + J( -z) = 0 for all numbers z E desired. Remark. A function J J( )J(i ) 2 J( ) satisfying the relation z z = Z is z = (- �( Ti + iA�d) z. tu 583n )reescu, Revista Matematica Timi§oara (RMT) , No. 2 ( 1976) , pp. 56 , Problem 2 40. Setting X = Y = 1 yields J2 ( 1 ) + J2 ( a ) = 2 J ( 1 ) (J(l) - 1) 2 = 0 so J(l) = 1. Substituting Y = 1 gives and . J(x)J(l) + J (�) J(a) = 2J(x) -
as
=
C,
:
C
-+
C
or
Y Xa and observe that J(x)J (�) + J (�) J(x) = 2J(a) .
Take now =
-
as
1. 2. SOLUTIONS f(x)f (;) = 1,
Consequently,
37
f2 ( ) = 1 , x > o. = y = 0, that gives 12 (,ji) + 12 (�) = 2/(t)
therefore x Now set x
f( ) = 1 ( ) 12 ( 1977) , 45, 10 ( 1980) , 439, 18455) f
and because the left-hand side is positive, it follows that is positive and x for all x. Then is a constant function, as claimed. Revista Matematica Timi§oara RMT , No. pp. Problem Gazeta Matematica (GM-B , No. pp. Problem
f ( Titu Andreescu, 2849;
)
The function is not periodical. Suppose, by way of contradiction, that there is a number > such that 41 .
T 0 f(x + T) = f(x) or sin[x + T] = sin [x] , for all x E JR.
It follows that
[x + T] [x] = 2k (x)7r, X E 1R, -
where k : JR -+ Z is a function. Because all x E JR and therefore
7r
is irrational, we deduce that
[x] = [x + T] for
all
x E JR
which is false, since the greatest integer function is not periodical. Revista Matematica Timi§oara RMT , No. Pro blem
(Dorin Andrica, 3430)
42 .
k(x) = 0 for
( )
1 ( 1978) , pp. 89,
Considering the determinant
3 32 nn2 3n nn we have In 1 122 13 I S (i , j)1 = 8 · 21 2 23 2 n = 82 , �= 2 n3 n n n1 n 8 because the second determinant is obtained from by interchanging rows and columns. 12 8= 1 In
2 22 2n
38
1 . ALGEBRA
On the other hand, 1 1
8 = n!
1 2
1
1 3
n
1n- 1 2n - 1 3 n- 1
n
n- l
(here we used the known result on Vandermonde determinants). Therefore
(Dorin Andrica, Revista Matematica Timi§oara (RMT) , No. 1(1982), pp. 52,
Problem 3862)
43. The determinant is
al � 2n =
0 0
0 0
0
a2 0
b2n0 l b2n 0-
0 0
a3 b2n 2 0 0
0 0
b3 a2n-2 0 0
b02 0
0
b1 0 0 0 0
a2 n - l 2 0 an
Expanding along the first and then the last row we obtain which gives
�2n = IIn (ak a2n k+l - bk b2n k+d k= l -
(Dorin Andrica, Revista Matematica Timi§oara (RMT), No. 2(1977), pp. 90,
Problem 3201; Gazeta Matematica (GM-B) , No. 8(1977) , pp. 325, Problem 16808) 44. a) Adding the last three columns to the first one yields that
x+y+z+v
divides the determinant. Adding the first and second columns and subtracting the last two columns implies that divides the determinant. Analogously we can check that and divide the determinant, and taking into account that it has degree in each of the variables, the determinant equals
x+y- z- v
where
..\
x-y+z-v 4 x-y-z+v ..\(x + y + z + v)(x + y - z - v)(x - y + z - v)(x - y - z + v) ,
is a constant.
39 1 . 2. SOLUTIONS 4 Because the coefficient of X is equal to 1 , we have A = 1 and so x y y x v = (x + y + + v)(x + y - - v)(x - y + - v)(x - y - + v) v x y v y x b ) As shown above, we have b e d a 6.. = b ad d eb = (a + b + e + d) (a + b - e - d) (a - b + e - d) (a - b - e + d) de e ab a 000, the second by 100, 1 On the other hand, mUltiplying the first column by 0 alld these to the fourth, we obtain on the last column the third by 1 b dandb dadding d b b all those numbers are divisible by the the numbers a e , a e, e a , e ao Because 6.. prime numberb it follows d. at least one of the d, a +that d, a - b +ande -therefore d, a - b - divides b - e -divides + e + e + numbers a + ( Titu Andreescu) () () 45. Because the quadratic polynomials h x and t2 X have zeros of the same z
z
w z
z
z
z
z
z
p,
p
p
nature, it follows that their discriminants have the same sign, hence Consequently,
(P1P2 + 4q1q2 )2 - 4(P1q2 + P2Q 1 )2 2:: O .
Note now that the left-hand side of the inequality is the discriminant of the quadratic polynomial and the conclusion follows. Revista Matematica Timi§oara (RMT ) , No. pp. Problem Gazeta Matematica ( GM-B ) , No. Problem pp.
( Titu Andreescu, t 3 2 67;
( 1978) , 63 , 1 5 ( 1979) , 19 1, 1 7740) T 46. Because the quadratic polynomial has nonreal zeros, the discriminant 6.. = b2e2 - 4a(b3 + e3 - 4abe)
is negative. Observe that
6.. = (b2 - 4ae)(e2 - 4ab) < 0 , 6.. b2 4 6.. 2 4 b where 1 =T - Tae and 2 = e - a are the discriminants of the quadratic 6.. 6.. 2 polynomials 1 and • Hence exactly one of the numbers 1 and 2 is negative and 0 since( Ti a > A, thed conclusion follows. tu 8 1n0)reescu, Revista Matematica Timi§oara (RMT) , No. 1 ( 1977) , pp. 40 , Problem 2
1 . ALGEBRA (_ l) n- I an are real numbers, 2 2 . . . . +a + + 47.PObserve that a ·+a and a + . -a l l n (l ) and P( - 1) are real numbers. Hence that is P (l) = P( l) and P(- l) = P( - l) (1) P( ) ( ) ) () Because x = x - Xl . . . (X - x n , the relations 1 become ( 1 - xI ) . . . ( 1 - xn ) = (1 - xI ) . . . ( 1 - xn ) and ( 1 + xI ) . . . (1 + xn ) ( 1 + xI ) . . . ( 1 + xn ) 40
=
Multiplying these relations yields
(1 - x� ) . . . ( 1 - x� ) = ( 1 - x�) . . . ( 1 - x�) , b b b Q (l) Q (l) or ( Ti A d . Therefore l + 2 + . . . + n is a real number. tu 86n4)reescu, Revista Matematica Timi§oara (RMT) , No. 1 ( 1977) , pp. 47, Pro blem 2 P (O) 0 , there is a polynomial Q with P (x) xQ (x) . Then 48. Because Q (k) = k 1 l ' k = 1,n. + d H( ) H(k) 0 for ) H( ( ) ) ( Q l 1 It is clear that eg x = n and Define x = x + x . k all = 1 , n, hence ( 1) H(x) = (x + l ) Q (x) - 1 = ao (x - l)(x - 2) . . . (x - n) (1) yields Setting x = m > n in relation Q ( m) = ao ( m - 1 ) ( m - 2 ) . . . ( m - n ) + 1 . m+1 1 On the other hand, setting x = - in the same relation implies ( _ l ) n+ 1 ao = (n + I) ! Therefore Q (m) (_ l ) n+l (m( - ll)()!(m - 2)1.). . (m - n) + _1_ n+ m+ m+1 and then P(m) = (_ l) n+(l m (ml)-! ( 1) . . . )(m - n) + �. n+ m+1 m977+ )1 3 9 8 (Dorin Andrica, Gazeta Matematica ( 1 2 , Problem ( GM-B ) , No. 16833 ; Revista Matematica Timi§oara (RMT) , No. 1-2 (1980) , p. ,67pp., Problem 4133) 49. We are looking for a polynomial with integral coefficients P (x) = aoxn + alxn-l + . . . + an , ao -:j:. O. =
=
=
=
m,
=
41 1 .2. SOLUTIONS We have ( l) 2 P' (x) = naOxn -l + n - alXn - + . . . + an - l and by identifying the coefficient of x(n - l )n in the relation P(P' (x)) = P'(P(x)), we
obtain or
aonn -l = 1. Hence
a0 - nn1- l and since ao is an integer, we deduce that n 1 and ao = 1. Then P(x) x + aI, 1 and P(P'(x)) = P' (P(x)) yields 1 + al = 1 or al = P' (x) Therefore P(x) = x is the only polynomial with the desired property. ( Titu An dreescu, Revista Matematica Timi§oara (RMT), No. 1-2(1979), Problem 3902) O2 , . . . , On be the roots of the equation 50. Let xn + Xn -l + . . + x + 1 = Of i 1, n. They are all distinct and +l = 1, P (Oi) = 0, l n n . . . Because P(x) is divisible by x + + + x + 1, it follows that x i 1, n, hence Pl ( l) + OlP2 (I) + . . . + Or- lpn ( l) = 0 PI (1) + 02P2 ( 1) + . . . + O�- lPn (l ) = 0 --
=
=
O.
=
(h,
.
O.
=
=
The above system of equations has the determinant
0 on- l O21 O�1 -l V= On on- l 1 n 0 O 0 Because all of the numbers 1 , 2 , . . . , n are distinct, it follows that V -:f. 0 and I ) ( ) ( l . . . 2 so the system has only the trivial solution I dividesPI Pi(1)(X=) forP all =i = 1,=n. Pn = This is just (another way of saying that x Dorin Andrica, Revista Matematica Timi§oara (RMT) , No. 2(1977) , pp. 75, 1 1
O.
Problem 3120; Gazeta Matematica (GM-B) , No. 8(1977), pp. 329, Problem 16834)
42
51.
Consider the determinant
1. ALGEBRA nII+1 ( ) ak - al = an
v=
"' , 1 = 1 "' > 1
an-I , the third by an-2 , ... , the last by ao and P ( a d P ( a2 ) P ( an+ d an2 +1 al� a2� nII+1 (a - a ) v= an + 1 = an k t a a a) �+1 a? P( )a� On the II other hand, ar 0 (mod) p , for all r = 1, n + 1 and an ;j. 0 (mod p) ) ( implies ak - al 0 (mod p . Therefore there are at least two numbers + k 1 1:Sl �n < ai, aj,(D i =Ii j Asuchd ithat ai - aj 0 (mod p) and so ai aj (mod p) , as desired. or n n r ca, Gazeta Matematica (GM-B), No. 8(1977) , pp. 329, Problem 16835) d P( ) 52 . Let m = eg x and let P (x) = aoxm + Q (x) , ao =I 0 d Q( ) If follows that( )eg Px( =) r � m - 1. From pn x = xn we obtain ( a�xmn + (�) a�- l xm n-l ) Q (x) + . . + Qn (x) = aoxmn + Q (xn ) , or S a�xmn + R (x) = aoxmn + (x) , where degR(x) = m(n - 1) + r and degS (x) = nr. Because ao =I 0, it follows that ao = 1 if n is even and ao = 1 or ao = -1 if n is odd. Moreover, degR(x) = degS (x) or m(n - 1) + r = nr and so (n - l) (m - r) = Multiplying the second row by adding all to the first yields
"' , 1 =1 "' > 1
==
==
==
==
.
O.
43 1. 2. SOLUTIONS Q( ) Q( ) This is impossible if x -:f. 0, because n > 1 and m > therefore x 0 and the polynomials are P (x) = xm , for n even and P (x) ±xm, for n odd. Alternative solution. Let degP (x) = m and let P(x) aoxm + alxm- 1 + . . . + am . P( ) Q( ) k If x = xk x with a positive integer, then kn Qn (x) = xkn Q (xn ) or Qn (x) Q (xn ) x P. Assume that P (O) -:f. O . Q Note that satisfies the same condition � am ' Then am 1 if n is even Setting±x 0 in the initial condition yields a and am 1 if n is odd. Differentiating the relations implies pn-1 (x) P' (x) = nP' (xn )xn- l . n (1) ' P O O ( ) Setting now x 0 gives 0 and so am- l . Differentiating again in relation (1) yields analogously a m - 2 0 and then am-3 = am-4 = . . . = ao O. The polynomials are P (x) xm , if n is even and P(x) = ±xm , if n is odd. ( Titu Andreescu, Revista Matematidi Timi§oara (RMT) , No. 1-2(1979) , pp. 59, r,
=
=
=
=
as
=
=
=
=
=
=
=
=
=
=
Problem 3884) 53.
From the relations between the zeros and the coefficients we obtain
am = ( _ l) m L XIX2 . . ' Xm ao
and It follows that
and by applying the triangle's inequality for complex numbers, we deduce that 1
L I ll 1 I l > ( n ) . Xl1 x2 I · · . xnl}-m m I { Consider Xo = min l x l l , X2 , · · . , xn- m . Then
so
Xo < 1,
as
claimed.
44
1. ALGEBRA
( Titu Andreescu, Revista Matematica Timi§oara (RMT), No. 2(1978) , pp. 52,
Problem 3531) 54.
We have
Ln
PI (x) n2 = . -;X · P(x) � i=l Integrating the equation yields n ln IP(x) - xi i = n2 In C lxl ,
L
C>0
i=l
or
n 2 2 In IT IP(x) - xi i = In Cn Ixl n i=l
Hence or
I ll. (F(x) -Xi) I = klxl n' ,
k>0
2 IP(P(x)) 1 = k l xl n . Eliminating the modules gives 2 P(P(x)) = AXn , A E JR.
Therefore P (x) = axn with a E R Revista Matematica Timi§oara (RMT) , No. 1(1977), pp. 47, Problem 2863; Gazeta Matematica (GM-B), No . 1(1977) , pp . 22, Problem 17034)
( Titu Andreescu,
Define Q(x) = xP(x) . Because an -:f. 0, the polynomial Q has distinct real zeros, so the polynomial QI has distinct real zeros as well. Consider H (x) = XQI (X) . Again, we deduce that HI has distinct real zeros, and since HI (x) = x2 PII (x) + 3xPI (x) + P(x) 55.
the conclusion follows. Revista Matematica Timi§oara (RMT) , No. 2(1978), pp. 52, Problem 3530)
(Dorin Andrica,
56.
Let m = degP(x) and let P(x) = ao (x - x I ) (x - X2 ) ' " (x - xm ).
Because P(x) E R[x], X l , X 2 , . . . , X m are distinct zeros. Now P(Q(x)) = (Q (X) - x I ) (Q(X) - X2 ) . . . (Q(X) - Xm )
45 1.2. SOLUTIONS P (Q (a)) 0, we have has a multiple zero a of order k. Since Q (a) Xi ) 0, ao iII( =l Q(a) - xp 0. Observe that and so there is an integer m, such that 1 � � ) Q (x) - Xj has Q (a - xp -:f. 0, for all j -:f. otherwise X'j xp , which is false. Hence ' ' ( ) ) ) ( ( ) ( Q Q Q x has a multiple x x - xp the mUltiple zero a of order k and so concludes the proof. zero (of Dororder in Ankdri- ca,1. This Romanian Mathematical Olympiad - final round, 1978; Revista Matematica. Timi§oara (RMT), No. 2(1978) , pp. 67, Problem 3614) P (x) has less than two nonreal zeros. 57. Assume by way of contradiction that P( ) As a polynomial with real coefficients x cannot have only one nonreal zero, hence P ) ( 2 all of its are real. Let Xl, X , . . . ,Xn be the zeros of x . Then P' ( x ) n 1 P (x ) - � _ _ X �=l x'� and differentiating we obtain P" (x) P (x) - [P' (X)] 2 n 1 P2 ( X ) tt (xP-( X) i )2 Setting x a we reach a contradiction, therefore x has at least two nonreal zeros,( Tias claimed. tu Andreescu, Revista Matematica Timi§oara (RMT) , No. 1-2(1979) , pp. 59, =
m
=
-
p,
p
=
p,
=
=
=
_
=
=
Problem 3883)
P (x) aoxn + alxn- l + an P" , . . . , p(n-2) . Because P (n- 2) (x) (n - 2)! [n (n - l)aox2 + 2 (n - l )alx + 2a2] 2
. + be a polynomial with real coefficients. = Let If all of its zeros are real, then the same is true for the polynomials
58.
.
.
=
is a quadratic polynomial with real zeros, we have
or
- l)a� a a2 P (x) x3 + (a + 1) x2 + (a + l) x + a, [
(n 2: 2n O . The reciprocal is not always true, as we can see from the following example: with
aE(
-
00 ,
-1] U 2, (0).
=
P' ,
46
1 . ALGEBRA ( 2 ( + 1),)( or (a + l) (a - 2) ;::: 0, so the inequality Observe that 2 a + 1) ;::: 2 . 3(a holds. drica) hand, P(x) = x + a x2 + x + 1) does not have all zeros real. D( OnorintheAnother 59.
For m = ° the equation becomes
X 4 - x3 - x2 + X = and has roots Xl = 0, X2 = -1, X3 = X4 = 1. If m -:j:. 0, we will solve the equation in terms of m. We have (2 2 4 2 2xm2 + x - x3 + l)m + X - x3 - x + X = °
and
°
It follows that The initial equation becomes [m Hence
(x2 - x)] [m - 2X x2 - 1 ] = 0.
x2 - x - m = 0, with solutions Xl ,2 = I ± Jl2 + 4m and 2 2mx - 1 = 0, with solutions X3,4 m ± VI + m2 . x (Dorin Andrica, Revista Matematica Timi§oara (RMT) , No. 2(1977), =
-
Problem 3121) 60.
pp .
From the relations between the zeros and the coefficients we obtain
75,
L XIX2 · · · X2n- 1 = 2n and XIX2 · · · X2n = 1 . Hence L2n -1 = 2n, Xk k=l so we have the equality case in the AM-GM inequality. Therefore Xl = X2 = . . . = X2n ' Since Xl X2 . . . X2n = 1 and 2n 1 � Xk > 0, 2 = . . . X2n = 1. we have ( TituXlA=ndXreescu, Revista Matematica Timi§oara (RMT) , No. 2(1977) , 52, =
Problem 2299)
pp.
Chapter 2 NUMBER THEO RY
PROBLEMS 1.
1
How many 7-digit numbers that do not start nor end with are there?
2. How many integers are among the numbers
l·m 2·m
p·m
-n ' n -" " ' -n
where p, m, n are given positive integers?
Let p > 2 be a prime number and let n be a positive integer. Prove that p n n n divides 1p + 2p + . . . + (p l )p . 3.
_
4.
Prove that for any integer n the number 55 n +1
+
55 n
is not prime. 5.
+
1
Let n be an odd integer greater than or equal to 5. Prove that
is not a prime number. 6.
Prove that
5 + 456
34
102002 . [ \1'111] divides 111 . 7. Find all positive integers n such that b 2 3 b2 ) 8. Prove that for any distinct positive integers a and the number 2a ( a
is a product of two integers, each of which is larger than
+
is not a perfect cube.
5 Let p be a prime greater than . Prove that p 4 cannot be the fourth power of an integer. 9.
-
Find all pairs (x, y) of nonnegative integers such that x2 + 3y and y2 + 3x are simultaneously perfect squares. 10.
49
2. NUMBER THEORY
50
1 1 . Prove that for any positive integer
n (17 + 120)
n the number
(17 4v'2 _
_
120)
n
is an integer but not a perfect square. 12. Let
(Un) n� l be the Fibonacci sequence:
Prove that for all integers n 2:: 6 between
Un and un+1 there is a perfect square.
n the number n! +5 is not a perfect square. 14. Prove that if n is a perfect cube then n2 + 3n + 3 cannot be a perfect cube. 13. Prove that for all positive integers
15. Let p be a prime. Prove that a product of 2p + 1 positive consecutive numbers
cannot be the 2p + I-power of an integer.
1
16. Let p be a prime and let a be a positive real number such that pa2 < 4'
Prove that
for all integers n 2:: 1 7. Let
[
[n0i - �] = [n0i + �]
]
a + 1. y'1 - 2avp
n be an odd positive integer. Prove that the set
{ (�) , (�),
contains an odd number of odd numbers. 18. Find all positive integers m and
• • •
,
( n� ) } 1
n such that
19. Solve in nonnegative integers the equation
(:) = 1984.
x2 + 8y2 + 6xy - 3x - 6y = 3 20. Solve in integers the equation
(x2 + 1)(y2 + 1) + 2(x - y) (1 - xy) = 4(1 + xy) 2 1 . Let p and q be prime numbers. Find all positive integers x and y such that
1 1 1 - + - = -. x y pq
2.1. PROBLEMS
22. Prove that the equation
has infinitely many solutions in positive integers such that 23. Find all triples
u and v are both p:
(x, y, z) of integers such that x2 ( y - z ) + y 2 ( Z - x ) + Z 2 ( X - y ) = 2 .
24. Solve in nonnegative integers the equation
x + y + z + xyz = xy + yz + zx + 2 25. Solve in integers the equation
xy (x2 + y2 ) = 2z4 .
26. Prove that for all positive integers n the equation
has integral solutions.
x2 + y2 + Z2 = 59n
27. Let n be a positive integer. Prove that the equations
and
x n + yn + z n + u n = vn + 1
have infinitely many solutions in distinct positive integers. 28. Let n be a positive integer. Solve in rational numbers the equation
x n + y n = x n - 1 + yn - l . 29. Find all nonnegative integers x and y such that x(x + 2)(x + 8) = 3Y. 30. Solve in nonnegative integers the equation
(1 + 31. Solve the equation
xl )( l + yl) = (x + y) ! . x l + y l + z l = 2v . . I
52
2, NUMBER THEORY 32 . Find all distinct positive integers Xl, X 2 , . . . ,Xn such that 1 + Xl + 2XIX2 + . . . + (n - 1 )XIX2 . . . Xn-l = XIX2 . . . Xn· Prove that for all positive integers n and all integers aI, a2 , , an , bl, b2 , . . . , bn the33.number IIn (a� - b� ) k=l • . .
can be written as a difference of two squares. z,
34. Find all integers
x, y, v, t such that ( )( ) X + y + + v + t(= xyv) t + x( + y )v + t xy + + vt = xy v + t + vt x + y . b d such that a and b are relatively 35 . Prove that for all nonnegative integers a, , prime, the system axb - y d 0 x - yt + = 0 z
z
c,
z - c=
has at least a solution in nonnegative integers.
Xl, X2 , ,xp be nonnegative integers. Xl� + X�2 + . . . + x�p 0 (mod p)) x + x + . . . + x 0 (mod p f- l + X�-l + . . . + X�- l 0 (mod p) X k l {I 2 } k ) then there are , E , , . . . ,p , =I l, such that Xk - Xl 0 (mod p . 37. Prove that for any odd integers n, aI , a2 , . . . ,an , the greatest com mon divisor of numbers a I, a2 , . . . ,an is equal to the greatest common divisor of al +2 a2 a2 +2 a3 ' " ' ' an +2 al () 3,8. Let
p
. . •
==
==
==
==
=
53
2. 1. PROBLEMS 40. Let
Pk denote the k-thf primer number.r;:Provemthat p + p + . . . + p > n +l
for all positive integers m and n .
41 . Let n be a positive integer. Find the sum of all positive integers less than 2n and relatively prime with n . 42. Prove that any number between n distinct divisors of n!.
1 and n! can be written as a sum of at most
43. Find the largest value of n such that the complementary set of any subset contains at least two elements that are relatively ith n elements of w prime.
{ I , 2, . . . , 1984}
a, if a2 � n then aln. (Un ) n� l' (Vn )n�l defined by Ul 3 , Vl = 2 and 45 . 3Consider the sequences Un+l1 = un + 4vn , Vn+l[ V22uJn + 3vn , n �1 1 . Define Xn Un + Vn , Yn Un + 2vn , for all n � . n � . Prove that Yn = x n 2 1 46. Define X n = 2,, - 1 + for all positive integers. Prove that 2 (i) X n = XIX 2 . . . Xknl- l + , n E ( ) 1 , , E k -:f. l (ii) Xk ' xz 3 (iii) X n ends in 7 for all n � . ( ) 1 and 47. Define the sequence an n � l by al 2an + J3a� - 2 a +l n 1 for all integers n � . Prove that an is an integer for all n. ( ) (b ) 1, 48. Define the sequences an n �o and n n �o, by ao 2an-l an = 1 + 2an2 _ l and bn = 1 _ 21a2n- l ( ) for all positive integers n . Prove that all terms of the sequence an n �O are irreducible b ( ) fractions and all terms of the sequence n n �o are squares. 3, Yo 2 , ( ) ( ) 49. Define the sequences Xn n �o and Yn n �O by Xo Xn = 3Xn- l + 4Yn-l and Yn 2(Xn)- l + 3Yn- l 1 4 ; ; for all positive integers n . Prove that the sequence zn n �o, where Zn + x y , 44. Find all positive integers n such that for all odd integers
=
=
N,
=
=
=
N
=
=
=
=
=
=
=
contains no prime numbers.
2. NUMBER THEORY 50. Let p be a positive integer and let X l be a positive real number. Define the ( ) sequence Xn n �l by Xn+l Vp2 1xn pvx; 1 for all positive integers Prove that among the first terms of the sequence there are at least [�] irrational numbers. ( ) 5 1 . Define the sequence xn n � o by 2)1) XXnn+l 0 ifX[and] only( -ifl)nx[�0] and for all 2:: o. � Find X n in closed form. ( ) 2 , a3 6 and 52. Define the sequence an n �O by ao 0, al 1, a2 54
+
=
+
+
n.
=
m
n =
+
=
n
=
Prove that n divides
=
=
=
n
an for all > O. 53. Let Xl X 2 X3 1 and X n+3 X n X n+IXn+ 2 for all positive integers =
=
=
=
+
Prove that for any positive integer m there is an integer k > 0 such that
Xk ·
m
n.
divides
(an ) n�O be the sequence defined by ao 0, al 1 and an+l - 3a2n an- l ( _ l )n for all integers > O. Prove that an is a perfect square for all 2:: O. 55. Let al a2 97 and an+l anan- l V(a; - l)(a;_ 1 1), > 1. Prove that 2 2an is a perfect square. a) 2 + J2 2an is a perfect square. b) 2 56. Let k 2:: be an integer. Find in closed form for the general term an of the sequence defined by ao 0 and an - a [ �] 1 for all > O. 57. Let ao al 3 and an+l tan - an- l for 1. Prove that an - 2 is a perfect square for all 1. 2 58. Let a and (3 be nonnegative integers such that a 4(3 is not a perfect square. ) ( Define the sequence xn n �O by 54. Let
=
+
=
=
n
=
n
=
=
+
+
n
-
+
=
=
=
= n
=
2::
n
n
2::
+
2.1. PROBLEMS
55
for all integers n 2:: 0, where and are positive integers. Prove that there is no positive integer no such that
X l X2 X2no - Xno-IXno +l. _
59. Let n > 1 be an integer. Prove that there is no irrational number
Va + Ja2 - 1 + Va - Ja2 - 1 is rational.
+ and - the expression ± 1 ± 2 ± 3 ± . . . ± (4n + 1), yields all odd positive integers less than or equal to (2n + 1)(4n + 1). 60. Prove that for different choices of signs
a such that
SOLUTIONS 1 . The problem is equivalent to finding the number of functions
1 : { I , 2 , 3 , 4, 5 , 6 , 7} {O , 1 , 2 , . . . , 9 } such that 1(1 ) f. 0, 1(1) f. 1 and 1(7) f. l. 1(1 ) E {2, 3, . . . , 9} , there are 8 possibilities to define 1(1) . For 1(7) there Because 9 1(2) 1(3) 1(4) 1(5) 1(6) there are 10 . are possibilities and for 8 9, 0 , 72, 0 , 1 5 . 1 5 numbers with the desired property. To are . . in Andrithere (Dorconclude, 11 ( 1979) , pp. 421 , Problem ca, Gazeta Matematica (GM-B) , No. 17999) d d d 2 . Let be the greatest common divisor of m and n. Hence m m l and n n l for some integers ml and n l . The numbers are 1 · ml 2 · ml p · ml , nl , . . . , n l nl [: ] and, since are relatively prime, there are , integers among them. Because d(m, n)p ] . [ n n gC . mtegers. follows that there are nl d n 11 ( 1 979 ) 429, Problem (DoringcAd(nm,drica,n) ItGazeta Matematica (GM-B) , No. , pp. 0:89) 3 . Define k pn and note that k is odd. Then dk + (p - d) k p[dk- 1 dk-2 (p d) + . . . + (p d) k -l ] d 1 to d [�] implies that p divides 1k + Summing up the equalities from 2k + . . . + (p - l) k , claimed. (Dorin Andrica, Revista Matematica Timi§oara (RMT) , No. 1-2(1979) , pp. 49, 3813) Problem 55n • Then 4. Define m 55n +1 + 55n + 1 m5 m + 1 (m2 + m + 1 )(m3 - m2 1) 57 -+
=
=
--
=
--
ffi" n,
=
=
=
=
_
_
_
=
=
as
=
=
+
=
+
58
2. NUMBER THEORY 1 and, ( since factors are greater than , the conclusion 2follows. dn reescu, Titu Aboth 001) Korean Mathematics Competition, N = (�) - 5 (�) + 52 (�) - + 5n- ' (�) . Then 5. Let 5N = 1 - 1 + 5 (�) _ 52 (�) + 53 (�) _ . . . + 5n (�) = 1 + ( - 1 + W . Hence N = � W + l) = � [C2n + 1 )2 - (2 � r] � � = �" ['2n - 2 + 1'] [2n +" 2' + 1] = ' = � [(2 ;- - 1r + 2n-5] [(2 ;- + 1r + 2n- ] . Because greater than or equal to , both the numerator are 5. One ofn isthem 5, call 5N1 ,factors N1 > 1of, the N2greater than is divisible by it other being N = N1 N2 , where N1 and N2 are both integers greater than 1 , and we are done.. Then ( Titu Andreescu, Korean Mathematics Competition, 2001 ) 4 3 4 4 4 and 6. The given number is of the form m + � n , where m = n=4 4 =2 2 ' "
�
�
•
The conclusion follows from the identity
and the inequalities
4) 1 2 i1 3 2 2 2 _ 1 ( 56 4 i 2 - > m - mn + n > n ( n - m) = 2 4 > 2 2 (2 562- 1 - 22 4 )4 > 2 2 . 251 2 (42 562-1 -5 12 - 1) > 21 0. 5�t1 25 1 2 > 21 0.5 . 210.50 > 103 .5 . 103.50 > 102002 > (Titu Andreescu, Korean Mathematics Competition, 2002) 111 1 3 37 111 7.) The positive divisors of 2 are , , , 7 . So we have the following cases: yl [ ] 1 111 1 1 111 n �4 . 2) [ yl111] == 3, oror 3n 111<
�
.
.
$
$
$
$
�
2. 2. SOLUTIONS
( Titu Andreescu)
8. Note that
59
2a(a2 + 3b3 2 ) = (a + b)3 + (a b)3 -
The Fermat equation for n =
x3 + y3 = z 3
has no solution in positive integers (see T. Andreescu, D. Andrica, "An Introduction to Diophantine Equations" , GIL Publishing House, pp. Hence there is no integer c such that
2002, 87-93) .
if
a > b.
On the other hand, if
b > a then there is no integer
c
such that
This concludes the proof. Revista Matematidi Timi§oara (RMT) , No. Problem
( Titu Andreescu, 1911)
1 (1974) , pp. 24,
4 = q4 for some positive integer q. Then p = q4 + 4 and q > 1 . We obtain ( 2 2q + 2)(q2 + 2q + 2) , p= q 1 a product integers greater than , contradicting dreescu, ( Titu Aofntwo ) fact that p is a prime. 2003the Math Path Qualifying Quiz, 9 . Assume that p
-
_
10. The inequalities
0 � x + y + 8, which is 3 3 2) x2 +2 y 3 (x + 2 2) 2 y2 + x (y + 2) 2 is true. Without 3y = 2 x + 1 . 2x3k 1x2 + 3y 2k(x +1x2)+2 y (x + x2 k+. 3y0 (x + 1 ) 2 , hence 3x = 4k2 + 13k + 4. 2 x y + � y + k> (2k + 3)2 < 4k2 + 13k + 4 < (2k + 4)2 3 k E {O, 1 , 2, 3, 4} , y2 + 3x so y2 + x cannot be a square. It is easy to check that for 0 4 3 k 22 is not a (square but for = , y2 + x = = . Therefore the only solution is ) 1 1 (x, y)( Ti= A, d. ) tu n reescu
cannot hold simultaneously because summing them up yields false. Hence at least one of or < < loss of generality assume that < From < < we derive = Then = + and = for some integer and so If 5, then
2. NUMBER THEORY 1 7 + 1 2v'2 = (v'2 + 1 ) 4 and 1 7 - 1 2v'2 = (v'2 _ 1 ) 4 , so 1 1 . Note that (17 1 2v'2) n - ( 1 7 - 1 2v'2) n ( v'2 1) 4n - (v'2 _ 1 ) 4n 4v'2 4v'2 (v'2 1) 2n (v'2 - 1 ) 2n (v'2 + 1 ) 2n _ (v'2 _ 1 ) 2n 2 . 2v'2 -
60
+
_
_
+
+
+
Define
A = (v'2 + 1) 2n +2 (v'2 - 1 ) 2n
and
B = (v'2 1) 2n2v'2 - (v'2 _ 1) 2n +
U sing the binomial expansion formula we obtain positive integers that
x
and
2n v2 1 + ( ) = X + y v2, (v2 - 1) 2n = X - y v2 Then (v'2 + 1 ) 2n + (v'2 - 1) 2n (v'2 1) 2n _ (v'2 _ 1) 2n 2 2v'2 and y = X= AB is as integer, as claimed. and so
y such
+
Observe that
A B
so and are relatively prime. not a perfect square. We have
It
is sufficient to prove that at least one of them is
A = (v'2 + 1) 2n +2 (v'2 _ 1) 2n = [ (v'2 l ) nv'2(v'2 _ 1) n ] 2 - 1 +
and
Since only one of the numbers
(1)
l ) n - (v2 _ l) n v'2 v'2 ( 1) and (2) we derive is an Ainteger - depending on the parity of n - from the relations that ( D isinotA a dri square. This completes the proof. or4n285n) ca, Revista Matematica Timi§oara (RMT) , No. 1 ( 1981 ) , pp. 48, Problem 7 8 9 6 13 < 16 < 12. The claim is true for n = and n = , because U 6 = < < U7 = 21 U8 = . (v2 +
l) n
+
(v2
_ l) n
+
(v2 +
If n
� 8, then
2. 2.
61
SOLUTIONS
Un - 1 > r,;-;--:-: . � -_ v'UnUn+1 --'-+1 +- UnVU;; >- 2v'Un +1 - 1 = .1Fi v'Un 1 > 1 � 2Y. Un� oJUn- 1 2y 3
y. Un+1 - y Un
-
--=
and so between Un and Un+ 1 there is a perfect square.
(Dorin Andrica)
13. If n = 1, 2, 3 or 4 then n! + 5 = 6, 7, 11 or 29, so it is not a square. If n
then n! + 5 desired.
� 5,
= 5(5k + 1) for some integer k and therefore is not a perfect square, as (Dorin Andrica, Gazeta Matematidl. (GM-B ) , No. 8(1977) , pp. 321, Problem
16781; Revista Matematica Timi§oara ( RMT ) , No. 1 (1978), pp. 61, Problem 3254)
14. Suppose by way of contradiction that n2 +3n+3 is a cube. Hence n(n 2 +3n+3)
is a cube. Note that n (n2 + 3n + 3) = n3 + 3n2 + 3n = (n + 1) 3 - 1 and since (n + 1) 3 - 1 is not a cube, we obtain a contradiction. Gazeta Matematica ( GM-B ) , No. 8(1977) , pp. 312, Problem E5965; Revista Matematica Timi§oara (RMT) , No. 1-2(1979) , pp. 28, Problem 3253)
(Dorin Andrica,
1 5 . Consider the product of 2p + 1 consecutive numbers
P(n) = (n + 1) (n + 2) . . . (n + 2p + 1) Observe that P(n) > (n + 1) 2P+1 . On the other hand, (n + l) + (n + 2) + . . . + (n + 2P + l) 2P+ 1 = (n + p + 1) 2p+1 P(n ) < 2p + 1 from the AM-GM inequality. then E {n + 2, . . . , n + pl . Assume by way of contradiction If P(n) that there is k E {2, 3, . . . , p} such that P(n) = (n + k) 2p+1 . Then (n + 1) (n + 2) . . . (n + k - 1) (n + k + 1) . . . (n + 2p + 1) (n + k) 2P . (1)
]
[
= m2p+1 ,
m
=
We have two cases: I. k p 1) If n == (mod p) , then (n + k) 2p is divisible by p2p. The left-hand side of the equality (1) is clearly not divisible by p2p, hence we reach a contradiction. 2) If n == r (mod p) , then the left-hand side of the equality (1) is divisible by p2 , because of the factors n + p - r and n + 2p - r, while the right-hand side is not, since (n + k) 2p == This is a contradiction. II. k E {2, 3, . . . , p - l }
= 0
r f. 0 , r2 P .
62
2. NUMBER THEORY
1) If n - k (mod p) , then the right-hand side of ( 1) is divisible by p2p , but the left-hand 2) nside -qis not.(mod p) , q f. k and q E {O , 1 , . . . ,p - I} , then the left-hand side of ( 1 ) is divisible by p. On the other hand (n + k)2p (q - k) 2 (mod p) � 0 (mod p) , 0 I kl because < q - < p. Both Andend ir ca)up in contradictions, so the problem is solved. (Dorincases 16. It suffices to prove that there are no integers in the interval � y'P � vp (n - n ,n + n] for n � [ JI -a2avp] + 1 . k Assume by way of contradiction that there is integer such that ny'P - -an < k <- ny'P + -.an Hence 2 2 n2p + an - 2ay'P < k2 <- n2p + an + 2ay'P. 1 a2 + 2a..;p < 1 an 2 ..,fP 1 a Observe that 2 - a > - . If n � [ J1 2 vp ] + , then n -a so 2p - 1 < k2 < n2p + 1 n k2 pn2 or vp � , which is false, since p is prime. It follows that (Dorin Andrica, Gazeta Matematica n (GM-B) , No. 8 ( 1977) , pp. 324, Problem 16804) 1 the claim is clear, so let n � 3. 17. For n Define Sn (�) + (;) + . . . + ( n�l) . Then ==
If
==
==
2"
2"
2"
2"
=
=
=
=
S!,- 2n -1 - 1 .
Sn
Sn
= or contains an odd is odd it follows that the sum Because number of odd terms, as desired. Revista Matematica Timi§oara (RMT) , No. pp. Problem
( Titu Andreescu, 5346)
2(1984) , 71 ,
) . we can assume that m :O; [iT : ( ': ) ( 0 , then 1 1984m, false.n If n 1 , then m 1984. If n 18. Because =
=
=
=
=
2.2. SOLUTIONS
63
m(m - 1) = 1984, with no integral solutions . n If n :::: 3, then (:2) :::: (�), so 1984 m(m - � (m - 2) . Hence 11904 :::: m3 - 3m2 + 2m or (m - 30)(m2 + 27m + 812) � -12456 < 0, and so m < 30. This m (m - 1) . . . (m - n + 1) does not contam. the . that (m) 198 4 , because implIes n n., factor 31 of 1984. To the solutions are m = 1984, n 1 and m = 1984, n 1983. Andreescu, ( Tituconclude, Revista Matematica Timi§oara (RMT ) , No. 1 (1985) , pp. 80, If = 2, then
::::
-Ir
=
=
Problem 5)
1 9 . The equation is equivalent to
(x + 2y) (x + 4y) - ( + 2y) =
3x
or
(x + 2y) (x + 4y
{
3,
- 3) = 3.
We have ( I')
or
(" ) {
X + 2y
x+
=
3 4y - 3 = 1
, with solution ( x, y)
=
(2, -21 ) (
)
x + 2y = 1 ' ( x, y ) = - 4, -5 . , WI. th soIutlOn = 2 x+ Note that there are no solutions in integers, as claimed. Revista Matematica Timi§oara (RMT) , No. 1 (1971), pp. 20, Problem 312) 11
4y - 3 3
( Titu A ndreescu,
20. The equation is equivalent to
22
2 2
x y - 2xy + 1 + x + y - 2xy + 2(x - y) (l - xy) = 4, or
2
2
(xy - 1) + (x - y) + 2 (x - y) (1 - xy) = 4. Hence (1 - xy + x - y) 2 = 4 and, consequently, 1 (1 + x) ( l - y)1 = 2. We have two cases: I. (1 + x) ( l - y) = 2. Then a) 1 + x = 2, 1 - y = 1, so x = 1, y = b) 1 + x = -2, 1 - y = -1, so x = y = 2. c) 1 + x = 1, 1 - y = 2, so x = y = -1 . d) 1 + x = -1, 1 - y = -2, so x = -2, y = II. (1 + x) ( l - y) = -2. Then
0.
-3,
0,
3.
2 . NUMBER THEORY
64
a) 1 + b) 1 + c) 1 + d) 1 +
x = 2, 1 y = -1, so x = 1, y = 2. x = -2, 1 - y = 1, so x = -3, y = O. x = 1, 1 - y = -2, so x = 0, y = 3. x =d -1, 1 - y = 2, so x = -2, Y = - l. A Ti ( tu n reescu, Revista Matematica Timi§oara (RMT) , No. 4-5(1972) , pp. 43, -
Problem 1383)
2 1 . The equation is equivalent to
We have the cases: 1 + pq, pq(l + pq) . 1) pq p2 q2 , so pq 1, 2) - pq p, - pq pq2 , so p( l + q) , pq( l + q) . 2 3) q(l + p), pq( l + p) . pq q, - pq p q, so 4) - pq p2 , - pq q2 , so p(p + q) , q(p + q) . 2pq, 5) pq pq, - pq pq, so 2pq. The equation is symmetric, so we have also: 6) pq( l + pq), 1 + pq. 7) pq( l + q) , p( l + q) . 8) pq( l + p) , q( l + p). 9) q(l + q) , p(p + q) . ( Revista Matematica Timi§oara (RMT) , No. 2(1978) , pp. 45, Problem 3486)
X= y= X= y= x= y= X= Y= x= Y=
x = y = x = y = x = y = x = Y = x = y = x= y= x= y= x= Y= Tixtu=Andreescu,Y = -
-
-
-
Xn+l = 2xn + Yn
for all � l . By induction we obtain that
n
and
Yn+l = Xn + 2Yn
3n = xn2 - yn2 ' n _> 1 Denote by Pk the k-th prime number. Then x = xPlo ' Y = YPlo ' U = 3, v = Pk is a solution of equation UV for any integer k > O. Alternat ive solutixon2 -. Lety2 p= and q be two arbitrary primes, p � 3. Then pq = 2k + 1, for some positive integer k. Because 2k+ 1 = (k+ 1) 2 - k2 , it follows that all quadruples (pq + 1 pq - 1 ) . the equatIOn. . ( x, y, u, v ) = -- ' -2- ' P, q satIsfy D( orin Andri2ca) 23. The equation is equivalent to
(
x - y) (x - z) (y - z) = 2.
65
2.2. SOLUTIONS
2 Observe that (x - y ) + (y - z) = x -z. On the other hand, can be written as a distinct2 integers in the following ways product2 of three 1 1 i) 2= ( - ) · 2( - ) · , 1 1 ii) 2 = . . , 1 1 2 iii) = ( - ) · · ( - ) .
{ xx -- zy == l2l y-z= 2 { xX -- yz == l--1 y-z=l { xx -- yz == -21
Since in the first case any two factors do not add up to the third, we only have three possibilities: a) b)
c)
x,y, z) = (k + 1 , k , k - 1) for some integer k ; k k 1k 1k so (x, y,z ) = ( - , + , ) for some integer ;
so (
so (
x, y, z) = (k , k - 1 , k + 1) for some integer k .
=- . ( Titu yA-ndzreescu, Revista Matematidl. Timi§oara (RMT) , No. 1-2(1989) , pp. 97, 2 Problem ) 24. We have
xyz - (xy + yz + zx) + x + y + z - 1 = 1 , and, consequently, 1 1 1 1 ( x - ) ( y - ) (z - ) = . Because x, y, z are integers, we obtain I = y - 1 = z - 1 = 1, x 2 so x (=Tiy =Az =d . tu n reescu, Revista Matematica Timi§oara (RMT), No. 3(1971), pp. 26, Problem 487)
25. Multiplying by 8 yields
8xy (x2 + y2 ) = 16z4 or 4 (x y ) 4 = ( 2 z ) 4 , (x + y ) and so 4 24 4 ( x - y ) + ( z ) = (x + y ) . This is Fermat's equation for the case n = 4 and it is known that this equation has 2 0 0 solutions only if x -y = or z = (see T. Andreescu, "An Introduction 2002D., Andrica, to Diophantine Equations" , GIL Publishing House, 85-87) . _
_
pp.
2. NUMBER THEORY ± 0 Case I. x - Y = , then x = Y and Z = x. The solutions are x = y = m, z = ±m for any integer m. 0 0 O ( )4 ( )4 Case II. z = , then x - y = x + y and so x = or y = . The solutions are x = O , y = m, z = o and x = m, y = O, z = o for any m. Andreescu, ( Tituinteger 1 ( 1978) , Problem Revista Matematica Timi§oara (RMT) , No. 2813; Gazeta Matematica (GM-B), No. 11 ( 1981) , pp. 424, Problem 0: 264) ( ) ( ) ( ) 26. Consider the sequences Xn n �l ' Yn n �l ' Zn n � l , defined by Xn+2 = 5592xn ' Xl = 1 , X2 = 14 Yn+2 = 59922Yn ' YI = 73, Y2 = 4239 Zn+2 = zn , Zl = , Z2 = 1 for all n � . � + y� + z; = 59n , for all integers n � 1 . It is easy to check that x (Dorin Andrica, Romanian, Mathematical Olympiad - second round, 1979 , Revista 1 2 ( 1980) , pp. 58, Problem 4075) Matematica Timi§oara (RMT) , No. 66
If
If
27. Observe that the equation
( )
has infinitely many solutions in distinct nonnegative integers, for example
1
k � 0. (Xkl ,Ykl , Zkl ) and (Xk ' Yk ' Zk ) be two solutions of equation (1 ) with kl f. k2 . 2 2 2
for any integer Let Then
and multiplying yields
(Xk1 Xk ) n + (Xkl Yk )n + (Ykl Xk )n + (Ykl Yk ) n = (Zkl Zk )n- 1 2 2 2 2 2
This means that
2.2. SOLUTIONS
67
is a solution to the equation
kl f. k2
Since are arbitrary positive integers, the conclusion follows. For the second equation, the proof is similar, based on the fact that the equation has infinitely many solutions in distinct nonnegative integers, for example Xk
k � O. (Dorin Andrica)
for any integer
= 1 + kn ,
Yk
= k ( l + kn) ,
Zk
( 2)
= 1 + kn ,
x = 0, = 0 is a solution to the equation n + yn = xn -1 + yn- l x f. 1 Let a - be a rational number such that y = ax. Hence 28. It is clear that
Y
so
1 1 x = 1++aann- 1 ' y = a 1++aann-1 U sing the symmetry of the equation, we also have the solution 1 + an - 1 1 + an -1 x = a 1 + an , y = 1 + an f. 1 with a - rational. 1 , then again x = y = O. This concludes the solution. drica,n >Revista 2 (1981 ) , pp. 62, (Daor=in-A1nand Matematica Timi§oara ( RMT ) , No. 4578 ) Problem 3 2 3v 8 3 29. Let x = u , x + = , x + = t so U + + t = y. Then 3v - 3u = 2 and 3t - 3u = 8. It follows that 3U (3V-U - 1 ) = 2 and 3U (3t-u - 1) 8. 0 1 2 1 23 8 Hence = and 3v - = 3 , t - 1 = , therefore = , t = . 1 The is x = , y = . Andreescu, 2 ( 1978) , pp. 47, ( Titusolution Revista Matematica Timi§oara ( RMT ) , No. 281 2; Gazeta Matematica (GM-B) , No. 1 2 (1980) , pp. 496, Problem 18541 ) Problem l 2 1 l 1 l 30. If x, y � then + x and + y are both odd and (x + y ) is even. Hence If
v
=
u
the equation has no solutions.
v
2. NUMBER THEORY 1 . The equation becomes Consider the case x 2 ( 1 + y l) (1 + y) l 2 . If y � 3, then 3 divides ( 1 +y) l but and 2it( is notl ) difficult to notice the solution y 1 + y and y 1 does not2 satisfy the equality. not 2 or x , 1 due to the symmetry of the equation. 1 Hence x , y ( Titu Andreescu, Revista Matematidl. Timi§oara (RMT) , No. 2 ( 1977) , pp. 60, 3028; Gazeta Matematica (GM-B), No. 2 ( 1980) , pp. 75, Problem 0 : 118) Problem 3 1 . Without loss of generality we may assume that x � y � z. The equation is equivalent to l [x (x - 1 ) . . (z + 1 ) + y (y 1 ) . . . (z + 1) + 1] 2vl . z 3 3 If z2 � , then the right-hand side is divisible by but the left-hand side is not, so z � . We have two cases. 1 . Then we have l ! v I. z x + y 2 l - 1, or ! [x (x - 1 ) . . . (y + 1) + 1] 2vl - 1. y y1� 2, then the right-hand side is an even number but the left-hand side is odd, so y . Then l 2 ( 2V I - 1 - 1 ) . x 4 2 (2VI -1 - 1) 0 (mod 8) , false. If x � , then 1 , x 2, and x 3. the cases x It remains to examine V 2 2 ) ( x 21 , then 21 2 ( 2VII --11 - 11) , impossible. ! 2 2. If x 3 , then 2v orI v 3 , so v 3 x , then l -1 - 1 or v , false. Hence the only solution in this case is 2, 1 , z 1 , v 2 x 2. Now we have II. z xl + y l 2 v l - 2 , or l [x (x - 1 ) . . . (y + 1) + 1] 2 (2VI - 1 - 1 ) . y 4 2 (2VI - 1 - 1) 0 (mod 8) , false. If y � , the 1 , y 2 , or y 3. It follows that y l 2v 1 l - 3. Since x � 2 implies 2vl - 3 0 (mod 2) false, then If y 2, then x x 1, v 2 . l 2vl 4 3 so xl 4(2VI -2 - 1) . If y 3 , then x4 ( 2 V We must have v � . I-2 - 1 ) 0 (mod 8) , false. If x � , then 68
=
=
=
=
=
Y =
=
=
.
=
-
=
=
=
If
=
=
==
If
=
=
=
=
If
=
=
=
=
=
=
=
=
=
=
Y =
=
=
=
=
==
=
=
=
=
=
==
=
=
=
=
=
==
2.2. SOLUTIONS 69 2 3 1 Hence x3 , x l or2vx 8 and all those3cases lead to a contradiction. , then5 x l - . Then5 v � and xl 23 (2VI -3 - 1) � 623 7. It If y follows x � and because x does not yield a solution and x � implies 23 (2v! -3that - 1 ) 0 (mod 16) , which is false, we do not obtain a solution here. In case II we have found only x = 1 , y = 1 , 2, v 2 , which does not satisfy the condition x � y � To conclude, we have the solution - from case I x 2, Y 1 , 1 , v = 2 and, due to the symmetry of the equation, we also have x 1 , y 2, 1, v 2 and 1 , y 1 , 2, v 2. x ( Titu Andreescu, Revista Matematidl. Timi§oara (RMT) , No. 2 ( 1981) , pp. 62, 4576) Problem =
=
=
=
=
=
•
=
==
z =
=
z.
=
=
z =
=
=
z =
=
=
=
z =
=
32. The equation is equivalent to
or
[X2 . . . Xn - (n - l) x2 . . . Xn-l - . . . - 2X2 - 1] 1 . X I 1 Hence X l = and 2 [X3 . " Xn - (n - 1) x3 ,Xn-l - - 3X3 - 2] 2. X 2 and f:. Since X2 Xl, it follows that X2 Xf:.3 [X4 Xn -f:.(n - l) x4 Xn- l - 3 - 4X4 - 3] 3. Because X 3 X2 and X3 Xl , we obtain X3 . Continuing with the same procedure we deduce Xk k for all k. Remark. Turning back to the equation we find thethatidentity 1 + 1 1 . 1 + 2! . 2 + . . . + (n - 1) ! (n - 1 ) nIno ( Titu Andreescu, Revista Matematidl. Timi§oara (RMT), No. 3 ( 1973) , pp. 23, 1 509) Problem 1 the claim is true. Using the identity 33. We proceed by induction on n . For n (X2 - y2 )(U2 v2 ) = (xu + YV) 2 - (xv + YU) 2 =
• •
•
. •
=
•
. •
=
=
.
.
. •
•
.
=
=
=
=
_
70
2. NUMBER THEORY
and the fact that the claim holds for we deduce that the property is valid for n + as desired. We have n n n = + = Pn =
n
Alternative solution. II (a� b� ) II (ak - bk ) II (ak bk ) k= l k= l k= l
1,
-
where An , Bn are integers. Hence as
claimed .
(Dorin Andrica, Revista Matematica Timi§oara (RMT) , No. 2 ( 1975) , pp. 45, 2239; Gazeta Matematica (GM-B ) , No. 7( 1975) , pp. 268, Problem 1 5212) Problem 34. Subtracting the equalities yields
(x + y - xy) + (v + y - vt) = (x + y - xy)(v + t - vt) , or [(x + y - xy) 1][(v + t - vt) - 1] 1 , so (1) (1 - x)(1 - y)(1 - v)(1 t) 1 . It follows that 1 1 - xl = 1 1 y l = 1 1 v i 1 1 - tl = 1 , ( 1) we obtain and using (x, y, t) (0, 0, 0, 0) , (0, 0, 2, 2) , (0 , 2, 0 , 2) , (0 , 2, 2, 0) , (2, 0 , 0 , 2) , ( 2, 0 , 2, 0) , (2, 2, 0 , 0) and (2, 2, 2, 2) . Turning back to the system we obtain (x, y, v, t) (0 , 0, 0 , 0 , 0) , (0 , 0 , - 4, 2, 2) , (0 , 2, 0 , 0, 2) , (0 , 2, 0 , 2, 0) , (2, 0 , 0 , 0, 2) , (2, 0 , 0 , 2, 0) , (2, 2, - 4, 0, 0) and (2, 2, 24, 2, 2) Ti A d (RMT) , No. 2 ( 1978) , 46, ( tu343n reescu, Revista Matematica Timi§oara 8 5 1 ; Gazeta Matematica (GM-B) , No. ( 19 1 ) , 216, Problem 18740) Problem 35. We start withd ab useful llemma. ively prime positive integers, then there are positive Lemma. I f a an are re at integers u and v such that au - bv 1 . =
-
-
-
z,
z,
-
=
=
=
=
pp .
=
pp.
2.2. SOLUTIONS
Proof. Consider the numbers
71
1 . 2, 2 · a, . . . , (b - 1) . a
(1) b Whenk divided by the remainders of these numbers are distinct. Indeed, otherwise b k f. 2 E {I, 2, . . . , - I} such that we have l kl a = Pl b + r, k2 a = P2 b + r for some integers PI , P2 ' Hence b Ikl - k2 1 0 (mod b) , which is Since a and Ikare relatively prime it follows that 1 l - k2 1 < b. false because b On the otherkhand, none of the numbers listed in (1) is divisible by . Indeed, if so, then there is E {I, 2, . . . - I} such that k . a = P . b for some integer p. k k = kl d, P = PI d, for d Let be the kgreatest common divisor of kand p. Hence b k d( ) 1 . Then l a = PI and since ged(a, b) = 1, we = p , with ge some kintegers l l I' PI b k b have I = , PI = a. This is false, because l < . follows that one of the b so Itthere b numbers from (1) hasb the remainder 1 when divided by is U E {I, 2, . . . , - I} such that au = v + 1 and the lemma is proved. We prove now that the system { axbx -- ytyz+-de == 0O b d d( b) with a, , e, nonnegative integers and ge a, = 1 has at least a solution in nonneg ative integers. d( b) Because ge 1 a, = 1 using the lemma, there are positive integers u and v such b that au - v = . Hence x = eu + dv, y = ad + be, z = v, t = u, is a solution the system. ( Titu Antodreescu, Revista Matematidl. Timi§oara (RMT) , No. 2(1977) , pp. 60, ==
::;
,n
Problem 3029)
36. Consider the determinant
� = 1l 12 Xpl - l p2 - l X
X
X
1
P
p = II ( pp-l
X
X
i , i=l i >i
Xi - Xj )
72
2. NUMBER THEORY
Summing up all columns to the first one and applying the hypothesis yields
�
==
0 p ) ) II (mod p , hence (Xi - Xj ) 0 (mod p . Because p is}a prime number, it follows that) there are distinct positive integers 2 {I k, E( D , i , .A. . ,p such that Xk - Xl 0 (mod p . d i ) or n n r ca ==
i,j=l i>j
==
1
37. Let
a = gcd(al' a2 , . . , an ) and b - gcd ( al +2 a2 ' a2 +2 a3 , . . . , an +2 al ) 12 Then ak = aka, for some integers ak, k = , , . . . , n. It follows that ak +2ak+l = ak +2ak+l a, (1) where an+l = al and an+l a l' Since ak are odd numbers, ak are also odd, so ak +2ak+l are mtegers. . ak +2ak+l for all so a divides b. From relation (1) it follows that a divides ak +2ak+ l = f3k b, for some mtegers . f3k. Then On the other hand, 2b) ak + 0 (mod ak+ l { I 2 } Summing up from k = 1 to k = n yields for all k E , , 2 (al + a2 + . . . + an) 0 (mod 2b) 2) Since al, a2 , . . . , an are all odd al + a2 + . . . + an � 0 (mod , hence (3) al + a2 + . . .2+ an 0 (mod b) . Summing up for k = 1, 3, . . . , n - implies al + a2 + . . . + an- l 0 (mod 2b) and furthermore al + a2 + . . . + an- l 0 b)(mod b) . (4) 2 ) ( Subtracting (4) from (3) implies an 0 (mod , then using relation we obtain ak ( Ti0 (modA db) for all k. Hence bl a and the proof is complete. tu28 1n4)reescu, Revista Matematica Timi§oara (RMT) , No. 1(1978) , pp. 47, Problem ·
=
==
. . .,n .
==
n,
==
==
==
==
==
38. It is known that
cp(kl) = cp(k)cp(l) for any relatively prime positive integers k and
l.
2.2. SOLUTIONS 73 On the other hand, it is easy to see that if P is a prime number, then (pl ) pI _ pl - I
n - 1, n � 1. (n + 1) Then 2 n PI + P2 + . . . + Pn > 2 - n n2 . The inequality al" + a;" : . . . + a;:' ;:: ( a1 + "" : . . . + an ) m holds for any positive real numbers aI, a2 , . . . , an . Hence m m+ 1 : l" ) . . + Pn r p . + ( : J n , >n P + pr + . . . + P::' ;:: n ( as desired. (Dorin Andrica, Revista Matematica Timi§oara (RMT) , No. 2(1978) , pp. 45, 3483) Pro blem S L d and let
=
=
=
=
=
=
=
=
1'2
=
d <2n gcd(d, n ) = l
=
2. NUMBER THEORY
74
Note that
d(n, d) = 1 gcd(n, n - d) = 1 gcd(n, n + d) = 1 (1) gc d1, d2 , , dcp(n) be the numbers less than n and relatively prime with n. From ( 1 ) weLetdeduce that d1 + dcp(n) = n d2 + dcp(n) - l = n {:}
{:}
• • •
hence d
d = ncp2(n) .
On the other hand,
L d = L (n + d) = ncp(n) + L d = (n) 3ncp(n) ( ) ncp 2 - 2 . - ncp n + -Therefore (n) 3ncp(n) 2 ( ) ncp s = 2 + 2 = ncp n . 2 ( 1981 ) , pp. 61 , (Dorin Andrica, Revista Matematica Timi§oara (RMT) , No. 4574) Problem 3 42 . We proceed by induction. For n = ! the claim is true. Assume that the 1 1 hypothesis holds for n - . Let < k < n and let kl , q be the quotient and the 0 � q < n and 0 � kl < � < = remainder when k is divided by n. Hence k k i n + q, n n-! = (n - I)! . n From the inductive hypothesis, there are integers di < d� < . . . < d� , s � n - 1di, suchd�that dil(n d�- I) ! , i = 1 ,02, . . . , s andd kl =d di + d� d+ . . . + d�d. Hencedi + . . . + n + q. If q = ,! then k = 1 + 2 + . . . + s , where i = n , ik== 1n, 2, .+. .n, s, are distinct divisors of dn . 0 d If q f. , thend kl = 1 + 1 2+ . . . + s+1 ,dwhereI ! di = ndL iI , 2, . . . , s, and ds+1 = q. ! i It dS+1is clear d thatd i n , =d , , . . . , s and d s+1 n , since diq < dn. On the other hand, < 1 < 2 < . . . < s , because S+1 = q < n ! � n = 1 . Therefore k can be written of at most n distinct divisors of n , claimed. 2 ( 9 83 ) dreescu, ( Titu Aansum 1 , pp. 88, Revista Matematica Timi§oara (RMT) , No. C4 1 0) Problem : d
n < d <2n gcd( d , n ) = l
_
d
_
rh
as
as
75
2.2. SOLUTIONS 43. If
n � 992, take the set with all 992 odd numbers from {I, 2, . . . , 1984}.
Its complementary set has only even numbers, any two of them not being relatively prime. Hence n � 991. Let c be the complementary set of a subset with 991 elements of the set {I, 2, . . . , 1984}. Define D = {c + 11 c E C } If C n D = 0, then C U D has 2 · 993 = 1986 elements but C u D C {I, 2, . . . , 1985}, which is false. Hence C n D f. 0, so there is an element a E C n D. It follows that a E C and a + 1 E C and since a and a + 1 are relatively prime we are done. Revista Matematica Timi§oara ( RMT) , No. 1(1984) , pp. 102, Problem C4:7) .
( Titu Andreescu,
n, hence n < (a + 2) 2 . If a � 7, then a - 4, a - 2, a are odd integers which divide n. Note that any two of these numbers are relatively prime, so (a - 4) (a - 2)a divides n. It follows that (a - 4) (a - 2)a < (a + 2) 2 Oso a3 - 6a2 + 8a < a2 + 4a + 4. Then a3 - 7a2 + 4a - 4 � 0 or a2 (a - 7) + 4(a - 1) � . This is false, because a � 7, hence a = 1, 3 or 5. If a 1, then 12 � n < 32 , so n E {I, 2, . . . , 8} . If a = 3, then 32 � n < 52 and 1 . 31n, so n E {9, 12, 15, 18, 21, 24}. If a 5, then 52 � n < 72 and 1 . 3 . 51n so n E {30, 45}. Therefore n E {2, 3,( D4, 5,i 6, A 7, 8, 9, 12, 15, 18, 21, 24, 30, 35}. or n ndrica and Adrian Ghioca, Romanian Winter Camp 1984; Revista 44. Let a be the greatest odd integer such that a2 <
=
=
P.
Matematica Timi§oara (RMT) , No. 1(1985) , pp. 78, Problem T.21) 45 . We prove by induction that
u� - 2v� = 1, For have
n = 1 the claim is true. Assuming that the equality is true for some n, we
U�+ l - 2V�+1 hence (1) is true for all We prove now that
=
n�
(3u + 4Vn ) 2 - 2(2un + 3V ) 2 = u� - 2v� = 1 1.
n
n
Y
2x� - � = 1,
Indeed,
n n
n
(2)
n�1
2x� - Y� = 2(u + V ) 2 - (u + 2v ) 2 as
(1)
n � 1.
n
=
u� - 2v� = 1,
claimed. It follows that
(Xn V2 - Yn) (Xn V2 + Yn) 1, n � 1. Notice that xn ..,fi + Yn > 1 so 0 < xn V2 - Yn < 1, n � 1. Hence Yn [Xn V2] , as claimed. =
=
2. NUMBER THEORY (Dorin Andrica, Gazeta Matematica (GM-B) , No. 11 (1979) , pp. 430, Problem 0 :97)
76
46. ( i) We have
hence
Xk = 22k -1 + 1 = 22k-2 .2 + 1 = (Xk-l - 1 ) 2 + 1 = X2k-l - 2Xk- 1 + 2, Xk(1 )- 2 = kXk- l2(Xk-kl - 2) .
Multiplying the relations
from = to = n yields
(1)
X l = 3, we obtain 2 (2) = X l X 2 . . . Xn - l + X n 2. for all n For a different proof use the identity x2n - 1 -1 I = nII-l (X2k- 1 + 1) . X - k=l d( d d(xn' X2 ) = . . . = ( ii ) From relation (2) we obtain gc x x gc n' gck d(xnl, Xn - l ) = 1 for all n, hence gcd(Xk ' xz ) = 1 for all distinct positive integers and . (2) it follows that Xn (iii ) Since X2 = 5 and X I X2 . . . X n - l is odd, using relation end with 7 for all integers n 2:: 3. (Doridigit n Andrica) and since
2::
47. We have
Then and
a�+2 + a�+l - 4an+2 an+1 = - 2, Subtracting these relations yields a�+2 - a� - 4an+l (an+2 - an ) = and (an+2 - an)(an+2 + an - 4an+d = 0, n 2:: 1 . ( ) Since the sequence an n 2:1 is increasing, it follows that °
77 2.2. SOLUTIONS Taking into account that al = 1, a2 = 3 and inducting on n we reach the conclu sion.( Ti A d tu n reescu, Gazeta Matematica (GM-B) , No. 11(1977), pp. 453, Problem 2 16947; Revista Matematica Timi§oara (RMT ) , No. 1(1978) , pp. 51, Problem 840) 2 2 2 2= 48. For n = we have a2 = 1+ 3" ' d( ) Suppose that an = Pn , where Pn , qn are integers and gc Pn' qn = 1. qn Then 2pnqn an+ 1 = qn2 + 2P2n ' d(2 � 2 �) and it suffices to prove that gc Pnqn' q + p = 1. d kk Assume by way of contradiction that there are integers and l' 2 such that 2Pnqn = kI d and q� + 2p� = k d. 2 2 2 2k � � d If = , then q = 2 - p , so qn is even. This is a contradiction since ql , q2 , ·d. . ,q2n are all� odd.(k k )d dl If > � then 2 �q =k d2 qn - lPn sodl qn' d( ) Since q + p = 2 it follows that Pn , which is false, because gc Pn' qn = 1. This proves our claim. We prove that � - 2p� = 1, n 2:: 2. (1) q 2 2 2 2 � � For n = we have q �- p = 32 - . 2 = 1. 2 � Suppose that q - p = 1. Because
is irreducible, we obtain Hence
qn2 + 1 - 2Pn2 +1 = (qn2 + 2P2n)2 - 8p2nqn2 = (qn2 - 2P2n) 2 = 1 , claimed. Hence �I bn = 21 1 q -2P2 - qn2 - I = 2 2 2 1 - an2 - I q P I 1 n n - - n- I 1 2 q b 2'rl- I and (soTi n isA adperfect square for all n 2:: . tu n reescu, Revista Matematica Timi§oara (RMT) , No. 2(1977), pp. 63, as
_
--::-_.:.:-=-=_
_
Problem 3083; Gazeta Matematica ( GM-B ) , No. 1(1981), pp. 44, Problem C :88)
2. NUMBER THEORY
78
49. Inducting on n we obtain
xn2 - 2Yn2 - 1 , Hence Zn = 1 + 4X2n Yn2 = (X2n
_
2yn2 ) 2 + 4X2n yn2 = ( 2n + 2yn2 ) 2
= (x; + 2y� + 2xnYn ) (x; + 2y�
X
_
4X2n yn2 =
- 2xnYn) , for all n � 1.
Since both factors are greater than 1, it follows that all numbers Zn are not prime. Revista Matematica Timi§oara (RMT) , No. 2(1976), pp. 54, Problem 2571)
( Titu Andreescu,
50. We have
or Hence and
Xn Jp2 + 1Xn+ 1 ± PJX;+1 + 1. From hypothesis we have X n+l > Xn , hence Jp2 + 1Xn+ 1 -pJX�+ + 1. Xn = 1 =
(1)
But
(2) and so (3) by summing up the relations (1) and (2) . 2 + 1 is irrational. From (3) Because is a positive integer, it follows that we deduce that among any three consecutive terms of the sequence there is at least an irrational term. Hence there are at least irrational terms among the first m terms of the sequence. Revista Matematica Timi§oara (RMT) , No. 1-2(1980), pp. 68, Problem 4139; Gazeta Matematica ( GM-B ) , No. 6(1980), pp . 281, Problem C:48)
p
[;]
( Titu A ndreescu,
5 1 . Setting n = 0 and n = 1 yields From the given condition we obtain
Jp
Xl = x� and X2 = x� , hence Xl X2 X2n+ 1 = 2n+l + Xn2 and X2n Xn2 +l - X2n-l . X
=
=
=
1.
2.2. SOLUTIONS
79
Subtracting these relations implies hence We induct on
(1) (2)
X2n+1 = X2n + X2n- l , n � 1 .
n to prove that � 1. n , X2n = X2n-1 + X2n-2 (2)
X2 = Xl + Xo and assume that is true up to n . Then X2n+2 - X2n = Xn2 +2 - Xn2 - Xn2 + l + X�n2 - l (*)= (Xn+ l + Xn ) 2 - Xn2 - Xn2 +l + + (Xn+l - Xn ) 2 = X + l( 1+) X� = X2n+ l , and the induction hypothesis) . as claimed (the equality because of O. Because ( 1 ) and( )(2holds ) it follows � From relations that = for all n x X X + +2 + n n n l 0 Xo = and Xl = 1 the sequence (xn ) n 2:0 is the Fibonacci's sequence, hence Indeed,
*
( Titu Andreescu, Romanian Winter Camp 1984; Revista Matematidi Timi§oara 1 ( 1985) , pp. 73, Problem T.3) (RMT), No. 1 2 25 48 52. From the hypothesis it follows that a4 = , a5 = , a6 = . We have �l , a22 = 1 , a33 = 2 , a44 = 3 , a55 = 5 , a66 = 8 so an = Fn for all n = 1 , 2 , 3 , 4, 5 , 6 , where (Fn ) n 2: 1 is the Fibonacci's sequence. F n kFk We prove by induction that a for all n. Indeed assuming that a = = n k n n k 3 for � n + , we have 2 (n + l) Fn+1 - nFn = a2 n+4 = F2 (n + 3) Fn+32 +F(n + 2)F +2 n = (n + 3) n+32+( (n +3)F) n+2 -2F2 (n + l()Fn+1 -Fn(Fn+2 - Fn+l ) = = 3nF+ n+2F3 + n+( 2 - 2n +F 2) n+1 = 2 ( = n + ( ) n+4)3 +F n+2F- n + () ( n+4 3F- Fn+2 ) = = n + ( n+3 + n+2 ) = n + ) n+4 as desired. (Dorin Andrica, Revista Matematidl. Timi§oara (RMT) , No. 1 ( 1986) , pp. 106, C8 2) Problem : O 53. Observe that setting Xo = 0 the condition is safisfied for n = . k that Xk divides m. Let rt be the We prove that there is integer � m3 such 2 0 1 3 remainder of Xt when divided by m for t = , , . . . , m + . Consider the triples (ro , rl , r2 ) , (rl ' r2 , r3), ' . . , (rm3, rm3+ 1 , rm3 +2 ) ' Since rt can take m values, it follows -
-
-
-
-
-
'
2. NUMBER THEORY by the Pigeonhole Principle ( that at least) two triples are equal. Let p (be the smallest) number such that triple rp , rpO+l, rp+ 2 is equal to another triple rq, rq+l, rq+ 2 , p < q :::; m3 . We claim that p = . Assume by way of contradiction that p 2:: 1. Using the hypothesis we have rp+2 Tp- l + rprp+l (mod m) and rq+2 rq -l + rqrq+l (mod m) . rp d= rq,( rp+l = rq+l) and rp+2 = rq+2 , it follows that rp-l = rq - l so (rp- Since with the minimality of p. , which is a contradiction l ,rp ,rp+0 rq- l ,rq,rq+l 0 0 Hence = = ) m (mod Xq therefore and , r = rq so , ( Tiptu Andreescu ando Dorel Mihet, Revista Matematica. Timi§oara (RMT) , No. 106 , Problem C8: 1) 1(1986), pp. F6 , al = F12 , a2 = F? , 4 5 = so 25, = a 9, = 54. a 4, = a3 1, = a that Note ao 2 F F F F sequence. a3 = i , a4 = ; , a5 = i , where ( n )Fn2:o� is the Fibonacci Ff O We induct on n to prove that an = for all n 2: . Assume that ak = for all k :::; n . Hence (1) an - Fn2 ' an -l - Fn2- l ' an-2 = F�_2 ' 80
==
==
=
==
From the given relation we obtain
and
an - 3an- l + an-2 = 2( - l )n- r ,
n 2: 2.
Summing up these equalities yields
(2) Using the relations (1) and (2) we obtain
an+l = 2F� + 2F�_1 - F�_2 (Fn + Fn_ 1 ) 2 + (Fn - Fn_ 1 ) 2 - F�_2 = =
as desired.
( Titu Andreescu, Revista Matematica Timi§oara (RMT) , No. 2(1986), pp. 108,
Problem C8:8)
a2 - 1 and 2 + 2a ask for the substitution a � (b2 + b� ) . 2 The equality � ( 2 + � ) = 97 implies ( + �) = 196, hence + � = 14. Setting 55. The expressions b
b
=
b
b
81 2.2. SOLUTIONS 2 2 = c yields (c + �) = 16, thus c + � = 4. Let c = 2 + va. We will prove by induction that F a = � ( C4 . + 4�. ) , n 2: 1, n C F where n is the nth Fibonacci number. Indeed, this is true for n = 1, n = 2 and, assuming that 4 41_ ) ' k <_ n a k = �2 (e Fk + _ e Fk b
implies
and
2 + J2 + 2an = 2 + e2Fn + 21 = (eFn + -1 ) 2 e Fn eFn Let for all positive integers m. +� dn reescu,thatUSAemMathematical ( TituusAmention em is an integer Olympiad Shortlist, 1997) 56. We induct on m to prove that km n < km+1 , then an = 1 + m. (1) if k For m = 0 the claim is true, since if 1 n < , then [ �] = 0 hence an = l + ao = 1 + 0. k k k k Assume that (1) holds for m. Then if m+1 n < m+2 , we have m � < m+ 1 , so km [�] < km+1 . --
�
�
�
�
�
Using the inductive hypothesis, we deduce that a [�] = 1 + m so
an = 1 + a [ �] = 1 + (m + 1)
as claimed. Therefore for any positive integer n with m � logk n < m + 1, we have = 1 + m thus 1 + [log kn]. Revista Matematica Timi§oara (RMT) , No. 2(1982) , pp. Problem 4997)
an
(aTintu= Andreescu,
an - 2 = b � ,
57. We prove that where bo all n 2:: 1. This is clear for n = 0 and n = 1.
66,
= -1, b1 = 1 and bn+1 = 3bn - bn- 1 for
2. NUMBER THEORY 82 b� . Subtracting the relation an = k Assume that for any = � n we have ak -2 7an- l - an-2 from an+l = tan - an l yields an+l = 8an - 8an l + an 2 or - 8( - 8 ( ( an+l -2 = an -2) - an-l - 2) + an-2 -2) Hence an+l -2 = 8b� - 8b�b_ 1 + b�b�_2 = 8b�3b- 8b�b _1 + (3bn-b� 1 - bn )2 = 9b� - 6bn n-1 + l = ( n - n_ 1 ) 2 = +l = Alternative solution. The general term- of the sequence is given by ( v'5 ) 4n - 2 ( 1 v'5 ) 4n-2 1 _ + an - 2 + 2 ' n 2:: 0 _
Hence
( 1 + V5) 2n-l + ( 1 - V5) 2n-l 2 2
and an inductive argument proves that
is an integer for all n.
( Titu Andreescu)
58. Note that all terms of the sequence are positive integers. Assume by way of contradiction that there is a positive integer no such that
X�o = Xno - lXno +l ' Then Xno +l _- xno - t xno Xno - l where t is rational. (3 Because Xno +l o:xno + Xno- l, it follows that Xno+l 0: + (3 � xno Xno - l or t2 - o:t - (3 = O. --
--
,
=
=
� 0:2 4(3 is not
The last equation has no rational roots since the discriminant = + a perfect square. This is a contradiction and hence the problem is solved. From the condition of the problem we obtain
Alternative solution.
(1)
83
2.2. SOLUTIONS
Assume by way of contradiction that there is a positive integer no such that 2 _ X l X no+l Xno - no '
() (a2 + 4,8)x�o = (Xno+l + ,8xno - d2 4,8 is not a square. This completes the proof. 2+ is false, since a which A (Dorin ndrica) From the relations 1 and (2) we deduce that
(2 )
59. Assume by way of contradiction that there is an irrational number a such
that is rational. Define a = \ja + a2 - 1 and observe that \ja - a2 - 1 = � .
J
J
Hence A = a + � is rational.
a
a
We prove that an + �n is rational. a Indeed, 1 2
(
is rational and a3 +
U sing the identity
:3 = (a + � r
(
1
= ak - 1 + _ ak + � ak a k -1
)
1 2
a + _2 = a + a a
-
-2
3 (a + �)
is rational.
a+
ak - 2 + _ ak- 2
) ( �) ( _
a
1
)
k,
it follows by induction that ak + -\- is rational for all positive integers hence a 1 . . 1 n l a +IS rat Ona . an Thus a + a2 - 1 + a - a2 - 1 = 2a is rational, which is false. The solution is complete. Romanian IMO Selection Test 1977; Revista Matematidi Timi§oara RMT , No. 1 1978 , pp. 78, Problem
J J ( Titu Andreescu, ( ) ( )
3344) 3 4 60. We induct on n. For n = 1, from ±1 ± 2 ± ± ± 5 we obtain all odd positive )( integers less than or equal to (2 + 1 4 + 1) = 15: + 1 22 + 33 + 44 55 31 1+ + + 1+2+3 4+5 5 -
-
-
-
-
=
-
=
=
84
2. NUMBER THEORY
- 11 + 22 - 33 + 44 + 55 = 97 -1 - 2 +3 +4 +5 = 11 +1 - 2 + 3 + 4 + 5 = 13 - 1 + 2 + 3 + 4 + 5 = 15 + + ± +(4 + 1 )+ = ± ± 2 ± 1 . . . n + with suitable Assume that from ) . + and (2n +choices 1)(4n of+ 1signs we obtain all odd positive integers less than or equal to (4n+ 2) + (4n+ 3) + (4n+ 4) - (4n+ 5) = O . Hence from ± 1 ±2± Observe that . . . ± (4n + 5) for suitable + and - we obtain all odd positive numbers choices of signs 2 )(4 ( ) 1 1 less than or equal to n + n+ . It suffices to obtain all odd integers m such that (1) (2n + 1 )(4n + 1 ) < m :s; (2n + 3)(4n + 5) (2n + 3)(4n + 5) - (2n + 1 )(4n + 1) 8 7 There are 2 = n+
such odd integers m. We have
(2n + 3)(4n + 5) = + 1 + 2 + . . . + (4n + 5) (2n + 3)(4n + 5) - 2k = + 1 + 2 + . . . + (k - 1 ) - k+ + (k + 1 ) + . . . + (4n + 4) + (4n + 5) , k = 1 , 2, . . . , 4n + 5 and (2n + 1 )(4n + 5) - 2l = + 1 + 2 + . . . + (l - 1) - l + + (l + 1) + . . . +( 1 )(4n + 4) + (4n + 5) , l = 1 , 2, . . . , 4n + l. Hence from are obtained, desired. Andrica,mGazeta 2 ( 1986) , pp. 63, Problem C570) ( Dallorinnumbers Matematica ( GM-B ) , No. as
Chapter 3 GE OMETRY
PROBLEMS 1. Triangle
ABC has side lengths equal to a, b, c. Find a necessary and sufficient
condition for the angles of the triangle such that a2 , b2 , e can be the side lengths of a triangle. 2. Prove that the triangle whose side lengths are equal to the length of the medians
ABC has area equal to 3/4 of the area of triangle [ABC] . ABC the median AM meets the internal bisector BN at P. Let Q 3. In triangle C AB . Prove that triangle BNQ is isosceles. be the point of intersection of lines P and ABC the midline parallel to AB meets the altitudes from A and B 4. In triangle D E AC intersects the altitudes from A and C at points F and G . The midline parallel to at points andDC. II BF I GE Prove that . ABC. Lines AM, BM, CM intersect LetCA M be a point in the interior of triangle 5.BC ' ' ' A sides , , AB at points 'B ,AB' C, C , respectively. ' A Denote 'A by 81 , C82'B, 83 , 84, 85, 86 ' A B C B C the areas of triangles M , M , M , M , M and M , respectively. of a triangle
Prove that if
81 82
+ ABC+ 83 84
85 - 3, 86 -
then M is the centroid of the triangle
. ABC and let M, P, Q be three 6. Let M be a point in the interior of triangle AB, BC and line CA. colinear points on sides Prove that if
AC+
SMAN 8MBN
then MP is skew-parallel with
5MBP = 8M C P
.
2J
SMAQ , 8McQ
ABC. Prove c and 8 be the sides lengths and the area of a triangle P ABC such that that if is a point interior to triangle aPA + bPB + cPC = 48 7. Let a, b ,
87
3. GEOMETRY
88
then P is the orthocenter of the triangle. 8. Let 11 , 12 be the incenters of triangles A l Bl Cl and A2 B2 C2 . Prove that if II and 12 divide the internal bisectors Al A� , A2 A� in the same ratios they divide the internal bisectors Bl BL B2 B� , then triangles A l Bl Cl and A2 B2 C2 are similar. 9. On side BC of a triangle ABC consider points M and N such that ifAjJ
CAN.
Prove that
MB MC
==
NB 2: 2 AB + NC AC
1 0 . Let M be a point on the hypotenuse BC of a right triangle ABC and let points N, P be the feet of the perpendiculars from M to AB and AC. Find the position of M such that the length NP is minimal. 1 1 . Let ABC be an equilateral triangle and let P be a point in its interior. Let the
lines AP, BP, CP meet the sides BC, CA, AB at the points AI , Bl , Cl respectively. Prove that
1 2 . Let S be the set of all triangles ABC for which
5( 1 + 1 + 1 )3 6 min{P, BQ, CR} = r ' AP BQ CR
where r i s the imradius and P, Q, R are the points of tangency of the incircle with sides AB, BC, C A, respectively. Prove that all triangles in S are isosceles and similar to one another . 1 3. Let ABC be a triangle inscribed in a circle of radius R, and let P be a point in the interior of ABC. Prove that PB PA PC BC2 CA2 AB2 R '
+
+
2: 1
1 4 . Let I b e the incenter of an acute triangle ABC. Prove that
AI · BC
+ BI · CA + CI · AB = SABe
if and only if triangle ABC is equilateral. 1 5 . Let ABC be a triangle such that
(8AB - 7BC - 3CA) 2 = 6(AB2 - BC2 - CA2 ).
Prove that A = 60°.
3 . 1. PROBLEMS
89
P be a point in the plane of triangle ABC such that the segments PA, 16. Let PB and PC are the sides of an obtuse triangle. Assume that in this triangle the PA. Prove that BAlJ is acute. obtuse angle opposes the side congruent to ABC has the following property: there is an interior point P such 1 7. Triangle ° 0 20° , PcA = 30° , and PAC = 40° . Prove that the triangle 1 that PAJj = , PEA = ABC is isosceles. ABC be a triangle such that max{A, B } = C + 30° . Prove that ABC is 18. Let right angled if and only if !!. = J3 + 1 . r ABC there is a point P such that 1 9 . Prove that in the interior of any triangle PAB, PBC, PCA are equal. the circum radii of triangle 20.CThe ' of the triangle ABC touches the sides AB , BC, CA at the ' A' incircle B Prove that the perpendiculars from the midpoints of 'B' B' C, ' C, 'A,' respectively. Apoints , , to AB , BC , CA, respectively, are concurrent. I. Let Ci , i = 1 , 2 , 3 , AAA 2 1 . Let I 2 3 be a non-isosceles triangle with the incentre Ai Ai+1 and Ai Ai+2 (the addition of indices be the smaller circle through I tangent to CHI and Ci+2 . 3) B i 1 2 3 being mod . Let i , = , , , be the secondApoint of intersection of BI ABI ABI Prove that the circumcenters of the triangles I I , 2 2 , 3 3 are collinear. AB and AC of a triangle ABC consider points B' and C' such that 22. On sides AB' ' AC' ' k B B + C C = is constant. BC' and CB' . Find the locus of the intersection point of lines ABC be an equilateral triangle of side length 1 . Find the locus of points 23. Let P such that PA · PB · PC PA PC PB { } 2 max , , = PA . PB + PB . PC + PC · PA - 1 24. Prove that in any acute triangle
Ja2 b2 - 482 + Ja2 c2 - 482 = a2 .
25. Prove that a triangle in which r,;.-
y. Ta
is equilateral.
r,;::
�
+ y rb + y rc = Jrarbrc -r .
.
-'--
3. GEOMETRY
90
26. Prove that a triangle is equilateral if and only if
+ + --+-+-
1 1 1 1 1 1
-
m2a
-
m2b
-
rb2
m2c - ra2
. rc2
1 a,bb, c has circum radius equal to . Prove that a + + c ;::: abc.
27. A triangle with side lengths
28. Prove that in any triangle
a)(2 ) � 6R b � (p - p - c -;:-. 29. Prove that in any triangle 1 1 1 1
+ + h2b
h2a
30. Prove that in any triangle
Jr arb 31 . Prove that in any triangle
+
Jrb rc
h2c
+
;:::
3r2
Jrc ra ;::: gr.
32. Prove that
for any triangle .
� ib if and only if (p - b)(p - c) � 4e '
33. Let ABC be a triangle. Prove that A
34. Three equal circles of radii r are given such that each one passes through the
centers of the other two. Find the area of the common region. 35. Let ABCD be a nonisosceles trapezoid with bases AB and CD. Prove that
+
AC 2 - BD 2 AB CD AD 2 _ BC2 - AB - CD ' 36. Let ABC D be a trapezoid with bases AB and CD. Prove that if
(AB
+
CD) 2 = BD 2 + AC2
91
3.1. PROBLEMS
then the diagonals of the trapezoid are perpendicular. 37. Prove that if in a trapezoid with perpendicular diagonals the altitude is equal to the midline, then the trapezoid is isosceles. 38. Let ABCD be a trapezoid with bases AB and CD. Prove that
AB 2 - BC2 + AC2 CD 2 - AD 2 + AC2
AB AB2 - AD2 + BD2 = CD 2 _ BC 2 + BD2 = CD '
39. Prove that a trapezoid whose difference of the diagonal lengths is equal to the difference of nonparallel side lengths is isosceles. 40. Let ABCD be a cyclic quadrilateral. Prove that
lAB - CD I + l AD - BC I 2: 21AC - BD l · 41 . Prove that a right trapezoid whose altitude length is equal to the geometrical
mean of the lengths of its bases has perpendicular diagonals.
E and F be the projections of the vertices A and B of a trapezoid ABCD E and F onto BD and AC, respectively of on line CD . Let M and N be the projections E and let P and Q be projections of and F onto BC and AD, respectively. 42. Let
Prove that the quadrilateral MN PQ is cyclic.
43. Let ABC D be a convex quadrilateral that is not a parallelogram and let M
and N be the midpoints of the diagonals AC and BD . Prove that numbers AB + CD,
BC + AD,
AC + BD,
2MN
can be the side lengths of a cyclic quadrilateral.
E. E Point F is the reflection of C across . Prove that lines AF and B D are perpendicular 44. Let ABCD be a cyclic quadrilateral and let lines AB and CD meet at point
if and only if lines AB and CD are perpendicular.
45. Find all cyclic quadrilaterals having the side lengths odd integers and the
area a prime number. 46. A cyclic quadrilateral has the side lengths a,
and the semiperimeter p . Prove that
b, c, d, the diagonal lengths
e,
f
92
3. GEOMETRY 47. A cyclic quadrilateral has the side lengths a,
I e and .
Ib
{
:::; 1 , then II :::; V2. Ie -
Prove that if max l a - c l , - d l }
b, c, d and the diagonal lengths
48. A cyclic quadrilateral has area S and semiperimeter p . Prove that if S =
then the quadrilateral is a square.
(�) 2 ,
49. Let ABCD be a convex quadrilateral and let P be the intersection point of its diagonals. Prove that SPAB + SPCD = SPBC + SPDA if and only if P is the midpoint of AC or BD. 50. Let M be a point on the circumcircle of the cyclic quadrilateral ABCD and
let points A', B', C', D' be the projections of M onto AB, BC, CD, DA, respectively. Prove that (i) lines A' B' , C'D' and AC are concurrent (ii) lines B'C' , D' A' and BD are concurrent. 5 1 . A convex quadrilateral ABCD with area 2002 contains a point
P in its interior
such that PA = 24, PB = 32, PC = 28, and PD = 45. Find the perimeter of ABCD . 52. Find the locus of points P in the plane of a square ABC such that
max(PA, PC) =
1
V2
(PB + PD).
. LetA P be the set of all quadrialterals with the same diagonal lengths and let (P)5 3and 2 (P) be the lengths of the segments determined by the midpoints of two opposite sides of a quadrilateral p E P.A�(P) A� (P) Prove that for all p E P the sum + is constant and find the value of >'1
this constant .
54. Let ABCDEFGHIJKL be a regular dodecagon and let
dius. Prove that
4
R be the circumra
AB AF = and AB2 + AC 2 + AD 2 + AE2 + AF2 = 0 2 . + AF AB
1R
93
3 .1. PROBLEMS
55. Prove that if inside a convex poligon there is a point such that the sum of the
squares of its distances to the vertices of the poligon is twice the area of the poligon, then the poligon is a square. 56. Let AI A2 . . . An be a cyclic polygon and let P be a point on its circumcircle.
Let PI , P2 , . . . , Pn be the proj ections of P onto the sides of the polygon. Prove that the product n
II PA��
i= l P��
is constant.
57. Let AI A2 . . . A2n be a cyclic polygon and let M be a point on circumcircle. Points KI , K2 ' . . . , K2 n are the projections of M onto sides its AI A2 , A2 A3 , . . . , A2 n AI and points HI , H2 , . . . , Hn are the projections of M onto diagonals AI An+ I , A2 An+2 , . . . , An A2n · Prove that
MKI · MK3 . . . MK2 n-l = MK2 · MK4 . · . MK2 n = MHI · MH2 . . . MHn · 58. Find the circumradius of a cyclic polygon with 2n sides if n sides have the length a and n sides have the length
b.
59. Let P be a point in the interior of a tetrahedron ABCD such that its projec
tions AI , BI , CI , DI onto the planes (BCD) , (CDA) , (DAB) , (ABC), respectively, are all situated in the interior of the faces. If S is the total area and r the inradius of the tetrahedron, prove that SBeD PA l
+
Se DA PBI
When does the equality hold?
+
SDAB PCI
+
SABe PDI
> -
� r
60. Let AIA2 A3 A4 be a tetrahedron, G its centroid, and A� , A� , A�, A� the points where the circumsphere of AI A2 A3A4 intersects GAl , GA2 , GA3 , GA4 respectively. Prove that
1+1+1+1 1+1+1+1 GA l · GA2 · GA3 · GA4 � GA� . GA� · GA� . GA�
and
GA�
GA�
GA�
GA4
�
GAl
GA2
GA3
GA4 ·
SOLUTIONS 1 . The positive real numbers
only if Because
a2 , b2 , c2 can be the side lengths of a triangle if and (1)
2 + b2b 2 + a2 - b2 c a cos B = ' cos C = 2a ' 2 ca O 0 0 relation (1) is equivalent to cos A > , cos B > , cos C > . Hence the necessary and sufficient is that triangle AB C is acute. ( Doricondition n 07Andrica, Revista Matematidl Timi§oara (RMT) , No. 2 ( 1978) , pp. 48, - c2
Problem 35 )
A', B', C' be the midpoints of sides B C, CA, AB respectively. Construct point M such that BC M C' is a parallelogram . Note that AC' C M and B B' M A' are also parallelograms hence AM = CC' and A'M = BB'. Hence the triangle determined by the medians is AA' M. M A 2 . Let
B
A'
C
Let N be the intersection point of lines B' C' and AA'. Because AC' A' B' is a parallelogram, we have
C'N = B'N = !B'M 95
2 '
3. GEOMETRY
96 SO
BI is the centroid of triangle AAI M . Hence
Alternative solution: Consider the vectors = BC, b = AC , c = AB , ma = AAI , mb = BBI and observe that a
We obtain
1-
(-b - 2c) = 43 (c -b),
ma mb = 4 (b + c) 3 I l henceTima A m b = 4) 1c bl and the conclusion follows. ( tu ndreescu AC at P, then by Ceva's theorem we obtain B 3. If P meets MB NC 1 QBA MC Q ' ' NA = . MB = MC, we have Because QBA NAC Q N' QN I BC and then NBC = CiiiB. Therefore BN angle --xiiC, it follows that NBC = QiiN. On the other hand, because Bbisects NQ is isosceles, desired. Hence CiiiB = QiiN and triangle A d Ti ( tu n reescu, Revista Matematica Timi§oara (RMT) , No. 1 ( 1978) , pp. 66, 3286) Problem ABC and let AI be the midpoint of side H be the 4. Let orthocenter of triangle I I BC . Because FA I AC, it follows that FA 1- BH. Moreover BAI 1- HF, so AI is the BHF. Hence orthocenter of triangle HAI 1- BF. ( 1) I D I AB AI D HC HD AI D A D On the other hand, , so 11it follows that . Since l HA C thus is the orthocenter of triangle DC 1- HAI . ( 2) x
x
x
x
x
as
3.2. SOLUTIONS
97
A
B �------�-*�--� F
Note that GH 1- AB and EH 1- AG . Since ABI I A'E and AG I IGA', we have GH 1- A' E and EH 1- GA' , so H is the orthocenter of triangle A' EG. Therefore
(3)
A'H 1- GE . 2 ( ) and (3) we obtain From relations ( 1 ) , DG II BFI IGE,
as desired.
( Titu Andreescu)
5 . Using Ceva's theorem, we obtain
G'A A'B B'G - · _ · - = 1' G' B A' G B ' A
hence
S1 S3 S5 . . = S2 S4 S6
l.
The given condition now reads S1 S2
so This implies
+ + S3 S4
.3 / S1 . S3 . S5 S5 = 3V S2 S4 S6 S6
G'A A'B B'G = l. = = G' B A' G B ' A
It follows that A' , B' , G' are the midpoints of the triangle's sides and M is the centroid, as claimed. Revista Matematica Timi§oara (RMT) , No. 1 ( 1974) , pp . Problem 1904; Gazeta Matematica ( GM-B ) , No. 2 ( 1979 ) , pp. 63, Problem 0:16)
( Titu Andreescu,
23,
98
3. GEOMETRY 6. Using Menelaus' theorem yields
NA AB . QC . NB BC QA
Hence
= 1.
SMAN . 5MBA . SMCQ 5MBN SMC A SMAQ -
or
SMAN 5MBP . 5MBN SMCP A
1,
= SMQA ' SMCQ
B
From the condition in the hypothesis it follows that 5MBP + 5MBP = 2 V. j SMAN , . 5MBN SM CP SMCP
SMAN SBMN
and so
SMAN 5MBP ' 5MBN SMCP
Thus
NA NB
PB PC
hence N P and AC are skew-parallel . Revista Matematica Timi§oara (RMT) , No. Problem Gazeta Matematica (GM-B), No. Problem pp.
( Titu Andreescu, 3287;
1 ( 1978) , pp. 66, 17607) 2 ( 1979) , 56,
7. Let AI , Bl , Cl be the feet of triangle ' s altitudes and let A', B', C' be the pro
jections of P onto its sides. If P is the orthocenter of the triangle, then the equality holds. Assume by way of contradiction that P is not the orthocenter of the triangle and aPA
+ bPN + cPC = 4S.
Then at least two of the inequalities PA + PA � AAl ,
PB
+ PB' � BBl ,
PC + PC' � CCI
are strict.
A
3.2. SOLUTIONS
It follows that
99
l ) b (BB PBI) (CC PC/ ) AA PA PA PC bPB ( a + +c > a l - + l - +c l or AA bBBl + cCCl - (aPAI + bPBI + cPC/ ) . 4S > a l + 6S - 2S = 4S , which is false. Hence ( 1978) , pp. 74, ( Titu A4Snd>reescu, Revista Matematica Timi§oara (RMT ) , No. 2 3 89 980 6 ) ( Problem ; Gazeta Matematica (GM-B) , No. 2 1 , pp. 64, Problem 18 122) 8. Using the angle bisector theorem and Van Aubel's theorem, it follows that IA BIA CIA b + c k IAI = BI C + CIB = -a- = l IB AlB CI B c + a k and IBI = AI C + CI A = -b- = 2 IC _ a + b k Let ICI - C - 3 and note that kl + 1 = -;;:2p k2 + 1 = b' 2p k3 + 1 = -2p , ' c where p is the semiperimeter. Hence kl 1+ 1 + -k2 1+ 1 + -k3 1+ 1 = 1 -_
so
Furthermore, since
k2 + 1 k3 + 1 (k2 + 1) (k3 + 1 ) 1 A [ ( kl + 1 ) 2 ' cos = !2 k 1 + k 1 _ 3+ 2+
1 00
3. GEOMETRY
we observe that cos A depends only on kl and k3 • Hence cos A = cos Al and, analo gously, cos B = cos BI , thus triangles ABC and AIB I CI are similar. Romanian Regional Mathematical Contest " Grigore Moisil" , 1999; Revista Matematica Timi§oara ( RMT ) , 1999, pp.
(Dorin Andrica,
87)
9. Using Steiner's theorem, we obtain
MB · NB AB2 = MC · NC AC2 Applying the AM-GM inequality yields MB MC
NB . / MB · NB AB + NC =2 � 2V MC . NC AC '
as desired. Note that the equality case occurs only if AM and AN coincide with the internal bisector of angle A.
( Titu Andreescu)
10. The quadrilateral APMN is a rectangle, therefore NP = AM. Hence NP is
minimal if AM ..1 BC, so M is the foot of the altitude from A.
( Titu Andreescu)
1 1 . Applying the law of cosines to triangle A l BI C we obtain
and using the inequality x2 get
+
y 2 - xy � xy, which holds for all real numbers x, y, we
AI B� � AI C , BI C. A
101
3.2. SOLUTIONS
�
Similarly, we obtain Bl Cr Bl A · CIA and CI A� three inequalities together, we obtain
� Cl B . Al B . MUltiplying the
Now the lines AAI , BBI , CCI concur, so and after substituting and taking square roots, we have Al Bl . Bl CI . CI AI
� Al B · BI C · CIA,
the desired inequality. Equality holds if and only if CAl CBl , ABI ACI and BCI BAI , which in turn holds if and only if P is the center of triangle ABC . IMO 1996 Shortlist)
=
( =Titu Andreescu,
=
1 2 . We start with the following lemma. Lemma. A, B, C
Let
tan
A
-
2
be the angle of a triangle ABC. Then
+ tan -2 + tan -2 tan -2 + tan -2 tan -2 B
B
C
A
C
= 1.
P f. We present two arguments. h. Since Firoorst approac
+ ,8) [1 - tan a tan ,8] = tan a + tan ,8, tan(90 ° - a) = cot a = l/ tan a, and A/2 + B/2 + C/2 = 90° , the desired identity follows from ( tan - tan - + tan - tan - = tan - tan - tan - ) = 2 2 2 2 2 2 2 = tan 2 tan (�2 + 2 ) [l - tan �2 tan C2 J = = tan � tan (90° - � ) [1 - tan � tan �J = tan(a
A
B
B
B
C
B
A
+
C
C
A C = 1 - tan '2 tan '2 ' Second approach. Let a, b, c, r, s denote the side lengths, inradius and semiperime ter of triangle ABC, respectively. Then SABe = rs, AP = s - a, and tan(A/2) = r/(s a) . Hence A ( e tan - = SAB 2 s s - a) -
.
Likewise,
B
tan 2
(s -eb) = -:sSAB
--
and tan
C
2
( s ec) . = sSAB -
1 02
3. GEOMETRY
Hence
A B tan - tan -
by Heron's formula.
2 2 + tan -B2 tan C2 + tan -C2 tan A2 = b) ) + + ( a c ( 8 ( 8 ( 8 ) ) S �BC b) = = 82 ( 8 - a ) ( 8 - ( 8 - c) 1 S�BCb ) = , 8 ( 8 - a )( 8 - ( 8 - c) -
0
-
A
Q
B
C
Without loss of generality assume that AP = min{AP, BQ, CR}. Let = tan( A;2 ), = tan( ii/2) , and = tan( C;2 ) . Then AP = BQ = and CR = The condition given in the problem statement becomes
/r z. y
z
and the equation in the lemma is
r/x,
2x + 5y + 5z = 6 ,
1 xy + yz + zx = . (1) and (2) yields Eliminating x from 5y2 + 5z2 + 8yz - 6y - 6z + 2 =
Completing squares, we obtain
r/y,
x
( 1) ( 2)
O.
(3 y _ 1 ) 2 + (3 z _ 1 ) 2 = 4 ( y - Z ) 2 . 3 1 3 ( 1)/3, z = (v + 1 )/3) gives Setting y - 1 = u, z - = v ( Le. , y = u + 5u2 + 8uv + 5v2 = 82 4 25 < 0, the only Because the discriminant of this quadratic equation is real solution to the equation is u = v = Hence there is 4only / 3) set of /3, yone= zpossible = 1 . Thus values for the tangents of half-angles of ABC (namely x = all triangles in S are isosceles and 4similar / 3 4/ 2 /3 andtoy =onez another. Indeed, we have x = r/AP = = r/BQ = r/CQ =2 1 = 1 , so 3 4 1 we can set r =5 , AP = AR = , and BP = BQ = CQ = CR = . This leads to 24 1 BC = . By scaling, all triangles in S are similar to the triangle AB = AC = and 558 with side lengths , , . O.
O.
x
103
3.2. SOLUTIONS
We can also use half-angle formulas to calculate C
. C= . B = sm sm
2 tan -
2
1 + tan2 C "2 3 : 4 : 5 and BC : AC : BC = 5 : 5 : 8. From this it follows that AQ : QB : BA = Alternative solution. By introducing the variables p = y + z and q = yz - 1 , ( 1) and (2) become 2x + 5p = 6 and xp + q = 0, respectively. Eliminating x relations
yields
p(6 - 5p) Note that
(3 )
+ 2q = O .
y and z are the roots of the equation (4 ) t2 - pt + (q + 1) = O . (3 ) ( 4) Expressing q in terms of p in , and substituting in , we obtain the following quadratic equation in t: 2t pt + 5p2 - 26p + 2 - 0 . O 2 $ . Hence the equation has real This equation has discriminant - (3p 2) 1/3. solutions if p = 2/3, and y = z = Note.only We can also let x = AP, = BQ, z = CR and use the fact that r (x + y + z) = SABC = Jxyz (x + y + z) _
Y
to obtain a quadratic equation in three variables. Without loss of generality, we may set = Then the solution proceeds as above. ( USA Mathematical Olympiad,
x Ti 1 . A d tu n reescu, 13. Let a,
2000)
b, c, A, B, C be the side lengths and angles of triangle ABC. Let X, Y, Z
be the feet of the perpendiculars from P to lines BC, CA, AB, respectively. Recall the inequality (the key ingredient in the proof of the Erdos-Mordell inequality): PA sin A � PY sin C
+ PZ sin B.
( 1)
104
3. GEOMETRY
BC PY PZ, BC. = = 90° , AZPYY Z PA A AP M . Z Y PX=. 90° . = = PZBX ZM = P Z B . Y = PY C . = Y Z � Y + M Z. 2R PA � cPY + bPZ. a bPB � aPZ + cPX and cPC � bPX + aPY. Using these Likewise, we have inequalities, we obtain PA PB PC PX ( b c ) PY ( c a ) PZ ( a b ) a2 + b2 + 7 � c3 + b3 + a3 + + b3 + a3 � 2PX 2PY 2PZ � be + co: + � (AM-GM inequality) 4SABC 1 abc Y=ZR ' BC and so on. This Equality in the first step requires that be parallel to ABC. Equality in the second step P occurs if and onlyb if is the circumcenter of ABC is equilateral and P requires that a = = c. Thus equality holds if and only if is its( Ti center. tu Andreescu, USA IMO Team Selection Test, 2000) H is the orthocenter of a triangle ABC then 14. It is known that if AH · BC + BH · CA + CH · AB = 4SABC . YZ
This says that the length of is greater than equal to its projection onto the latter being equal to the sum of the lengths of the projections of and onto as a diameter In fact, since AYP Azp is cyclic with of its circumcircle. By the Extended Law of Sines, sin Let and N be the feet of perpendiculars from and to the line Since 1fZp BxP , sin Similarly, N is cyclic. Hence x:iPZ 13 and sin Thus (1) is equivalent to N Multiplying by and using the Extended Law of Sines, (1) becomes
c3
A
3. 2. SOLUTIONS
1 05
I H. Al, Bl, C1 I H ' ' ' A, B , C A , B , C I BC, CA, AB , IA + lA' � AA1 , IB + IB' � BB1, IC + IC' � CC1 are strict. It follows that ' ' ' ·a IA + b · IB + c · IC > a(AA1 - IA ) + b(BBl - IB ) + c(CCl - IC ) ,
We prove that == Assume by way of contradiction that points and are distinct. Let be the feet of the altitudes from and let be the projection of onto the sides respectively. Hence at least two of the inequalities
or
which is false. Therefore 1 =
H and ABC is an equilateral triangle, as desired. ( Titu Andreescu, Revista Matematica Timi§oara (RMT) , No. 2 ( 1981) , pp. 67, 46 16) Problem 1 5 . Using standard notations, we have
64c2 + 49a2 + 9b2 - 112ac - 48bc + 42ab = 6c2 - 6a2 - 6b2 .
This is equivalent to
15b2 + 2b(21a - 24c) + 55ab2 - 1 12ac + 58c2�= O.0 Viewing this as a quadratic equation in , the condition � is satisfied. That is 441a2 - 1008ac + 576c2 - 825a2 + 1680ac - 870c2 � O. The last relation is equivalent to 6(64a2 - 112ac + 49c2 ) � 0, 8 7c. Substituting back into the given condition (8 7 ) 2 or a 3- C 7b� O. follows that a yields a = . We obtain It
=
106
3. GEOMETRY
hence triangle ABC is similar to triangle A' B' C' having sides 7, 3, 8. In this triangle 3 2 82 - 72 cos AI = . 3 . 8 = 2' It follows that A = A7 = Korean Mathematics Competition,
1
2+
60 ° .
2002) ( Titu Andreescu, Fi l i 16. rst so ut on. By the Cauchy-Schwarz Inequality, ..jPB2 + PC2 ..jAC2 + AB 2 2:: PB · AC + PC · AB.
Applying the (Generalized) Ptolemy's Inequality to quadrilateral ABPC yields
+ PC . AB 2:: PA . BC. Because PA is the longest side of an obtuse triangle with side lengths PA, PB , PC, we have PA y'PB2 + PC2 , and hence P A . BC 2:: ..jPB2 + PC2 . BC. P B . AC >
Combining these three inequalities yields
..jAB2 AC2
+
2:: BC,
implying that angle BAC is acute. With some careful argument, it can be proved that quadrilateral ABPC is indeed convex. We leave it as an exercise for the reader. Let D and Q be the feet of the perpendiculars from B and P to line AC, respectively. Then DQ � BP. Furthermore, the given conditions imply that Ap2 > Bp2 PC2 , which can be written as Ap2 - PC2 > BP2 . Hence, AQ 2 2:: AQ 2 QC2 = (Ap2 PQ 2 ) (Cp2 PQ 2 ) =
Note. Second solution. +
_
_
_
_
AP2 - PC2 > BP2 2:: DQ 2 . Let I be the ray AC minus the point A. Note that, since PA > PC, Q lies on ray If D did not lie on then AQ would be less than or equal to DQ, a contradiction. Thus, D lies on and angle BAC is acute. B =
l.
l,
l,
o
P o
o�----�o�----�o
A
D
Q
C
1 07
3 . 2. SOLUTIONS
Third solution. Set up a coordinate system on the plane with A (0, 0) , B (a, O) , C (b, c) , and P (x, y) . Without loss of generality, we may assume that ab > O0 and that c > O . Proving that angle BAC is acute is equivalent to proving that > . Since PA2 > PB2 + PC( 2 , ) x2 + y2 > x a 2 + y2 + (x _ b)2 + (y _ C)2 . Hence 2b ( ) 2b b ( o > x - a 2 - x + 2 + y - C) 2 � - x. O. follows that b > 0, as desired. Since P A > P B, we have x > � > Fourth solution. We first prove the following Lemma. F f i W X Y Z in the plane, Lemma. or any our po nts , , , =
=
=
=
_
It
Proof. Pick an arbitrary origin 0 and let w, x, y, z denote the vectors from 0 to W, X, Y, Z, respectively. Then I w - x l2 + I x - Yl2 + I y - Zl 2 + Iz - w l2 - Iw - Y l 2 - I x - Zl 2 w · w + x · x + y . y + z · z - 2I (w , x + x · y + y . z + z · w - w · y - x · z) w + y - x - Z 12 , which is always Equality holds if and only if w + y x + z, which is true Wnonnegative. XYZ is a (possibly if and only if degenerate) parallelogram. Applying the Lemma to points A, B, C, P gives 0 :::; AB2 + Bp2 + PC2 + CA2 _ AP2 - BC2 (PB2 + PC2 PA2 ) + (AB2 + AC2 - BC2 ) < 0 < + (AB2 + AC2 - BC2 ) AB2 + AC2 - BC2 . Therefore AC is acute. Fifth solutangle ion.° InBthis 0° 90° . Note - 1 takes on values between and solution, sin 90 , since PB < PA. Applying the Law of Sines to triangle PAB yields that -p;fjj < =
=
=
=
=
=
0
=
=
_
=
-
sin PAB =
It
follows that
Since PA2
PB - PB sin ABP :::; PA PA '
-
> PB 2 +
_ PB PAB :::; sm 1 PA ' PC2 , we have similarly •
- < ' - 1 PC . _ 1 VPA2 _ PB 2 . PAC _ sm < sm PA PA
1 08
3. GEOMETRY
Thus
-
-
--
1 PB 1 BAC � BAP + PAC < sin- PA + sin then If = sin - 1
()
Hence
��,
()) sin (90° -
=
() cos
=
-
JPA2 - PB2 PA
JPA2 - PB2 () . V'l - sin2 PA J . 1 PA2 _ PB2 =
. - 1 PB + smBAC < sm PA and angle B AC is acute. C
=
PA
90° ,
P
P A B
B
As we mentioned at the end of the first solution, the conditions in the problem imply that quadrilateral ABPC is indeed convex. Thus, the diagram on the right-hand side is not possible, but this solution does not depend on this fact.
Sixth solution.
/
I
/'
I
(
\
I
\
\
"-
--
...--
I
\
'-...
_
,/
'\.
I
/
_
""
B
;'
P
-....
--
C
,/
...--
./
I
Note that PA2 > PB 2 + PC2 . Regard P as fixed and A, B, C as free to rotate on circles of radii PA, PB, PC about P, respectively. As A, B, C vary, will be maximized when B and C are on opposite sides of line PA and and are right angles, i.e., lines AB and AC are tangent to the circles passing through B and C. Without loss of generality, we assume that PA > PB � PC. In this case, ABPC is cyclic and AB 2 = PA2 - PB 2 > PC2 , and similarly AC2 > PB 2 . Hence on
IfA1J Jfijp ;[(j'p
109
3.2. SOLUTIONS
the circumcircle of ABPC, arcs AB and AC are bigger than arcs PC and PB, respectively. Thus, IiPC > BAiJ. Because these two angles are supplementary, angle BAC is acute. B -
,.,
A
P /'
( Titu Andreescu, USA Mathematical Olympiad, 2001) 17. All angles will be in degrees. Let x
=
F0B. Then PiiC
=
80 - x. By the
Law of Sines (or the trigonometric form of Ceva' s Theorem) ,
PB PC sin � sin !!.!!. sin � = PA PB PC PA sin PAB sin PBC sin PCA x sin 40 cos 10 20 sin x sin 40 = sin sin10 sin(80 � - x) sin 30 4 sinsin(80 - x)
1
=
=
------
---------
B
A
C
b The identity 2 sin a cos
formula) now yields
1= so
=
b) b) sin(a - + sin(a + (a consequence of the addition
2 sin x(sin 30 sin 50) sin(80 - x)
+
=
sin x(1 2 cos 40) , sin(80 - x)
+
2 sin x cos 40 = sin(80 - x) - sin x = 2 sin(40 - x) cos 40. This gives x = 40 - x and thus x triangle ABC is isosceles.
=
20. It follows that AcE = 50
=
BAiJ, so
3. GEOMETRY
110
Alternative solution. Let D be the reflection of A across the line BP. Then triangle APD is isosceles with vertex angle 2 (180 - EPA) = 2 (PAii + AiiP) = 2 (10 + 20) = 60, APi5 = 2JiBA = 40. Since 1fAC = 50, we have DB .l AC. and so is equilateral. Also, ISiiA = B
C D Let E be the intersection of DB with CPo Then
= 180180CEi5 = 180 - (90 - AcE) = 90 + 30 = 120 and so PEi5 + I5A.J5 = 60. We deduce that the quadrilateral APED is cyclic, and therefore DEA = DPA = . 60 Finally, we note that I5EA = = 15EC. Since AC DE, we deduce that A across the line DE, which implies99that BA = BC, as desired. and (CTiare Asymmetric d 6) 1 tu n reescu, USA Mathematical Olympiad, 90° , B = 30° 18. Let max{ A, B} = A. If triangle ABC is right-angled, then A = 60° . In order to find !!:. , we may assume that ABC is the triangle with sides and C = r 1 2b 1 a = , = , c = y'3. We have R = and S y'3 2 2 r - - + 12+ J3 - 3 +y'3J3 ' 3 + J3 v'3 1 R so = -- = +. r y'3 R J3 1 Conversely, assume that = r + . From the identity r = 4R sm· 2A sm. "2B sm. "C2 PEi5
-
-
-
.l
_
ABC
--
8
-
-
_
_
---
3.2. SOLUTIONS
it follows that or
111
r = 4(V3 + l)r sin � sin � sin �
Then
0 - 1 ( A - C A + C) . B = cos -2- - cos -2- sm 2" ° , we obtain A C 30 = and, since B B 1 V2 + J6 o4- = ( sin 2 ) sin 2 ' B · Lettmg sm . 2" = x Yl.elds 0 1 + V2 + 4- _ 0 J6 x2 -, x B 5° B 5° V2 V2 J6 . whose solutlOns are x = and x = 2 ' AIt follows that B2" =301 ° Aor 2"90=° . B The is not because � . Hence = , = and ABCacceptable, C = second 60° . Thussolution is right angled. triangle ( Titu Andreescu, Korean Mathematics Competition, 2002) ABC three circles equal to the circum 1 9. Construct in the exterior of triangle ABC circle that pass through two vertices P of the triangle. ( By the five-coin theorem the circles will have a common point , as desired see Dorin Andrica, Csaba Varga, Daniel 2002 , pp.Vacare�u, 5 1-56) . " Selected Topics and Problems in Geometry" , PLUS, Bucharest, Alternative solution. Let H be the orthocenter of triangle ABC. The reflections ABC. H of across the sides of the triangle are points of the circumcircle of triangle HAB, HBC, HCA are equal to the circumcircle of Therefore circles of ABC and forthe Pcircum H the claim holds. 2 ( 978) , pp. 74) (Dorin Andr=ica, Revista Matematica Timi§oara (RMT), No. 1 A'B' , B' C' , C' A' by Co, Ao, Bo, respectively, and 20. Denote the midpoints of the ' C' perpendiculars in question by le, lA , lB. Consider the centroid of triangle A'Bthree . -4-
4
_
4
_
4
4
112
3 . GEOMETRY C
1 2,
Since AoG : GA' = BoG : GB' = CoG : GC' = : the dilatation h with center G and coefficient takes Ao, Bo, Co to A' , B' , C', respectively. Since dilatations carry straight lines into parallel lines, h transforms le into the line through C' perpendicular to AB. But C' is the point of tangency of the incircle and AB, so this line passes through the incenter of triangle ABC. The same applies to the images of l A and lB under h. Since the images of l A , lB, le under h are concurrent, so are lA , IB , le themselves. ( Titu Andreescu, Romanian IMO Selection Test,
-2
1986)
2 1 . Because triangle AI A2 A3 is not isosceles, it is not difficult to see that the
circumcenter of the triangles AI BI I, A2 B2 I, A3 B3 1 are defined. We start with a simpIe lemma. Lemma. Let ABC be a triangle with the incenter I. Let T be the circumcenter of the triangle BIC. Then T lies on the internal bisector of the angle A. Proof. Let us draw the external bisectors of the angles B and C as shown in the figure below. B
A
C
They intersect at the excenter E, which lies on the internal bisector of the angle A. Since BE ..1 BI and CE ..1 CI, the quadrilateral BECI is cyclic with the center of the circumscribed circle on IE. This center will be also the circumcenter of BIC. The lemma is proved.
3.2. SOLUTIONS
113
i
Let us prove the main statement. For = 1, 2, 3 we denote by the center of the Clearly, lies on the circle and by the circumcenter of the triangle By the lemma, also lies on the same bisector. Thus internal bisector of the angle are perspective from the point By Desargues ' and the triangles theorem these triangles are perspective from a line. This is to say that if we denote = 1, 2, 3, to be the point of intersection of the lines and then are collinear. But since is the perpendicular bisector the points these points are exactly is the perpendicular bisector of and of respectively. the circumcenter of the triangles A student not familiar with Desargues' theorem may proceed from the point as follows. Applying Menelaus ' theorem to the triangles respectively, one and to the triples of points can, observing usual agreement about the signs, write:
Q Ai+1 IAi+2 . i Oi Ai' Ti I. T1 T2T3 0i+1 0i+2 Ti+1 Ti+2 ' Qi, i Ti+1 Ti+2 QQ Q Bi li Ail 01 i,+120, i+32 A1 B1 I, A2B2I, A3 B3 I, Remark. 101 02 , 102 03, 103 0 1 (T1' T2 , Q3 ) , (T2 ' T3, Q 1 ) , (T3 ' Tt, Q2 ) , 01 T1 IT2 02 Q3 IT1 . 02 T2 . 01 Q3 - 1 , IT3 02 T2 Q3 Q 1 03 T3 . IT2 . 02 Q 1 - , IT1 03T3 0 1 Q2 01 T1 . IT3 . 03 Q2 - . MUltiplying them all one gets 0 2 Q 3 03 Q 1 01 Q2 0 1 Q 3 . 0 2 Q 1 . 03 Q 2 = QQ Q meansi that lthei points 1 , 2 , 3 are collinear. which Alternat ve so ut on. This proof will be based on inversion. We take the incenter I Ci
Ti 0 1 02 03
_
--
--
--
_
_
1
1
1,
to be the center of the inversion and the power of the inversion is arbitrary. Using primes to denote images of points under the inversion we have the following " dual" figure shown below.
11 4
B'I
3. GEOMETRY
B2' Gi is a straight line B�+I B�+2 ' with these lines Indeed, the imageBof the circle � B�B� . The line AiAi+1 will be transformed into the circle forming the triangle ri+2 , with the side AiAi+1 becoming the arc Ai Ai+1 which does not contain I. Note AAA I that all these circles have equal radii since the distances from to the sides of I 2 3 were equal. E I, E2 , E3 are three circles passing through the common point Let us note that if I and no two of them touch, then their centres are collinear if and only if there is i- I through which all these three circles pass. another common point Ei being the circumcircle of Ai Bi I. Since the We will useEthis observation for AiB� , the desired result is to show that the lines A� BL to the line Ainversion B� are iconcurrent. A� A� A� � B� , A�takes For this, it suffices to show that the triangles B� B� B� are homothetic, which is the same to say that their corresponding sides and r r r PPP are parallel. Since the radii of the circles I , 2 , 3 are equal, the triangle I 2 3 formed their centre has its sides parallel to the corresponding sides ofAthe triangle B� B�B�by. The A A� into I 2 � � / 1 centred at takes thePtriangle homothety of ratio P2 P3. Therefore the the triangle whose vertices are the midpoints of the triangle I A A A� � � and PI P2 P3 are also parallel and the corresponding sides of the triangles result( Tifollows. tu Andreescu, IMO 1997 Shortlist) I be the intersection point of lines GB' and G'B and let A' be the 22. Let AI and BG . intersection point of lines We have AB' AG' AI B'B + G' G = lA" I from Van Aubel 's theorem, therefore ::, is constant. Hence the locus of point is a BG line (segment dreescu) to . Titu Anparallel J
3 . 2 . SOLUTIONS
115
23. Without loss of generality assume that
PC = max{PA, PB, PC } .
The condition in the hypothesis is
PB . PC + PA . PC = PA . PB + 1 PC PA · PB + 1 · 1 or 1 - PA . 1 + PC . 1 . PACB is a From the converse of the second theorem of Ptolemy it follows that AB C P cyclic quadrilateral. Note that cannot be , or P otherwise the denominator of side equals OA. Hence the ABCright-hand B, C. the locus of point is the circumcircle of triangle without the vertices , ( Titu Andreescu, Revista Matematica Timi§oara (RMT) , No. 1 ( 1985) , Problem C7: 3) 24. We have Ja2 b2 4S2 + Ja2 c2 - 4S2 = Ja2 b2 - a2 b2 sin2 C+ + J-a2 c2 - a2 c2 sin2 B = ab cos C + ac cos B = = ab a2 +2ab2b c2 + ac a2 +2acc2 - b2 = a2 as desired. ( Titu Andreescu, Revista Matematica Timi§oara (RMT) , No. 8 ( 1971) , pp. 25, 1006) Problem _
25. The relation is equivalent to
+..j y'rb + Fc = -,1 r rarbre 1 1 1 1 or + + = -. Fay'rb FcFa Fay'rb r On the other hand, 1 1 1 1 -ra + -rb + -re = -r , so 1 1 1 1 1 1 + + = - + - + -. --Fa,y'rb y'rbFc FcFa, ra rb re Then ( 1 1 ) 2 + ( 1 1 ) 2 + ( 1 1 ) 2 = 0, - ..fib y'rb - Fc Fc - Fa ' It follows so ra( Ti that the triangle is equilateral, as desired. ( 974) = rb A= rdeFa, tu1903n )reescu, Revista Matematica Timi§oara (RMT) , No. 1 1 , pp. 2 1 , Problem Fa,
---
116
3 . GEOMETRY 26. If the triangle is equilateral the conclusion is true,
To prove the converse, we assume by way of contradiction that the triangle is not equilateral and say that Then
b f:. c, 2 a2 (b + C) 2 - a2 2 +� c b -2 =p(p - a) m= -4> 4 b � ) � and likewise m 2: p (p - , m 2: p(p - c) , It follows that 1 1 1 <1( 1 1b 1 ) m� p- p--b ab + p-m� + m� + - + p--c = -p2 + a + c + ca S2 On the other hand 1 1 1 1 _p2 + a2 + b2 + c2 �-r + -r� + -r� = -S2 [(p _ a) 2 + (p b)2 + (p C) 2] = ---=S2 b b b f:. 2 2 Since c then a + 1c + ca 1< a +1 b2 + 1c hence 1 1 < m2e ra + rb + re m2b + m2a + which d 2 ( 977) 66 ( Tiistua Acontradiction, 306n3)reescu, Revista Matematidi Timi§oara (RMT) , No, 1 , pp, , _
_
2
2"
_____
2'
Problem
27. We know that
,sm -A2 sm, -B2 sm, -C2 -< -81 ' A B C 'A'B' C so 2 cos -2 cos -2 -> sm sm sm , cos On the other hand, 'sm A + sm' B + sm' C = 4 cos "2A cos 2"B cos 2"C ' (1) gives so inequality A B C A B C sin + sin + sin 2: 4 sin sin sin , 1 Since the circumradius is , we have b = 2 sin B, c = 2 sin C, = a 2 sin A, (2) yields and relation a + b + c 2: abc,
as
claimed,
(1) (2)
3. 2 .
11 7
SOLUTIONS
Alternative solution. Let Zl , Z2 , Z3 be the afixes of points A, B , C such that I zd = 1 IZ2 1 = I Z3 1 = . We have BC = IZ - z3 1 , b = AC = IZ3 - zI i , c = AB = IZI - z 1 · a= 2
2
U sing the identity
Z; ( Z2 - Z3 ) Z� ( Z3 - zd Z� ( ZI - Z2 ) = ( Zl - Z2 ) (Z2 - Z3 ) (Z3 - Zl )
+
and triangle inequality it follows
+
abc = IZ I - z2 1 1z2 - z3 1 1z3 - zd ::; IZl l 2 1z2 - z3 1 IZ2 1 2 11z3 - zl l + IZ3 1 2 1z1 - z2 1 = = IZ2 - z3 1 IZ3 - zl l IZI - z2 1 = a b c.
+
+ + + + 2 (1978) 49 ( Titu Andreescu, Revista Matematidi Timi§oara (RMT) , No. , pp. , 35 0258 98 3 ( ) 1 1 1 11 Problem Gazeta Matematidi , Problem ; No. 2 (1988) , pp. 78; , Problem 21353) (GM-B), No. 28. For any positive real numbers x, y, Z we have 8xyz ::; (x + y)(y + z) (z + x) Setting = a - b + c, Z = a + b - c, x = -a + b + c, gives ( -a + b + c) (a - b + c) (a + b - c) ::; abc, Y
so
It follows that then Hence
3 <- 2 (p - a) (pabc- b) (p - c) = �2 . pabc S2 = abc E 6R = 6 4S . S = ' r desired. ( Titu Andreescu, Revista Matematidi Timi§oara (RMT), No. 2 (1974) , pp. 51 , 2028) Problem a2 L...J (p - b) (p - c) "
as
29. Using the inequality
3 (a2 + b2 + c2 ) � (a + b + C)2 ,
3. GEOMETRY a2 +4Sb22+ C2 � 3Sp22 ' 1 1 1 1 h� + h� + h� � 3r2'
118
we obtain Hence as
desired.
( Titu Andreescu, Revista Matematica Timi§oara (RMT) , No. 2(1977) , 66, 3062) Problem pp.
30. From the inequality
1 1 1 1 1 Jrcra + -Jrbrc + -Jrar1 b ' -ra + -rb + -rc >- -1 1 1 1 J J > J -r - -rcra + --. rarb rbrc + --
we obtain Then hence as
Jrarb + Jrbrc + Jrcra � 9r,
claimed.
( Titu Andreescu, Revista Matematica Timi§oara (RMT) , No. 2(1978) , 64, 3277) Pro blem pp.
3 1 . We have
and likewise
b2 + C2 a2 J (b + C)2 _ a2 "'; ( ) J 4 = pp-a ma = -2- - 4 �
It follows that
as
ma mbmc � p"';p(p - a)(p - b)(P - c) = pS = ra rbrc, desired. ( Titu A ndreescu, Revista Matematica Timi§oara (RMT), No. 1 ( 1978) , 64, 3276) pp.
Problem
32 . By the AM-GM inequality,
so
8p3 � 27abc. Hence
(a + b + C) 3 � 27abc, b c S 2 7R 2p2 > 27 a 4S - · -p = r,
3.2. SOLUTIONS
11 9 desired. ( Titu Andreescu, Revista Matematica Timi§oara (RMT) , No. 1 ( 1973) , pp. 43, 1 585) Problem as
33. We have
It follows that hence as
desired.
A 7r . A 1 _2 <- _6 sm 2 "2 ' V (P - bb)(P - C) < �2 ' e (p - b)(P - e) 4'be {:}
S;
S;
( Titu Andreescu, Revista Matematica Timi§oara (RMT) , No. 1 (1984) , pp. 67, 5221 ) Problem 0 O0 34. Let 1 , 2 , 3 be the centers of the three circles and S the area of the common
region.
0 1 , O2 , 03 which subtend the arcs three sectors with centers 0 0 02 0The 0 0 3, 1 3, 2 1 , respectively, cover the 2 surface of area S and twice more 0 0 0 r ) the surface of triangle 1 2 3 (which is � . On the other hand, the area of 2 these three circular sectors equals the area of a semicircle, which is � 7rr . Hence 1 2-7rr2 = S + 2 . r24J3 ' therefore 2 V3) . S = �2 r (7r (Dorin Andrica, Revista Matematica Timi§oara (RMT) , No. 2 (1978) , pp. 50 , 3522) Problem --
1 20
3. GEOMETRY 35. The parallel BD through C meets AB at point E. By Stewart's formula, we
obtain
AC2 · BE + CE2 . AB - CB 2 · AE = AB · BE · AE Because CE = BD and BE = CD, we deduce AC2 . CD + BD2 . AB - BC2 . (AB + CD) = AB . CD . (AB + CD) C
D
B
A
E
Drawing the parallel to AC through D and using similar computations yields BD 2 . CD + AC2 · AB - AD 2 . (AB + CD) = AB · CD · (AB + CD)
( 2)
(1)
( 1)
Subtracting the relation from gives (AC2 - BD 2 ) (AB - CD) = (AD2 - BC2 ) (AB + CD),
(2)
as desired.
(Dorin Andrica, Gazeta Matematidi (GM-B), No. 9( 1977) , Problem 6852; Revista 1 2 (1980) , pp. 64, Problem 4119) Matematidi Timi§oara (RMT), No. 36. Let I be the intersection point of the diagonals AC and BD. Since
AB · BD AB · AC and IB = AB + CD AB + CD the condition in the statement becomes IA =
90° , as desired. (Dorin Andrica, Revista Matematidi Timi§oara (RMT) , No. 2 (1978) , pp. 59, 3524) Problem Hence JfilJ =
37. Let ABCD be the trapezoid. Point I is the intersection of diagonals and
M, N are the midpoints of AB and DC. In a right triangle the length of the median corresponding to the hypothenuse is half of lenght of the the hypothenuse. Hence AB CD IM = and IN = .
2
2
3. 2. SOLUTIONS D F N C
12 1
B M E Then IM + IN = AB +2 CD = EF, which of the midline and hencelAB the lengthICD of the altitude.lAB It followsICD that IM andis theIN length therefore are also altitudes in triangles and and ABCD are isosceles. Thus is isosceles, as claimed. ( Titu Andreescu, Revista Matematidl Timi§oara (RMT) , No. 1 (1978) , pp. 48, 28 1 7) Problem A
38. From the Law of Cosines we deduce that
Note that
2AB . AC cos 1fAC = AB2 - BC2 + AC2 2DC · AC cos DcA = CD2 - AD2 + AC2 2AB . DB cos DiiA = AB2 - AD2 + DB2 2DC · DB cos Cfiiij = CD2 - BC2 + DB2
(1) (2) (3 ) ( 4)
1fAC = DcA, DiiA = Cfiiij , (2) (3) and (4) yields ) ( 1 so dividing relations and , AB2 - BC2 + AC2 AB2 - AD2 + DB2 AB ' CD2 - AD2 + AC2 = CD2 - BC2 + DB2 = DC as desired. D i A di ) ( or n n r ca b the lengths of the bases, c, d be the lengths of the nonparallel sides 39. Let a, dd and 1 , 2 be the lengths of the diagonals. From Euler's theorem for quadrilaterals, it
follows that Hence
(d1 - d2 )2 + 2d1 d2 = (c - d) 2 + 2 (ab + cd) ,
1 22
3. GEOMETRY
ab +
and d1 - d2 = C - d implies d1 d2 = cd. From Ptolemy's Theorem, we deduce that the trapezoid is cyclic and so isosceles, as claimed. Revista Matematica Timi§oara (RMT) , No. pp. 48, Problem
( Titu Andreescu, 1 ( 1978) , 281 7) First solution. Assume the opposite. Then lAC - BDI > lAB - CDI or 40.
l AC - BDI > l AD - BCI. Without loss of generality, l AC - BDI > lAB - CDI , otherwise switch B and D. We have (1) AC2 - 2AC . BD BD2 > AB2 - 2AB . CD CD 2 and, from Euler's relation,
+
+
(2) where M and N are the midpoints of AC and BD, respectively. From and (2) , AD 2 + BC2 - 2AC · BD > 4MN2 - 2AB · CD.
(1)
(3)
Let P be the midpoint of AB. Then NP = AD /2, MP = BC /2 and since MN � INP - MPI , it follows that 4MN2 � (AD - BC) 2 . (4)
(3 )
From and (4) , -2AC · BD > -2AB · CD - 2AD · BC, in contradiction with Ptolemy's Theorem. We are done. The cyclicity is essential. The inequality fails if ABCD is a parallelogram. Let E be the intersection of AC and BD. Then the triangles ABE and DCE are similar, so if we let x = AE, = BE, = AB, then there exists k such that kx = l)E, = CE, = CD. Now
Note. Second solution.
y z li l AB - CDI = I k - z and ) (k )1 1 1 l l AC - BDI = I (kx + y - y + z = Ik - . Ix y · Since I x - y l ::; z by the triangle inequality, we conclude that l AB - CDI � lAC - BD I , and similarly IBC - DAI � lAC - BD l. These two inequalities imply the desired Thirresult. d solution. Let 20', 2,8, 2,, 28 be the measures of the arcs subtended by AB, BC, CD, DA, respectively, and take the radius of the circumcircle of ABCD 1 to be . Assume without loss of generality that ,8 � 8. Then 0' + ,8 + , + 8 = 7r, and ky
kz
-
(by the Extended Law of Sines)
lAB - CDI = 21 sin - sin
a
1'1 = I Sin a; l' cos a � l' I
1 23
3.2. SOLUTIONS
and
lAC - BD I = 2 1 sin(o: + t3l - sin(t3 + "Ill = I Sin 0:; "I cos (0:; + t3) I . 0 ( + ) / 2 ( + ) /2 + (3 7r/ 2 (by the assumption (3 6) and the Since [0 , / 2] , we conclude that l AB cosine function is nonnegative and decreasing on A C A C BD A D BC CD I � l - I , and similarly l - I � l - BD l . ( Titu Andreescu, USA Mathematical Olympiad, 1999) E be the intersection point of the diagonals. Consider AD < BC the 41 . Let AB the altitude. Since basis of the trapezoid and AB2 = AD . BC, then AB BC AD = AB ' ABC and ABD are similar. so the right triangles "I
::;
a
,
::;
a
,
::;
7r
::;
On the other hand we have
BCA + CAB = 90°
-
hence
-
AEB = 90° so the diagonals are perpendicular, as claimed. ( Titu Andreescu, Revista Matematica Timi§oara (RMT) , No. 2 (1972) , pp. 28, 1164)
It follows that
-
Problem
I the intersection point of the diagonals AF and BE of the rectangle ABF42.E. LetNoticebethat NI is the median of the right triangle ANF with hypothenuse AF, so IN = AF2 = IA Likewise, BE IB IM = BE2 = IE ' IQ = AF2 = IF, fP = 2 = . fA = IE = IF = IB, it follows that 1M = IN = IP = IQ . Hence MNPQ Since
is cyclic, as desired.
124
A
3. GEOMETRY B
Q D E ( Titu Andreescu)
F
C
< al :::; a2 :::; a3 :::; a4 < al + a2 + a3, then there is a cyclic quadrilateral having side lengths aI, a2AB , a3, aBC4. CD DA AC b d j Denote by a, , c, , e, , m the lengths of the segments BD , MN, respectively. Without loss of generality assume that ,b + d, � a +, c. , , P the midpoint of the side BC. The segments MP and NP are midlines Let beCAB BDC, so in triangles and MP = "21 a and NP = "21 c. Then 2m = 2MN < 2MP + 2NP = a + c so 2m < a + c < b + d. 0 On the other hand, if is the intersection point of the diagonals, we have b + d = BC + DA < BO + OC + DO + OA = AC + BD = e + j, b d j 2 hence m < a + c < + < e + j . 2 b d It suffices to prove that e + < m + a + c + + . Note that e < c + d, j < b + c, e < a + b, j < a + d. Summing up these inequalities yields e + j < a+ b + c+ d < a + b + c+ d + 2m and (the complete. Andisreescu, Tituproof 66 Revista Matematidi Timi§oara (RMT) , No. 1 ( 1978 ) , pp. , 43. By Sturm's theorem, we know that if 0
Problem 3288; Gazeta Matematidi ( GM-B ) , No. 10 ( 1981 ) , pp . 402, Problem C148 )
125
3.2. SOLUTIONS
44. Let I be the intersection point of lines BD and AF. The parallel to BD through C meets line AF at point T. First we consider AF .l BD and prove that AB .l CD. F
F
A
�------�
D
C
i) Assume that D lies on the segment CEo Then .ArC = we obtain On the other hand CTIIBD so
( 1 ) (2) ° 90 ,
90° . Since 1fAC : Ifi5C, (1) (2 )
Relations and imply ifAE : EaT , therefore EATC is cyclic. hence AB .l CD, as desired. It follows that BEG = ii ) Assume that C lies on the segment DE. In the right triangle CTF, TE is the median, so Because CTIIBD, we have
(3) ( 4) (5)
Ere : EaT.
EaT : CnB . Also,
(3)°, (4) , (5) , we obtain Ere : 1fAC. Hence ATEC is cyclic, then AEC = 90 , and AB CD, desired. JfiiC =
so from
.l
as
Conversely, consider that AB .l CD. i) If D is on the segment CE, then ME == AcE On the other hand AcE == Aci5 == AiiJ5 , so ME == AiiJ5 , and FBIE is cyclic. It follows that BiF = 1iEF = hence B D .l AF, as desired. ii ) If C is on the segment DE, then .
90° ,
(6)
3. GEOMETRY
126
( 7)
AcE := AiiJ.
( 7)
From (6) and we obtain ME := AiiJ, hence FEB! is cyclic. Note that lfEF = 90°, so ifiF = 90° and BD ..L AF, as desired. Revista Matematica Timi§oara (RMT) , No. 1 (1986) , pp. 106, Problem C6:4)
( Titu Andreescu,
45. Let
a, b, c, d be the side lengths of the quadrilateral and let S be its area.
Because the quadrilateral is cyclic, we have
S2 (p - a)(p - b)(P - c)(P - d) . =
N umbers
a, b, c, d are odd, hence p=
is an integer . If p is odd, then p p which is false. Hence p is even.
(1)
d a + b +c+ 2 ---
--
- a, - b, - c, - d are even and so 82 is divisible by 16, p
p
12 7
3.2. SOLUTIONS 1
1
b c d. Since S is a prime number, a (1) p - d =p - c = l4md2p - a2 = 2p - b = £ b 1 + 1 . Hence a = = + Summing up these equalities yields p = S so p = S p d and c = = S . The quadrilaterals are either rectangles or kites. Andreescu, 2 (1977) , pp. 66, ( Titurequired Revista Matematidi Timi§oara (RMT ) , No. 7 30 ) 6 Problem
Without loss of generality assume that � � � from relation we obtain
46. From the AM-GM inequality it follows that
ba cd < ( a + b +4 c + d ) 4 - -p41 4 . Hence 16abcd � p4 , or 8 (ac + bd)2 -p4 � 8 (a2 c2 + b2d2 ) . The desired inequality is now obtained from Ptolemy's Theorem: bd = eJ. ac+ ( Titu Andreescu, Revista Matematidi Timi§oara (RMT) , No. 3 (1973) , pp. 36 , 1811 ; Gazeta Matematidi (GM-B) , No. 8 (1980) , pp. 364, Problem 18370) Problem _
47. Let
m
be the length of the segment determined by the midpoints of the
diagonals. From Euler 's Theorem for quadrilaterals we have
( 1)
128
3. GEOMETRY
and from Ptolemy's Theorem, From relations
ac + bd = e f.
(2 )
( 1) and (2) we obtain
- c/ , / b - d/ } � 1 , then 2 = 1 + 1 � (a - C)2 + (b - d) 2 = (e - f) 2 + 4m2 � (e - f) 2 . Hence l e - fl � V2, desired. ( Titu Andreescu, Revista Matematica Timi§oara (RMT) , No. 2 ( 1978) , pp. 51 , 3527) Problem Since max{/a
as
48. The quadrilateral is cyclic so
S=
J(p - a)(p - b)(P - c)(P - d)
2 (�) , we have {j(p - a)(p - b)(P - c)(P - d) = P.2 = (p - a) + (p - b) + (p - c) + (p - d) 4 b Note that this is the equality case in the AM-G M inequality, hence p - a = p - = b d d p - c( Ti= p -A .dIt follows that a = = c = , so the quadrilateral is a 1square. tu213n6)reescu, Revista Matematica Timi§oara (RMT) , No. ( 1977) , pp. 24, Problem Since S =
�
SPAB ' SPCD = SPBC ' SPDA , since both are equal to PA . PB · PC · PD · sin P. The numbers SPAB, SPCD and SPBC, SPDA have the same sum and the same product, thus SPAB = SPBC and SPCD = SPDA or SPAB = SPDA and SPBC = SPCD, i.e. P is the midpoint of AC or BD, as desired. 4�. Observe that
( Titu Andreescu, Korean Mathematics Competitions, 2001 )
50. (i) Let MI and Mil be the projections of point M onto diagonals AC and
BD.
1 29
3.2. SOLUTIONS D
We recall the Simpson 's theorem: the projections of a point of the circumcircle of a triangle onto the sides of the triangle are collinear. Applying this result to triangles M', are collinear and yields that and are collinear. Hence the lines and meet at M' , as claimed. and we deduce that Mil (ii) From Simpson's Theorem for triangles the conclusion follows. and Since Mil is a point of is on the lines Revista Matematidi Timi§oara (RMT), No. pp. Problem Romanian Regional Mathematical Contest " Grigore MoisH" ,
C' , M' , D ' ABD BDC AC, 1 ( 1979) , 995)54, 1 1 AC BD 5 1 . We have 2 . , SABCD AC BD with equality if and only if . Since 1 AC BD 2002 1-= SABCD 2 . 52 · 77 2002 PD ) 21 (AP +ACPC) . (BP + = 2- = , BD P . Thus, it follows that the diagonals and are perpendicular and intersect at AB V24 32 40 BC = V282 + 322 = 4Vl13, CD = V282 + 452 = 53, and ABCD is therefore DA == V4522 ++ 2422 == 51 . , The perimeter of 144 + 4V113 = 4(36 + V113) . ( Titu A ndreescu, American Mathematics Contest 12 (AMC 12 - Contest B) , 2002, 24) Problem ABCD . Assume without loss of gener 52. Let a bePA thePC side length of the square P A ) = . We have ality that m ( , y'2PA = PB + PD. A' B' 'A B' C'D,' AC , B' C' D'A' . (Dorin Andrica, 3855 ;
ABC DAC
S;
S;
S;
ax
S;
-
3. GEOMETRY
130
.j2PA = aPB + aPD , hence Then a BD · PA = AD · PB + AB · PD . PDAB is a cyclic From the converse of the Ptolemy's Theorem it followsABCD that P quadrilateral, therefore lies on the circumcircle of square . Conversely, using theDPtolemy's Theorem we deduce that any point of the cir ABC cumcircle of square has the given P property.
P
ABCD .
It follows that the locus of point is the circumcircle of square , be a point with the given property and assume Let > Point are considered like in the diagram. > Note that so
Alternative solution. P (x y) y O . APC , B , C, DPA , V2PC PB PD = +
Squaring both sides yields
2X2 + 2 (y + b)2 = (x - b) 2 + (x + b)2 + 2y2 + 2y'(x - b)2 + y2 + 2y'(x + b)2 + y2 , then 2by = y'(x - b)2 + y2 + y'(x + b)2 + y2 , hence (x2 + y2 + b2 _ 2bx)(x2 + y2 + b2 + 2bx) = 4b2y2 . It follows that
so Thus
2 + y2 = b2 x P BD that contains A. Likewise, for and 0so point lies on Pthe semicircle of diameter BD that contains C and y :::; we deduce that lies on the semicircleABCD of diameter finally we obtain the circumcircle of square .
3.2. SOLUTIONS y A(O, a)
131
P(x, y) 0) D (a,
x
C(O, -a)
( Titu Andreescu, Romanian IMO Selection Test, 1981; Revista Matematica
Timi§oara (RMT) , No. 2(1981), pp. 87, Problem 4751) 53. Let ABCD be a quadrilateral from the set
P and let
midpoints of sides AB, CD, BC, AD, respectively. The Euler ' s relation for the parallelogram
MNPQ is MP2 + PN2 + NQ2 + Q M2 = MN2 + PQ2 ,
or
C
M
A On the other hand, we have
MQ = DB , MP = AC 2 2
so Hence
B
M, N, P, Q be the
13 2
which is, clearly, a constant.
3. GEOMETRY
(Dorin Andrica)
54. From the law of cosines we derive
AB = 2R sin ;
or
AF = 2R sin �;. 2 The first equality is equivalent to .sm 512rr sm. 512rr ---rr + -rr- = 4, . sm sin 12 5rr 12. rr ' 1 = 4 sm sm ' 12 12 Furthermore, rr . rr or 1 = 2 sm' '6rr 1 = 4 cos sm
12 which is clear. For the second equality, we have
and
12
AC2 - 4R2 sm. 2 2rr AD 2 - 4R2 sm. 2 3rr AE2 - 4R2 sm. 2 412rr ' AF2 -- 4R2 sm. 2 512rr ' 12 ' 12 ' It reduces to 2rr . 2 3rr . 2 4rr . 2 5rr 5 2 2 rr . = sm - + sm - + sm - + sm - + sm 12 12 12 12 12 2 ' which dreescu, Revista Matematica Timi§oara (RMT) , No. 6(1971), pp. 27, ( TiistualsoA nclear. Problem 821) A A2 . . . An be the given polygon and let S be the area of AI A2 . . . An. 55. Let 1 M There is a point inside the polygon such that Ln MA% = 2S. k=1 We write n A A L = SA kM A k+l ' n+l = I . S k=1 •
Hence
-
-
3 .2. SOLUTIONS 133 In order to have equality everywhere we must have A MAk = MAk+l , k = 1 , 2, . . . , n. 1 sin k7i'i.A'k+ 1 = and M the circumcenter of a cyclic polygon and all sides subtent It follows ° Thatthatis theispolygon 90 is a square, desired. arcs (of Titu A. ndreescu, Revista Matematidi 2 ( 1978) , pp. 50, Timi§oara (RMT) , No. 3528) Problem ABC with the altitude AA' and the circumradius R the following 56. In a triangle equality holds: AB · AC = 2R · AA' . (1) A A A let PI , P2 , . . . , Pn be R Let be the circum radius of the polygon I 2 . . . n and P AA AA AA the projections of a point on the circumcircle onto the sides l 2 , 2 3, . . . , n l, respectively. ( ) 1 for triangles PAiAi+I , where An+l = AI , yields Applying PAi · PAi+1 = 2R · PPi , i = 1 , 2, . . . , n. as
Hence
which is a constant,
as
claimed. 2 ( 1980) , pp. 65, (Dorin Andrica, Revista 1 Matematidi Timi§oara (RMT), No. 41 23) Problem ( 1 ) from the previous problem to triangles MAI A2 , 57. Applying relation A M A MA3 A4 , MA5 6 , • • • , 2n I A2n , we obtain MA2n 1 . MA2n = 2R · MK2n l. Multiplying these equalities yields MA2n MKI . MK3 · · MK2n 1 -_ MAI · MA (2) 2 . . . 2 n Rn · For the triangles
relation
( 1 ) yields MA2 · MA3 = 2R · MK2 , MA4 • MA5 = 2R · MK4 , . . , .
3. GEOMETRY
134
Multiplying these equalities gives
MK2 . MK4 · · · MK2n = MAl . M2AR2 . . . MA2n (3) nn Similarly, we obtain MA2n MH1 . MH2 · · · MHn _- MAI · MA 2 · 2n Rn (4) A A M A A M A M A I n+ I ' 2 n+ 2 , . . . , n 2 n . by applying relation (1) to triangles 3 ) ( From , and (4) we draw the conclusion. Andrica,(2)Revista (Dorinequalities Matematica Timi§oara (RMT) , No. 2(1981), pp. 68,
Problem 4622)
58. Let x and y be the measures of the arcs subtended by the sides a and b,
respectively. We have nx + ny = 27r,
or
27r
x+y= -
.
n Let R be the circumradius of the polygon. Then x
. sm 2=
27r Now y = - x so
n
(
a ' y= b and sm 2 2R ' 2R
)
b X 7r 7r X 7r . x y SI. n 2 = sm ;; - 2 = sm ;; cos 2 - cos ;; sm 2= .
hence
.
2R '
b 7r X 7r SIn X . sm cos = - + cos -. 2 2R n 2 Squaring boths sides yields b X 7r . X 7r 2 X = b2 7r . 2+ 2- cos - sm - + cos2 - sm2 sm cos 2R n 2 2 2 4R2 or b b2 X 7r 7r . X 7r . X . 2. 21 - sm = + 2 - cos - sm - + cos2 - sm sm -. 2 2R 2 2 2 4R x a . ' Smce sm 2 = we 0b tam sin2 � (4R2 - a2 ) = b2 + 2ab cos � + a2 cos2 � . .
-
n
·
( n 2R '
Therefore
)
n
--
n
n
n
n
R=
(Dorin Andrica)
.
--
n
� Ja2 + 2ab cos :':n + b2 • 2 sin n
n
2
3.2. SOLUTIONS
59. We have from the Cauchy-Schwarz Inequality that
for any positive numbers Setting n = 4,
135
ai , bi , i = 1 , 2, . . . , n.
yields
3V " SPA BCD S2 - , V where is the volume of the tetrahedron. Then " SPBCDA 3VS2 = SS2 = � ' r r L....J
L....J
>
>
as desired. The equality occurs if and only if
a Jai bi i 1234 b i 1234 for = , , , hence i = a - I , = , , , . Then PAl = PBI = PCI = PDI , ABCD . P so is the kincenter of tetrahedron RTiemarA. The inequality holds for convex polyhedra circumscribed about a sphere. 982 ( tu ndreescu, 1 Romanian Selection49Test, ( 1 (1982) , pp.IMO82, Problem 10) ; Revista Matematidi Timi§oara RMT ) , No. i 1 i 4 0 and 60. All summations here range from = to = . Let be the circumcenter A A A A GA GA �= R be the 4 2 I · 3 circum radius of By the Power-of-a-point Theorem, . i 4 OG i 1 R2 - 2 , for � � . Hence the desired inequalities are equivalent to ( 1) and (R2 - OG2 ) L G� � L GAi • ( 1 ) follows immediately from i Now (3) �= V b;
3. GEOMETRY
136
) P denote the vector (4)
by the Arithmetic-Geometric-Mean Inequality. To prove (3 , let from to the point Then
0
P.
(4) vanishes. By Cauchy-Schwarz L GAi L GA1 2:: 16, and i GA1 i > "GA ·. GA1 i > -161 (" (GA · ) 2 ) 2 " 4-1 "GA2. " ). 3 ( from follows also (2) Hence ( Titu Andreescu, IMO 1995 Shortlist) )
This is equivalent to (3 , since the last term of Inequality, so
6
� 6
-
6
Z
6
-
6
�
C hapter 4 TRIGON OMETRY
PROBLEMS 1 . Prove that
27r 37r 7r ctg2 7 + ctg2 7 + ctg2 7
=
5.
2. Prove that
3 X + 47r X X + 2 7r cos3 - + cos3 -- + cos3 = - cos x 4 3 3 3 --
for all x E JR. 3. Evaluate the sum
Sn =
n- 1
L sin kx cos(n - k)x. k=1
4. Evaluate the sums
S1
=
sin x cos 2y + sin 2x cos 3y + . . . + sin(n
S2
=
cos x sin 2y + cos 2x sin 3y + . . . + cos(n - l )x sin ny.
-
l)x cos ny,
5. Evaluate the products
1) PI = (1 - tg1°) (1 - tg2 ° ) . . . (1 - tg8g 0 ) ; 2 ) P2 = ( 1 + tg1 ° ) (1 + tg2° ) . . . ( 1 + tg44°). 6. Prove that
(4 cos2 go - 3) (4 cos2 27° - 3)
=
7. Let x be a real number such that sec x - tan x 8. Evaluate the product
where Ixl
< 2n7r+2 • 1 39
tan go.
=
2. Evaluate sec x + tan x .
4. TRIGONOMETRY
140
9. Let a, b, c, d, x be real numbers such that x f:. k7r, k E Z and sin x sin 2x sin 3x sin 4x
=
a
--b
=
-
c
d
-'
10. Let a, b, c, d E [0, 7r] such that
2 cos a + 6 cos b + 7 cos c + 9 cos d = °
and
2 sin a - 6 sin b + 7 sin c - 9 sin d = 0 .
Prove that 3 cos(a + d) = 7 cos(b 1 1 . Prove that if
+ c) ,
arccos a + arccos b + arccos c = 7r,
then
1 2 . Let a, b, c be positive real numbers such that
ab + bc + ca = Prove that
-1
1
1.
-1
arctg + arctg - + arctg = 7r, b a c
13. Let x and y be real numbers from the interval
Prove that x + y = 2 ' 7r
cos2 (x - y) = sin 2x sin 2y
(o , �) such that
( i) such that 21 ( 1 - tga) (l - tg(3) ( 1 - tg,) = 1 - (tga + tg(3 + tg,) ,
14. Consider the numbers a , (3 "
E 0,
7r
Prove that a + (3 + , = 4"
( �), Prove that ( Si�2 a ) 2 + ( cos2 a ) 2 = 1
15. Let a, b E 0,
if and only if a = b.
sm b
cos b
141
4. 1. PROBLEMS 16. Prove that
sin3 a cos3 a + sin cos
b -> sec(a - b)
b
--
7r
b
for all < a, < 2" '
0
--
1 7. Let a, (3 be real numbers with (3 2:: 1. Prove that
(1 + 2 sin2 a) (3 + (1 + 2 cos2 a) (3
2::
2(3+ 1
for all a E JR. 1 8 . Let x be a real number, x E [- 1, 1] . Prove that
for all positive integers n .
_1_ < x2n + (1 _ x2 ) n < 1 2n- 1 -
19. Prove that
sec2 n x + cosec2 n x 2:: 2 n+ 1 for all integers n 2:: and for all x E
0
(0, %) .
20. Prove that
(1 + sin x) (1 + cos x) for all real numbers x.
:s
� + V2
2 1 . Find the maximal value of the expression
E = sin Xl COS X2 + sin x2 cos x3 + . . + sin x n cos X l , .
when X l , X 2 , . . . , Xn are real numbers. 22. Find the extreme values of the function f : JR
b f (x) = a cos 2x + cos x + b O b where a, , are real numbers and a, > .
-t
1R,
c,
c
23. Let ao, a I , . . . , an be numbers from the interval
tan ao -
(
Prove that
(0,
7r
/2) such that
�) + tan (a1 - � ) + . . . + tan (an - �) tan ao tan a1 . . . tan an
2::
n
n+ 1 .
24. Find the period of the function
f(x) = cospx + cos qx, if p, q are positive integers.
x E JR
2::
n
- 1.
12
4. TRIGONOMETRY
4
25. Let
aD = + + �
V3
for all n .
an+1 = 2(aa2nn-+25 for 2:: O. Prove that an = cot ( -2n3--37r) -2
v'6 and let
)
n
26. Let n be an odd positive integer. Solve the equation
cosnx = 2n-1 cos x.
27. Solve the equation
A sin2
x + B sin 2x + C =
0,
where A, B, C are real parameters. 28. Solve the equation
. cosy + s.m y cos z+sm. z cos x = 3 Prove that the equation . . 2xsm. 3xsm. 4x = 3 smxsm
'2
Slll X
29.
4
has no real solutions. 30. Solve the system of equations
=3 { 32sisinn xy ++ 23cosy cos x = 4.
{ xsix+yn y=+:1 - x cosy = V;
31. Solve the system of equations
4'
2
32 . Solve the system of equations
1
x+y+ Z = 437r tgx + tgy +tgz = 5 1 tgx . tgy . tgz = .
33. Prove that in any triangle
acosA + b cos B + c cos C = 2R2abc
4. 1 . PROBLEMS 34. Prove that in any triangle A B C A B C � cos3 "2 sin "2 sin "2 cos "2 cos "2 cos "2 � sm. 2 "2A ' �
1 43
�
=
35. Let n be a positive integer. Prove that in any triangle
L sin nA sin nB sin nC (_l) n+l + cos nA cos nB cos nC and L cos nA cos nB sin nC sin nA sin nB sin nCo ABC such that 36. Consider a triangle A B B C C A A sin sin + sin sin + sin sin =
=
=
and
( 1 + sin A) (l + sin B) (l + sin C) 2 (A + 1) AB C has a right angle. Prove that triangle ABC be a triangle such that 1 37. Let A > be a real number and let a>' cos B + b>' cos A c>' and a2>.- 1 cos B + b2>.- 1 cos A C2>'- 1 . Prove that the triangle is isosceles. ABC is equilateral if and only if 38. Prove that the triangle A B C 1 ( 2 b2 2 ) tg "2 + tg "2 + tg "2 a + +c . ABC be a triangle such that 39. Let in2 B + sin2 C 1 + 2 sin B sin C cos A. s ABC =
=
=
= 4S
=
Prove that triangle 40. Let
has a right angle.
ABC be a triangle such that
(cot � r (2 cot �r + (3 cot �r (�;)" , +
=
where s and r denote its semiperimeter and its inradius, respectively. Prove that triangle is similar to a triangle whose side lengths are all positive integers with no common divisor and determine these integers.
ABC
T
4. TRIGONOMETRY
144
41. Prove that in any triangle
A B C 9 . � sm - cos - cos - < - . 2 2 2 -8 L..J
42 . Prove that in any triangle
a-b2 + -b2 + -e2b - (sm. 2 -A + sm. 2 -B + sm. 2 -C ) . e ea a 2 2 2 >
4
43. Prove that in any triangle
� e3 - 16p3 . 44. Prove that in any triangle A 2B 2C 2 sec - sec - sec be 2 + ea _2 + ab 2_ p29 45. Prove that in any triangle 3V3. !!. ABC be a triangle. Prove that 46. Let 3A . 3B . 3C A - B B - C C A .sm < cos -- + cos -- + cos -- . + sm - + sm - 2 2 2 2 2 2 ( b) ( bi) 2002 = a- bi , a, b E 47. Find the number of ordered pairs a, such that a + cos A
a3
+
cos B
b3
+
__
__
cos C
>
__
> _. -
> r -
-
R
48. Find
Imz 5 z EC\1R Im5 z and the values of z for which the minimum is reached . min
-
ZI, Z2 , . . . , Z2n be complex numbers such that IZl l = IZ2 1 = . . . = IZ2n l and arg ZI arg Z2 . . . :::; arg Z2n Prove that 49. Let :::;
:::;
:::;
7r.
50. For all positive integers k define
Prove that for any integers m and n with 0 < m < n we have
Al A2 U
U···U
Am An m+1 U An m+2 U · · · U An . C
4. 1. PROBLEMS
145
Z1 ,Z2 ,Z3) be3 complex numbers, not all real, such that I Z1 1 = IZ2 1 = I Z3 1 = 1 Z2 + Z3 - Z1 Z2Z3 E JR.
5 1 . Let and 2 ( Z1 +
Prove that
� is odd and co, C1 , . . . , cn- 1 the complex roots of unity of order n. Prove that nII- l ( b % ) ( � b � ) 2 k=O ba + c = a + for any complex numbers a and . 53. Let n be an odd positive integer and co, C 1 , . . . , cn - l the complex roots of unity of order n. Prove that nII-1 ( b % ) n bn kb=O a + c = a + for all complex numbers a and . I 1 I 1 I 1 54. Let Zl,Z2 ,Z3 be distinct complex numbers such that Z1 = Z2 = Z3 = Prove that 1 1 1 1 I Z1 - z2 11z1 - z3 1 -- + I Z2 - zd lz2 - z3 1 + I Z3 - zl llz3 - z2 1 >- -. ---r2 I 1 I 1 I 1 55. Let Zl ,Z2 , Z3 be distinct complex numbers such that Z1 = Z2 = Z3 = r and Z2 f:. Z3· 52. Let n be an even positive integer such that
r.
Prove that
I 1 If Z is a complex number satisfying z3 + z - 3 :::; 2, the inequality show that I z + 56. 1 z -1 2. ( ) following property: 57. The pair ZI, Z2 of nonzero complex numbers has the O 2 there is a real2number 3 a E [-2, 2] such that z? - aZl Z + z� = . Prove that all pairs (zf, z�), n = , , . . , have the same property. AA A 1 58. Let 1 2 . . . n be a regular polygon with the circumradius equal to . Find IIn PAj when P describes the circumcircle. the maximum value of max j=l :::;
.
4.
146
TRIGONOMETRY
59. Let n be an odd positive integer and let interval [0, 71'] . Prove that
aI, a2, ... , an be numbers from the
L cos(ai - aj ) 2:: 1 --n · 2 ijn
l� < �
n be a positive integer. Find the real numbers ao and akl , k, 1 , n, l k > , such that L akl cos 2 (k - l) x sin2 -sm. -2nxx ao + 9
60. Let
=
for all real numbers x f:. m7l', m E Z.
1
SOLUTIONS 1 . We prove that
k7r n (2n - 1) ' 3 n+
(1)
� cot2 21 � k= l _
for all integers > O. Consider the equation
n
(2n + l)x 0, 27r with roots 7r 2n + 1 ' 2n + 1 ' . . . , 2nn7r+ 1 . (2 Expressing sin n + l)x in terms of sin x and cos x, we obtain 2n : 1 2 ( ( ) cos2n x sin x en : 1) COs2n-2 sin3 X + . . . sin n + l}x n 1 n 1 Sin2 n+ 1 X ( e : ) cot2n - e : ) cot2 n -2 x + . . . ) k7r 2 2 · SIn · 2n+l x 0 , we have , k - 1 , , . . . , n. Slnce Set x n + 12 ( n : 1) cat2n en: 1) cot2n-2 x + . . sin
=
_
=
X
=
-
=
--
-I-r
-
X
Substituting y = cot2 x yields l n y
_
.
= O.
en : ) en : l) yn- l + .
. . = 0,
-
with roots cot2 cot2 �, ficients and roots, we obtain
2n7r+_,1 2n + 1 . . . , cot2 2nn7r+ 1 . Using the relation between coef_ 2n + 1 ( ) n ( 2n - 1 ) k7r n 3 2 � cot2 n + 1 en : 1 ) 3 desired conclusion follows. 2 ( 979) , 51 , Dorin An ndr3,ica,theRevista (Setting Matematidi Timi§oara (RMT), No. 1- 1 �
=
=
pp.
Problem 3831)
147
148
4. TRIGONOMETRY
Applying the identity cos t = 4 cos3 3"t - 3 cos 3"t ' t E 1R for t = x, t = x + 271", t = x + 471" and summing up the three relations, we obtain X +-271" + cos3 X +-471" ) 3 cos x = 4 (cos3 3'X + cos3 3 3 X + 471" ) . -3 (cos -X3 + cos X +3 271" + cos -3 On the other hand, X +-471" = 2 cos 6471" cos 2x + 471" + cos x +271" + cos X + 271" cos 3"x + cos 6 3 3- = 3 2 x � 2" = 0 = (2 cos ; + 1) cos and (the follows. ir ca, Revista Dordesired in Andidentity Matematica Timi§oara (RMT) , No. 2(1975), pp. 44, Problem 2124) 3. We have 2.
--
28n = 8n + 8n = = sin x cos(n - l)x + sin 2x cos(n - 2)x + . . + sin(n - l)x cos x+ + sin(n - l)x cos x sin(n - 2)x cos 2x + . . . + cos(n - l)x sin x = = sin nx + sin nx . . . + sin nx = (n - 1) sin nx, n-1 = -- sinnx. 8 n (Dorin Andrica, Gazeta Matematica2 (GM-B), No. 8(1977), pp. 324, Problem .
+
+
so
16803; Revista Matematica Timi§oara (RMT), No. 2(1978), pp. 30, Problem 3055) 4. Note that 81 + 82 = sin(x + 2y) + sin(2x + 3y) + . . . + sin[(n - l)x + ny] and 82 - 81 = sin(2y - x) + sin(3y - 2x) + . + sin[ny - (n - l)x] . Setting x + y = hI and y - x = h2 yields 81 + 82 = sin(y + hI) + sin(y + 2hI) + . . . + sin(y + (n - l)hI ) = .
.
149
4.2. SOLUTIONS h n l sm. [y + (n - I) 2hI ] .sm T .sm -h I S2 - SI = sin(y + h2 ) + sin(y + 2h22 ) + . . . + sin(y + (n - I ) h ) = 2
h l . [ ( h I ] . n h2 . [ ( h 2 ] n . SI = 1 sm T sm y +h n - 1) 2 I sm T sm y +h n - 1) 2 2 2 s. 22 sm. 21 and h; ] hl [ ( hI ] [ n n 2 � ( S 1 sin 2 sin Y +h n - 1) 2 + 1 sin sin Y +h n - 1) . 2-2 2 sm. 22 sm. 2I (Dorin Andrica, Revista Matematica Timi§oara (RMT) , No. 2(1977), 65,
Hence
m
_
Problem 3056 ) 5 . We have PI = 0 because of the factor 1 - tan 45 ° = O. On the other hand we have 44° + sin 44°) = P2 = (cos 1 ° + sincos1 0)1 °. .. .. .(cos cos 44° � cos 10 + � sin 10 � cos 440 + � sm 440
(
(.;2)
) ( • • •
pp.
)
= �--------------� 44 --�----------------�
cos 1 ° . . . cos 44° sin 46° . . . sin 89° (v'2) 44 = 222 . = ( Titu Andreescu) cos 1 ° . . . cos 440 2
= 4cos3 x - 3cosx, so 4cos2 X - 3 = -- for all x f. (2k +6. 1)We. 90°,havek Ecos3x cos x Z. Thus 27° . cos 8 1° cos 81° sin 9 ° (4cos2 9 ° 3)(4 cos2 270 3) = cos cos go cos 270 = cos 9° = cos 9° = tan 9° , desired. ( Titu Andreescu) _
as
_
cos3x
1 50
4. TRIGONOMETRY
From the identity 1 + tan2 x = sec2 x it follows that 1 = sec2 x - tan2 x = (sec x - tanx)(secx + tan x) = 2(secx + tanx), = 0.5. so secx Andxreescu, ( Titu+ tan American High School Mathematics Examination, 1999, Prob lem 15) 8 . Since 1 + tan2 2kx cos2 2kx (1 - tan2 2k x)2 - cos2 2k+l X for all Ixl < 2:+2 ' it follows that cos2 2k xl = cos2 2x Pn = IIn cos2 k= l 2k+ X cos2 2n+1 x ' 7.
(Dorin Andrica) 9.
Let
4x - A sin3x - sin2x - sinsin-x - d b a Then sin2 4x = 2sin2 2x(l - sin2 2x). Because sin2 4x = 2 sin2 2x(1 - sin2 2x), we obtain dA2 = 2b2 (1 A A2 b2 ). On the other hand, sin 3x = C, sin x = a, and since sin 3x = sin x (3 - 4 sin2 x), we have = (a(31 ) - 4A2 a2 ). A Eliminating from the relations and (2) yields 2a3 (2b3 - � ) = b4 (3a ) desired. (D i A d i ) _
_
_
_
c
.
_
c
-c ,
as
or n n r ca
as
Rewrite the two equalities 2 sin a - 9 sin d = 6 sin b - 7 sin 2 cos a + 9 cos d = - 6 cos b - 7 cos By squaring the two relations and adding them up we obtain 1 85 + 8 cos(a + d) = 85 + 42 cos(b + ) and the conclusion follows. 10.
c
c.
c ,
(1)
(2)
4.2. SOLUTIONS
( Titu Andreescu, Korean Mathematics Competition, 2002)
15 1
From the hypothesis it follows that b sinearccos a + arccos + arccos c) = o. Using the identity L cosacos,Bsin'Y = sin(a + ,B + 'Y) + sin a sin,Bsin'Y and the formulas sin(arccosx) = � and sin(arcsinx) = x, x E [-1, 1], we obtainsb a � + be� + eaJ1=b2 = )(1 - a2 )(I - b2 )(I - c2 ) , as desired. ( Titu Andreescu, Revista Matematica Timi§oara (RMT) , No. 2(1977), pp. 64, Problem 3054) 1 2 . The identity arctgx + arctgy + arctgz = arctg 1 x-+(xyy ++zyz- +xyzzx) + k7r implies b + be + ea - 1 1 1 1 a b + e) + k7r. b = arctg arctg + arctg + arctg e ( a ea + b a Because ab + be + ea = 1, we obtain arctg -a1 + arctg -1b + arctg -1e = k7r, where k is integer. Note that 0 < arctgx < "27r for all real x > 0, hence 1 1 37r 1 o < arctg � + arctg z; + arctg c < 2· Therefore k = 1 and arctg -a1 + arctg -1b + arctg -e1 = 7r, as claimed. ( Titu Andreescu, Revista Matematidi Timi§oara (RMT) , No. 1(1977), pp. 42, Problem 2827) 13. The given relation is equivalent to (cos x cos y + sin x sin y) 2 = 4 sin x sin y cos x cos y, or (cos x cos y - sin x sin y) 2 = 0 11.
4. TRIGONOMETRY
152 Hence
cos2 (x + y) = 0, and (since (0, %), we obtain x + y = %, as desired. Titu x,AnydEreescu, Revista Matematidi Timi§oara (RMT) , No. 1(1977) , pp. 42, Problem 2826) 14. Expanding yields 1 (1 - tana - tan,B - tan')' + tan a tan,B + tan,Btan,),+ "2 + tan,), tan a - tan a tan ,B tan,),) = = 1 - (tan a + tan,B + tan,),), or tan a + tan,B + tan')' - tan atan,B tan,), = = 1 - tanatan,B - tan,Btan,), - tan')'tana. (1) Since a,,B, ')' E (0, i), we have ° < a + ,B + ')' < hence tan a + tan,B + tan')' f= tan a tan ,B tan,),. From relation (1) we derive 1 - tan a tan,B - tan,B tan')' - tan,), tan a tan a + tan,B + tan,), - tan a tan ,B tan ')' = 1, therefore cot(a + ,B + ')') = 1. Hence( Tia +A,B +d ')' = i, desired. tu n reescu, Revista Matematica Timi§oara (RMT) , No. 1(1973) , pp. 42, Problem 158 2) 1 5 . The relation in the statement is equivalent to 4 a cos4 a ) = 1, (sm. 2 b + cos2 b) ( -sin cos b sm. -2-b + � or sin2 b sm4 a + cos4 a + -cos .sm-22-bb sm. 4 a + � cos b cos4 a = 1. It follows that sin2 bb cos4 a = 1 , . 2 acos2 a + -cos.-2-b sm. 4 a + � 1 - 2sm cos sm2 b hence COS b . sin b cos2 a) 2 = 0. ( --:-2 a sm sm b cos b Furthermore, cos b sm. 2 a = -sin b cos b cos2 a, sm b 7r ,
as
•
--
--:--
153
4.2. SOLUTIONS
or tan2 a = tan2 b. Because a, b E (0, -i), we obtain a = b. The converse and we are done. Alterna it ve soislutclear ion. From the given relation we deduce that there is a number e E (0, -i) such that cos2 a sin2 a -sm. -b- = sine and -cos--b = cose Hence sin2 a = sin b sin e and cos2 a = cos b cos e. It follows that 1 = cos(b - e) and cos 2a = cos(b + e) 2 Since a, b, e E (0, i), we have b e = ° and a = b + e, hence a = b, ( Titu A ndreescu, Revista Matematica Timi§oara (RMT) , No. 1 (1977) , desired.41 , Problem 2825 ; Gazeta Matematica (GM-B), No. 11 ( 1977) , 452, Problem 16934) 16. Multiplying the inequality by sinasinb + cosacosb = cos(a - b), we obtain the equivalent form 3 a cos3 -a ) (sin a sin b + cos a cos b) 2:: 1 . ( Sin -sm. -b- + -cos b But this follows from Cauchy-Schwarz Inequality, because, according 1to this in equality, d ) side is greater than or equal to (sin2 a + cos2 a)2 = . ( Ti theA left-hand as
pp .
pp.
tu n reescu
17.
Using the inequality
for m 2:: 1 we obtain (1 + 2 sin2 a)p + (1 + 2 cos2 a)p :::: 2 ( 2 + 2 sin2 � + 2cos2 a ) = 2,8+1 , desired. ( Titu Andreescu, Revista Matematica Timi§oara (RMT) , No. 1 (1974) , 30, Problem 1942) [ 1 1] 18. Because x E - , , there is a real number y such that x = siny. It suffices to prove that 1 >- sin2n y + cos2n y > � - 2n 1 P
as
pp.
-
4. TRIGONOMETRY
154
For the left-hand side note that I sin yl � 1 and I cos yl � 1, hence sin2n y + cos2n y � sin2n-2 y + COS2n-2 y � . . . � sin2 + cos2 = 1, desired. For the right-hand side we use the inequality x? + x� - ( Xl + X2 ) n 2 2 Hence sin2n y + cos2n y - ( sin2 + cos2 y ) n 2 2 claimed. Alternative Solution. By setting u = x2 and v = 1 - x2 the inequality becomes [0, 1] and u + v = 1, we have un � U, vn � v, 1 n n E u,v Because 1. u v + � � 2n- l un + vn � U + v = 1. Also, by the power mean inequality, implying 1_ un + vn - 2 ( u +2 V ) n = 2 ( !2 ) n = _ -l . 2n (D i A d i ) y
y
as
>
y
>
as
>
or n n r ca
We have sec2 x = tan2 x + 1 2:: 2 tan x and cosec2 x = cot2 x + 1 2:: 2 cot x by the AM-GM inequality. It follows that sec2n x + cosec2n x 2:: 2n (tann x + cotn x) . Since tann x + cotn x 2:: 2, we obtain sec2n x + cosec2nx 2:: 2n+l , desired. Alternative Solution. Using the AM-GM inequality we obtain sec2n x + cosec2n x - 2Jsec2n xcosec2nx = 2 sinn x1cosn x 12 = 2 n+ l n _ x - 2n+lNo. 3(1975), 104, Problem sin (Dorin Andrica, Gazeta Matematidi (GM-B), 19.
as
>
_ _
>
pp.
14900)
20.
We have (1
.
<
( 1 + sinx) 2 + (1 + COSX) 2
+ mnx) (1 + coo x ) 2 - 2 + 2(sinx + cos x)2 + (sin2 x + cos2 x) _
_
=
In . v
as
1 55
4. 2. SOLUTIONS 7r ) 3 In 3 3 . = 2 + (sm x + cos x) = 2 + 2 sm (x + '4 � 2 + 2, v
desired. Note for x = � + 2k7r, where k is integer. Solutionholds Alternathattiveequality . Expanding the left-hand side, we see that 1 + (sin x + cos x) + � sin 2x = 1 + ../2 sin (x + �) + � sin 2x �
- + ../2 + �2 = �2 + ../2. A d Ti 2 ( 1978) , pp. 47, ( tu n reescu, Revista Matematidl Timi§oara (RMT) , No. Problem 3500) 2 1 . We have E = sin X l COSX2 + sinx2 COSX 3 + . . . + sinxn cos Xl � 2 + cos2 n 2 2 2 2 -< sin Xl +2 cos X2 + sin X2 +2 cos X3 + . . . + sin Xn 2 Xl = 2 Therefore the maximal value of E is n2 and it is reached, for example, when 7r X l = X 2 = . .A. = X n = '4 ' D( orin ndrica, Revista Matematidi Timi§oara (RMT), No. 2( 1977) pp. 65, Problem 3058) b f b 22. Because a, > 0, it follows that the maximal value of is a + + Setting y = cos X yields f (x) = a(2y2 - 1) + by + c = 2ay2 + by + c - a. If :a E [- 1 , 0) , then the minimal value of f is � b2 + 8a(c - a) 8a 8a b ) If - 4a E ( - 1 , then the minimal value of f is f ( - 1 ) = 2a - b + c - a = a - b + c. D i A di 1 ( 1981) , 52, ( or n n r ca, Revista Matematidi Timi§oara (RMT) , No. Problem 4315) Fi S l i b 1 23. rst o ut on. Let k = btan( ak - 7r / 4) , k = 0, , . . . , n . It follows from the 1 1 hypothesis that for each k, - k and <
1
,
c.
-
-
- 00 ,
pp.
<
<
1 + b k � L ( 1 - bl ) . 09¥k5;n
( )
1
4. TRIGONOMETRY 156 thek Arithmetic-Geometric-Mean Inequality to the positive numbers 1 bz, I Applying k 1 1 1 = 0, , . . . , - , + , . . . ,n, we obtain ( 2) L ( 1 - bl ) � n ( II ( 1 bl )) l in 09::fk �n O�l::fk �n 2 ) ) ( ( 1 From and it follows that ( n ) lin n !! ( 1 + bk ) � nn+ 1 Do ( 1 - bz ) n , _
and hence that Because
1 + bbk 1 + tan (ak - �) -1 - k -- 1 - tan (ak - �) = tan ( (ak - �) + �) = tan ak , the conclusion Second Solutfollows. ion. We first prove a short lemma: Let W,X,Y,z be real numbers with x + y = w + z and I x - y l < I w - z l . Then wz
and let
::;
Then - 1 < bk < 1 and
(1) tit. = G � :� ) G � :: ) = 1 + b1 + bbi�' _ 1 j+ +k b b b b 1 1 1 < < . . . + n � nb - b, it follows First note that because k and o + + 1 b b for all 0 j, k n with j f:. k . Next note that if j + k > 0 and that bj f:. bjk,+thenk >it0follows ( 1) that the value of tj tk can be fromb the lemma applied to closer together and with the made smaller by replacing b j and bk by two numbers b b < same sum. In particular, if j 0, then replacing j and k by their average reduces the problem to the case where bi > 0 for allb i . We . may now successively replace the i 's by their arithmetic mean. As long b the i are not equal, one is greater than the mean and another one is less than the mean. We can replace one of this pair by the arithmetic mean of all the bi 'S, and the ::;
::;
as
4.2.
1 57
SOLUTIONS
other by a positive number chosen so that the sum of the pair does not change. Each such change decreases the product of the ti 'S. It follows that for a given sum of the bi 'S, the minimum 1 producti is attained when all of the bi 'S are equal. In this case we n have bi - -n + 1' for each , so >
tO t1 . . . tn ;:::
( : : �) n+1
n+ 1
=
C;) n+ l = nn+ 1 .
This proof. Solution. the Thirdcompletes We present a solution based on calculus. We set a = bo + b1 + . . . + bn , where - 1 < bi < 1 , and assume that a ;::: n - 1 . We then show that the product lIn 1 + bk k=O 1 - bk attains its minimum when all of the bk 's are equal, that is, their common value is l( a n + 1). The desired inequality will follow immediately. We proceed by induction. The2 case n = 1 was established in the discussion of (1 ) in the previous solution. For n ;::: , set ' -1 nL bk = a = a - bn n - 2. k=O The last inequality follows from a ;::: n - 1 and bn < 1 . Set b = bn and c = a' ln , so b + nc = a. By the induction hypothesis, 1 + bk ) 1 + bn ( 1 + c ) n 11 + b . 1 k=O - bk 1 - bn - 1 - c - b Thus we need to prove that (1) ( 11 +- cc ) n (�) ( n + 1 + a ) n+ 1 , 1 - b n + 1 - a l 1) where the right-hand side is obtained by substituting a (n + for each bk , k = 0, 1, . . . ,n, in the product. Next, recall that a is fixed, and that b + nc = a. Thus we can eliminate b from ( 1 ) to obtain the equivalent inequality >
(IT
>
> _
n+ 1 ( G � � r G � : � :�) � :: � � : )
(2)
Now bring all terms in (2) to the left-hand side of the inequality, clear denomi nators, and replace c by x. Let the expression on the left define a function f with
158
4. TRIGONOMETRY
0 � x < 1 , I(x) attains its To establish (2) it isJ( sufficient to show that for minimum value at x = a n + 1) . Towards this end we differentiate to obtain ) t (x) = n (a - (n + l )x)((l + x)n- 1 (n + 1 - a) n+1 - ( 1 - x)n- 1 (n + 1 + a)n+1 (a - (n + l ) x) g (x) , = n where g (x) = (01 + x) n- 1 (n + 1 - a) n+1 ( 1 - x) n- 1 (n + 1 + a) n+1 . It is clear that a f' ( _ n +_1 ) , so we check the second derivative. We find 0 ( ll !" ( n : l ) = -n n + g ( n : l ) > , ' so I has a local minimum at x = (aJ() n + 1) . But I (x) could have another zero, t, obtained the equation g x = O. Because ' ( ) by solving )( ( 1 g x = n 1 + x) n-2 (n + 1 - a)n+1 + (n - 1 )(1 - x)n-2 (n + 1 + a)n+1 is( obviously one1))solution to (the equation positive for all x E [0 , 1 ) , there is at (most 0 ) ) J( l g x = in this interval. ItJ(is easy1) to check1 that g a( ) n + < 0 and g > O. Thus there is a real number t, a "n + < t < , with g' t = O. For this t we have I (t) = n (a - (n + l )t)g (t) < O. I , and no other extrema exist on the interval t Thus, is a local maximum for (0 , 1 ) . 1(1) � l (aJ(n + 1 )) . Note that the case x 1 The only thing left is to check that is also an extreme case with bo = b1 = . . . = bn- 1 = 1 . This case does not arise) in I( 1 our problem, but we must check to be sure that on the interval 0 � x < , x has a minimum at x = aJ(n + 1 ) . We have 1( 1) = 2n ( 1 + a - n)(n + 1 - a) n+1 � 0, since n - 1 � a � n + 1 , andJ(l (aJ(1n) + 1 )) = 0 (by design). Thus I(x) indeed attains a unique x=a n+ . Andreescu,at USA ( Tituminimum Mathematical Olympiad, 1998 , Problem 3) T d 24. Let be the greatest common divisor of p and q. We prove that = � is the lowest positive period of the function f . It is clear that I(x + T1 ) = I(x) I. for all real x, thereforeTT is 0a period of function T AT1 and l(x+ T1 ) = A 0 > > Suppose there are and an integer such that 1 T 0 I(x) for all real x. Then I( 1 ) = 1( ) = 2 , so cos pT1 + cos qT1 2, =
-
=
-
=
=
=
4.2. SOLUTIONS 1 59 therefore COSpT1 = COSqT1 = 1 . It follows that T1 = 2k 7r = 2k27r P q O k k 2 ' for some integers 1 T> . 27rA T AT Smce = 1 and 1 = d ' k1 k2 1 k (Ad) , q = k2 ( Ad) . = q = Ad and so p = 1 P d d(p, q) , so A = 1 , hence T = T1 desired. On the other hand, = gc (Dorin Andrica, Revista Matematidt Timi§oara (RMT) , No. 2 ( 1978) , 75, Problem 3695) l
•
as
25.
pp.
We have
7r 2 cos2 7r 1 + cos 17r2 cos 7r cot 24 = -sm.--247r- = 2sm. -7r cos24 -7r --=-==. -17r2 = sm 24 24 24 1 + cos (i - �) 1 + V; + 4 v: + VB + v'2 VB4 v'24 VB - v'2 sin (i - �) v'2) + ( VB + v'2)2 4(VB + v'2) + 8 + 4V3 4 ( VB + 6-2 4 = 2 + v'2 + V3 + VB = ao + 2. 2n-3 ) - 2 is true for n = O. Hence an = cot ( T 2n-3 ) , where b = an + 2, n � 1 . The It suffices to prove that bn = cot ( T n recursive relation becomes )2 5 ( bn - 2 bn+1 2 - 2b ' n b 2 -1 b 3 7r ' yields 2k -3or n+1 = ;b . Assuming, inductively, that bk = cotck, where Ck = n 1 bk+1 = cot2 2coCkt Ck- = cot(2Ck ) = cot Ck+1 , and (weTi areAdone. tu ndreescu, Korean Mathematics Competition, 2002) 1 26. If n = , then all real numbers x are solutions to the equation . ---���� = --����
=
_
_
_
_
_
_
1 60
4. TRIGONOMETRY
Let n > 1 and note that cosnx = (�) cosn X - (�) cosn-2 xsin2 x + . . . + 1 ) ( n 1) cos x sinn- 1 x. +(n We have two 1cases: � a) x f:. (2k + ) 2 for any integer k . Then I cosnxl = I cosxl l (�) cosn - 1 X - (�) cosn -3 sin2 X + . . . + ' ; ) n 1 (n � 1) sinn - 1 x l ::; +( ::; I cos x l ((�) I cosn-1 x l + (�) i cosn -3 X sin2 xl + . . . + (n � 1) I sinn -1 x l ) < n-l
-2-
_
hence there are no solutions in this case. b) x = (2k + l) i for some integer k. Then cos x = 0 and cosnx = 0, sinceAln is odd,i soS {l (2ki + 1 ) i l k integer } is the set of solutions. ternat ve o ut on. With the substitution x = i - y the equation becomes ( 1) cos (n � - ny) = 2n - 1 siny. Because n is odd, ( 1) is equivalent to ± sinny = 2n - 1 siny. Taking modules gives ( 2) I sinnyl = 2n -1 1 sinyl · But I sin nYI � n l sinyl for all y in lR, hence n l sinyl � 2n - 1 I sinyl · If y f:. k7r , k E Il , then n � 2n - 1, which implies n E { 1 , 3 } . The case n = 1 original equation reduces to cos3x = 4cosx, that is 4iscosclear3 X -and3 cosforx =n 4=cos3 x.theTaking 0 into account that cos x f:. , this yields cos2 x = �, which is not possible. It( Tifollows Il . that y = k7r , which gives the solutions x = (2k + l) i , k E304 A d 7 978 ( ) 1 tu n reescu, Gazeta Matematidi (GM-B),1 No2 ( 1. 980) , pp. , Problem 1 7297; Revista Matematidl Timi§oara (RMT), No. , pp. 63, Problem 4107)
4.2. SOLUTIONS
The equation(Ais equivalent to G ) + sin2 x + 2B sinxcosx + G cos2 X = 0 We have the following cases: i) A + G = 0 and G 1= O . Then 2B ' cos x = 0 or cot x = - 0 hence 1 )1r I k Z (2k 2 k 1 k E Z} . ; x E E { U { arcctg ( - g ) + 1r } ii) A + G 1= 0 and G = O . Then sm. x = 0 or tan x = - A2B+ G ' hence {k1r 1 k E Z } U { arctg (-}:c ) + k1r1 k E Z } . x E iii)) AA = GB = 0G = O . Then any real number is a solution. iv = = and B 1= O . Then sin 2x = 0 Zand so k� 1 k E } . x E { equation is equivalent to0 v) A + G 1= 0 and G (1=A O . The G G ) + tan2 x + 2Btanx + = , hence B ± JB2 AG + G2 A+G tan x = for B2 + G2 � AG . It{ follows thatk l k Z } { k 1 k E Z} arctgY + 7r E x E U arctgY1 + 7r 1 if B2(D+ Gi2 �AAGd .i Otherwise there are no solution. ( ) 1 ( 1978) , or n n r ca, Revista Matematica Timi§oara RMT , No. Problem 3429) 28. The equation is equivalent to 2 sin x cos Y + 2 sin Y cos z + 2 sin z cos x = 3, or (sin x - cosy)2 + (siny - COS Z)2 + (sinz - COS X)2 = O. It follows that sin x = cosy, siny = cosz, sinz = cosx. Hence
16 1
27.
-
_
pp.
89,
162
4. TRIGONOMETRY Z+X=
(4k3 + 1) '27r
for some integers k1 ' k2 ' k3 and therefore [4(k1 - k2 + k3 ) + 1] 7r ' y = [4(k1 + k2 - k3 ) + 1] 4"7r ' X= 4" and [4( k k k ) 7r k k k Z = - 1 + 2 + 3 + 1] 4" ' 1 , 2 , 3 E Z. ( Titu Andreescu, Gazeta Matematica (GM-B ) , No. 11 ( 1977) , 451, Problem 16931; Revista Matematica Timi§oara (RMT) , No. 1-2 ( 1979) , 52, Problem 3835) 29. Note that sin x sin 2x sin 3x sin 4x = � (cos 3x - cos 5x) (cos x - cos 5x) = 1 = 4 ( cos2 5x - cos 3x cos 5x - cos 5x cos x + cos x cos 3x ) = 1 6 3 = 8" ( 2cos2 5x - cos2x + cos8x - cos4x + cos6x ) < 8" = 4' hence( Tithe equation has no solution. A d tu n reescu, Revista Matematica Timi§oara (RMT) , No. 1 ( 1977) , pp. 41, Problem 2923) 30. Squaring both equations and summing up yields 4(sin2 x + cos2 x) + 9 (cos2 + sin2 y) + 12 (sinxcosy + sinycosx) = 25, or 13 + 12 sin (x + y) = 25. Hence sin (x + y) = 1, and so (4k + 1 ) '27r X+y= for some integer k. It follows that sin x = cosy and siny = cosx. Turning back to the system we obtain 4 .smx = cosy = 53 and smy . = cos x = 5 ' 4 hence 3 tanx = 4' tany = 3 · Note that sinx, cosx, siny, cosy are all positive, therefore x = arctan 43 + 2k7r pp.
Y
pp.
4.2. SOLUTIONS
163
4 217r and y = arctan 3' + , l k for some . Andreescu,andRevista 2 ( 1978) , 74, ( Tituintegers Matematica Timi§oara ( RMT ) , No. Problem 3694) [ 1 1] 31 . Observe that x E - , and x = sin (arcsin x) , � = cos(arcsin x) . From the first equation we obtain . .j2 cos(y - arcsmx ) = 2' then y - arcsin x = ± � + 2k1l". . x + y = "4 ' we get Usmg x + arcsin x = � ± � + 2k1l", k. for some Caseinteger 2k1l". Because x E [- 1 , 1] and arcsin x E [-i, iJ , we 1 . x + arcsin x = i have k = 0, hence x + arcsmx = 2' Therefore arcsmx = 2' - x, or x = cosx. For this equation there is only one solution Xo E (0, � ). The system has the solution Case 2 . x + arcsin x = 2k1l".x = Xo, = 4 - Xo Using similar arguments, k = 0, so arcsin x = -x. This equation has the unique solution x = ° so the system has the solution pp.
11"
11"
•
•
11"
Y
11"
-
x = o, ( Titu Andreescu, Revista Matematica Timi§oara (RMT) , No. 1 ( 1 977) , 41 , 2824 ) Problem pp.
32 .
Using the formula + tany + tanz - tanxtanytanz tan (x + y + z) = 1tanx - tanxtany - tanytanz - tanztanx
---=�---------=�--
----
4. TRIGONOMETRY
164
we have
- 1 = tan -37r4 = 1 - tan x tan y - tan5 -y1tan z - tan z tan x
-------
Hence
tan x tan y + tan y tan z + tan z tan x = 5. The equation
t3 - 5t2 + 5t - 1 = 0
has roots tan x, tan y, tan z, from the relations between the roots and the coefficients. On the other hand, the equation has the roots 1, 2 + V3, 2 - V3, hence {x, y, z} == 7r 57r 7r . 4' + k7r, + h7r, + p7r , for some mtegers 12 12 Revista Matematica Timi§oara (RMT), No. 8(1971), pp. 27, Problem 1018)
{ Ti A d ( tu n reescu,
}
k, l ,p.
33. Using the Extended Law of Sines we obtain a cos A +
b cos B + c cos C = 2R( sin A cos A + sin B cos B + sin C cos C)
==
= R(sin 2A + sin 2B + sin 2C) = R(2 sin(A + B) cos(A - B) + sin 2C) = A-B-C A-B + C = cos = 2R sin C(cos(A - B ) + cos C ) = 4R sin C cos 2 2 = -4R sin C cos as desired.
( i - B) cos ( i - A)
==
4R sin A sin B sin C ==
;!�,
( Titu Andreescu, Revista Matematica Timi§oara (RMT), No. 2(1977),
Problem 3060)
34. Because A + B + C = 7r, we have
B A B C . C . "2 cos "2 = sm cos "2 + sm "2 cos "2 '
Hence
B C A . A C . cos - + sm cos - = sm - cos - ' 2 2 2 2 2 A B . B C A . cos - = sm cos - + sm - cos - ' 2 2 2 2 2 A . B B C . C - cos - sm cos - sm - cos 2 2 2 2 2 B . C A . A C cos - sm - cos - sm - cos 2 2 2 2 2
.A
C B . B A cos - sm - cos - sm - cos 2 2 2 2 2
= 0,
pp.
65,
4.2. SOLUTIONS
165
because the first column is the sum of the other two. Computing the determinant, we obtain A . B . C A A B C ' 2- sm - = cos - cos - cos sm cos3 - sm 2 2 2 2 2 2 2' as desired.
L (Dorin Andrica)
L
35. Denote
E1 = L sin nA sin nB cos nC
Observe that
and
&
=
L cos nA cos nB sin nCo
(cos A + i sin A) (cos B + i sin B) (cos C + i sin C) = = cos(A + B + C) + i sin(A + B + C) = = cos 7f + i sin 7f =
By de Moivre's formula,
- 1.
(cos nA + i sin nA) (cos nB + i sin nB) (cos nC + i sin nC) = ( _ l) n . Expanding the brackets yields
- E1 + iE2 + cos nA cos nB cos nC - i sin nA sin nB sin nC = ( _ l)n . Hence E1 = ( _ 1) n+1 + cos nA cos nB cos nC and = sin nA sin nB sin nC. (Dorin Andrica, Revista Matematidi Timi§oara (RMT), No. 1(1978) , pp. 65, Problem 3278) 2 36. Subtracting from the second equality the first multiplied by yields O 1) (sin A - (sin B - l) (sin C - 1) = . 1 , so AB C is a right triangle. Hence sin A, sin B or sin C is A d Ti ( tu n reescu, Revista Matematidi Timi§oara (RMT), No. 2(1978), pp. 49, 1 &
Problem 35 4)
37. Note that
a cos B + b cos A The system of linear equations
{ aa2A
b
- c = O.
cos B + cos A - = 0 =0 cos B + cos A cos B + cos A
a A- 1
bA
c A b2 A- 1- c - C2 A- 1 = 0
4. TRIGONOMETRY
166
has the solution (cos B, cos A, - 1) and is homogeneous. Therefore the determinant Ll = is zero. On the other hand,
a a2A-A 1 a
b b b2AA- 1
c c2AA- 1 C
1 1 1 b b 1 A-1 A A Ll = a c ( a ) ( b ) ( c - 1) = A-1 2 A- 1 2 cA-1 2 a b ( b )( ) )(b = a c aA- 1 _ A- 1 aA - 1 _ cA - 1 A - 1 _ cA - 1 . b b or c = a, hence the triangle is isosceles. Therefore (Dorina A=nd,rica,= cRevista Matematidi Timi§oara (RMT) , No. 2(1977), pp. 89 , Problem 3199)
38. The relation is equivalent to
J(P - b)(p - c) J(P - c)(p - a) J(P - a)(p - b) 1 2 2 2 Jp(P - a) + Jp(p - b) + Jp(p - c) = 4S (a + b + c ) or 1 L( P - a)(p - b) = 4S1 (a2 + b2 + c2 ) . S Expanding the brackets yields _p2 + ab + bc + ca = 41 (a2 + b2 + c2 ) , then 4(ab + bc + ca) = a2 + b2 + c2 + (a + b + C)2 . It follows that (a - b)2 + (b - C)2 + (c - a)2 = 0, b hence( Tia = A =dc. tu n reescu, Revista Matematica Timi§oara (RMT) , No. 2 (1972) , pp. 28, ....--;= .:..-..: 7='====;�
-
-
Problem 1160)
39. By the Extended Law of Sines,
On the other hand, so
a = 2R sin A, b = 2R sin B, c = 2R sin C. sin2 B + sin2 C = sin2 A + 2 sin B sin C cos A.
4. 2. SOLUTIONS
167
From the hypothesis we have sin2 B + sin2 C = + 2 sin B sin C cos A,
1
1 . It follows that A = �, hence the triangle ABC is right, as (Dorin Andrica, Revista Matematica Timi§oara (RMT) , No. 1-2( 1977) , 52,
therefore sin2 A = desired.
pp.
Problem 3838)
2 7
40. Because 6 2 + 32 + 2 = 2 and
s
A B C A B C + cot + cot = cot - cot - cot 2 2 2 2 2 the given relation is equivalent to 2 2 2 = (62 + 32 + 22 ) cot + 2 cot + 3 cot - = cot
r
-
-
2'
-
(1)
[ ( �) ( !) ( � ) ]
(
A
B
= 6 cot "2 + 6 cot 2"
+ 6 cot 2C ) 2
This means that we have equality in the Cauchy-Schwarz inequality. It follows that
A cot 2 6
__
B 2 cot 2
C
3 cot 2
2 3 7 ) ( 1 Plugging back into gives cot � = , cot � = �, and cot � = �. Hence by 28 130 Thus the side the Double angle formulas, sin A = !.- , sin B = , and sin C = �. 45 25 65 lengths 26, 40, and . ( Tituof ATnaredreescu, USA Mathematical Olympiad, 2002, Problem 2) 41 . Summing up the formulas
ra = 4R sm. 2A cos 2B cos 2C ' rb = 4R sm. 2B cos 2C cos 2'A rc = 4R sm. 2C cos 2B cos 2A yields B C ra +4rb + rc '" Slll . A R " 6 2" cos 2 cos 2 = 4 R On the other hand ra + rb + rc = + r, hence 4R + r 1 r C '" sm . A cos B cos 2 = ----;w:- = + 4R " 2 6 2"
4. TRIGONOMETRY
168
Because � �
r,
it follows that A B C 1 9 . < 1 + '" sm cos - cos - = L...J 2 8 8' 2 2 -
as
desired.
-
(Dorin Andrica, Revista Matematica Timi§oara (RMT) , No. 2(1978), pp. 49,
Problem 3510)
42. We have
or
Because � + �
a2 b e 2 cos A + - = - + - . be e b � 2, we obtain A a2 � 2(1 - cos A) = 4 sin2 "2 be
and likewise
b2 ' B -> 4 sm2 - ' 2 ae Summing up these inequalities yields
(
)
A . 2B . 2C � � 2 . 2> 4 sm + sm - + sm - ' -+-+-be ea ab 2 2 2 as
desired.
( Dorin Andrica)
43. We have
so Likewise, and
2be cos A b2 +1=2 2 a a 2ae cos B b2
+ ae2
2'
+ 1 - ab22 + eb22 _
2ab cos C 1 a2 b2 + - 2 + 2' e2 e e Summing up these equalities implies 2be cos A 2ae cos B 2ab cos C b2 3+ + + b2 a2 a2 e2 _
_
( + b2 ) + a2
4. 2. SOLUTIONS hence
169
be cos A
ea cos B ab cos C a2 + b2 + e2 -> �2 ' 3 cos A cos B cos C > -+ + -. 3 3 3 a b 2abe e
and moreover
--
By the AM-G M inequality,
'
--
( + ) = 16p813 '
3 3 3 3 � 2abe 2 a b + e
therefore
cos A
-as
cos B cos C > 81 + ---,;s + � - 16pS '
(Dorin Andrica, Revista Matematidi Timi§oara (RMT) , No. 2(1975), pp. 46,
Problem 2134)
44. We have
= (p
2=
B 1 be ea A , sec2 , sec2 2" = --A p(p - b) cos2 - P - a) 2 so it suffices to prove that 1 1 1 9 > -. p-a p- b p-e - p Setting = p - a, P - b, = P - e in the inequality
Y=
x
yields
sec2
2 = p(p C
ab
- e)
,
-- + -- + -Z (x + Y + Z) ( .!. + � + � ) � 9 x Y Z 1 1 1 ) > 9, P ( -- + -- + -- p-a p-b p-e
and the solution is complete.
(Dorin Andrica)
45. By the AM-GM inequality,
p Then
= (p - a) + (p - b) + (p - e) � 3 \!(P -
a ) (p - b) (P - e),
p3 � 27(p - a) (p - b) (p - e)
so
=
p4 � 27p(p - a) (p - b) (p - e) 2782 • It follows that p2 � 3V38, and since 8 pr, !!. � 3va, as desired. Revista Matematica Timi§oara (RMT), No. 2(1982), pp. 66, Problem 4993)
( Titu Andreescu,
= r
4. TRIGONOMETRY
170
0° < a, (3" < 90° and a + (3 + , = 90° . We have ' B-C . ( a + 2, ) = . . 3A - cos sm: - = sm 3 a - cos ((3 - , ) = sm 3a - sm "'2 2 A
B
C
46. Let a = 2 ' (3 = 2 ' , = 2 ' Then
and
= 2 cos(2a + ,) sin(a - ,) = -2 sin(a - (3 ) sin(a - ,) . In exactly the same way, we can show that C-A . ( (3 - , ) . 3B - cos - = - 2 sm ' ((3 - a ) sm sm "'2 2
A-B . 3C - cos . (, - a) Slll . (, - (3 ) . - = - 2 sm sm T 2 Hence it suffices to prove that
sin(a - (3) sin(a - ,) + sin((3 - a) sin((3 - ,) + sin(, - a) sin(, - (3 ) � O.
Note that this inequality is symmetric with respect to a, (3 " , we can assume without loss of generality that < a < (3 < , < Then regrouping the terms on the left-hand-side gives
0°
90° .
a)] , ° ° 0 0 9 which is positive = sin x is increasing for < x < . Alternat ive Sasolutfunction ion. We ykeep the notation of the first solution. We have sin(a - (3 ) sin(a - ,) + sin(, - (3 ) [sin(, - a) - sin((3 -
sin 3a = sin a sin 2a + sin 2a cos a;
cos((3 - a) = sin(2a + ,) = sin 2a cos , + sin , cos 2a; cos((3 - ,) = sin(2, + a) = sin 2, cos a + sin a cos 2,; sin 3, = sin , cos 2, + sin 2, cos , . It follows that sin 3a + sin 3, - cos((3 - a) - cos((3 - ,) = = (sin a - sin ,) (cos 2a - cos 2,) + (cos a - cos ,) (sin 2a - sin 2,) = = (sin a - sin ,) (cos 2a - cos 2,) + 2(cos a - cos ,) cos(a + ,) sin(a - ,). Note that sin x is increasing and cos x is decreasing for < x < Since o < a" , a + , < each of the two products in the last addition is less than or equal to O. Hence
90° ,
0
sin 3a + sin 3, - cos((3 - a) - cos((3 - ,) � O. In exactly the same way, we can show that sin 3(3 + sin 3a - cos(, - (3) - cos(, - a) � and
0
sin 3, + sin 3(3 - cos(a - ,) - cos(a - (3) � O.
90° .
4.2. SOLUTIONS
Adding the last three inequalities gives the desired result. USA IMO Team Selection Test, 2002, Problem 1)
Z
171
( Titu Andreescu, bi bi I l J 2 b2 47. Let z = a + , = a - , and z = a + • The given relation becomes 2002 =
z.
Note that
Z
from which it follows that
I z l (l z l 200l - 1) = O. Il ( b) I z i = 1 . In the case I z l = 1, we have Z2002 = Hence z = 0, and a, = (0, 0) , or I l 200 2 1 200 1 which is equivalent to Z 3 = z = z = . Since the equation Z 3 = has 2003 1 distinct solutions, there are altogether + 2003 = 2004 ordered pairs that meet the required Andreescu, American Mathematics Contest 12A, 2002, Problem 24) ( Tituconditions. a, b be real numbers such that z = a + bi, b i- O. Then 1m z5 = 5a4b 01 a248.b2 +Letb5 and � ) 4 10 � ) 2 Imz5 = ( b - ( b + 1. Im5 z 2 Setting x = (�) yields 5 2 ( 1)2 - 4. 1m Z 5 = 5x - lOx + 1 = 5 x 1m z ( 1 ± i ) , a i- O. 1 The minimum value is -4 and is obtained for x = Le. for z = a ( Titu Andreescu, Revista Matematica Timi§oara (RMT), No. 1(1984) , pp. 67, Problem 5221) Ml , M2 , . . . , M2n be the points with the complex coordinates 49. Let AA A MM Mn Mletn+l. I , 2 , . . . , n be the midpoints of segments l 2n , MZl 2, ZM2 ,2n l,, Z2n , and , Z
z·
5
. • .
.
. •
172
4. TRIGONOMETRY
M i 12 The points i , = , n lie on the upper semicircle Mn Mandn+lwitarhe M2n l ,in. . .origin Ml M2n , Mcentered 2 , radius 1. Moreover, the lengths of the chords OAl , OA2 , . . . , OAn are increasing.- Thus in a decreasing order, hence Z1 , � Z2n 1 1 Z2 + ;2n-' 1 :::; ... :::; 1 Zn +2Zn+ 1 1 and Al the conclusion follows. ternative S1olut2 ion. Consider Zk = r(cos tk +i sin tk ) , k = 1, 2, . . . , 2n and observe that for any j = , , . . . , n, we have i ) 2] d2 2 [( = r costj +Cost2 n _j+ + (sin tj + s nt2 n _j+l = )] 22 = r [ + 2(cos tj cos t2 n-j+l + sin tj sin t2 n - j+l = 2 2 [1 ( )] 4 2 2 t2n-j+l2 - tj . = r + COS t2 n - j+l - tj = r cos l 2 t2 j - tj and the inequalities I Therefore Zj + Z2 n-j+l = r cos n - �1 I ZI + z2n l :::; I Z2 + Z2n-l l :::; . . . :::; IZn + zn+l l :::; :::; :::; are equivalent to t2n - t l � t2 n- l - t2 � . . . � tn+l - tn · Because 0 tl t2 . . . t2n :::;(D7r, the last inequalities are obviously satisfied. n Andrica, Revista Matematica Timi§oara (RMT), No. 1(1984), pp. 67, ori5222 ) Problem A 12 1 50. Let p = , , . . . ,m and 2let Z E p e Then zP = . l ,n-m+ , . . . , n are m consecutive integers, and, since p :::; m, Note that n-m+ 2 } k. k 1 there is an integer p divides E { n - m + , n - m)+' , . . . ,n suchA that A U k = kip. It follows that zk = (zp k = 1 , so Z E k n-m+l An-m+2 U Let A . . . R n , ask claimed. can be obtained by using the fact that emar . An alternative( solution )( 1 an(-k a1n)(- lk -1 1) . . . (an(- k+l1-) 1) a - a - 1 - 1) . . . akis an( Dinteger integers a > and n > . in Aforndrallica,positive Romanian Mathematical Regional Contest " Grigore Moisil" , or 1997) tk + i sin tk,)k E3{ I , 2, 3 } . 1R 5 1 . Let Zk = cos 2 ( The condition Zl + Z2 + Z3 - ZlZ2 Z3 E implies ) 3 ( ) (1) 2(sin t l + sin t2 + sin t3 = sin tl + t2 + t3 . :::;
:::;
C
U
4.2. SOLUTIONS 173 ( ) Assume by way of contradiction that max tl' t2 , t3 < i, hence tl , t2 , t3 < i' Let . functl. On IS . concave on [0 ' '67r ) ' so t = tl + t32 + t3 E (0 , '67r ) ' The sme ( 2) 1( . '3 sm tl + sm(.2) t2 + sm. t3 ) � sm. tl + t32 + t3 . From the relations (1) and we obtain ) ( 2 sin t l + t + t3 < . t l + t32 + t3 ' - sm then 3 2 sin t � sin t. It follows that 0 4 sin3 t - sin t � , 0 i.e. sin2 t � �. Hence sin t � �, then t � i, which contradicts that t E ( , i), (tl , t2 , t3 ) i, as desired. Therefore ( Titu Andmax 2)reescu, Revista Matematidi Timi§oara (RMT), No. 1(1986), pp. 91, Problem 586 2 (28 + 1) and b f:. 0, otherwise the claim is obvious. Consider a 52. Let n = 2 � and the polynomial complex number a such that a f = xn - 1 = (X - eo )(X - el ) . . . (X - en- I ) ' 2
;:::
=
We have
and hence
f
ioo ) a ion d , 1 + a (-' n (� r =
Therefore
(
•
•
•
( +
nII( - l + b � ) bn nII-l � bn nII- l ( 2 � ) a e = (� + e ) = a + e = k=O k=O k=O b b b ] [( ) = n f (T) f (-T) n a2 2s+1 + 1 2 = n [(� t+l + If 2s+1b + b2S+1 ) 2 ( � b �)2 ( b 2 2 a ( ) s+1 2s+1 = a + . 2000) (Dorin Andrica, =Romanian Mathematical Olympiad - second round, 0 b b 53. If a = 0, then claim is obvious, so consider the case when a f:. and f:. O. =
We start with a useful lemma.
174
4. TRIGONOMETRY h l
If co, C , , cn-l are t e comp ex roots of unity of order n, where n is an odd integer, then 1 n-l A B A B II ( + Ck ) = n + n , for all complex numbers A ank=Od B . Proof. Using the identity n- l nx - 1 = II (x - ck ) k=O A . for x = - B Ylelds An 1) n- l ( A ) ( B - n + = - !! B + Ck , and the conclusion follows. b 2 Consider the equation X + a = 0 with roots Xl and X2 . Since Lemma.
• • •
0
we have
nII-l ( b � ) bn nII- l ( }( 2 ) bn nII- l ( ) nII- l 2 ) a + c = k=O ck - Xt ck - X = k=O ck - Xl k=O (€k - X k=O A B B A Using the lemma for = -Xl, = 1 and then for = -X2 , = 1 gives nII- l ( } ( ) n 1 c k - xt = -X l + = 1 X� , k=O nII- l ( 2 ) ( 2 )n 1 1 ck - X = -X + = - x�. -
Hence
k=O
nII- l ( b T:) n a + c = b ( l - x�)(l - x�) = k=O = bn [l + (XlX2 ) n - (x� + x�)] = bn [1 + (�) n] = an + bn , }n = o. since( DXlXi2 =A �dand x? + x� = x? + (_xt or n n rica, Romanian Mathematical Olympiad - second round, 2000) 54. Consider the triangle with vertices of complex coordinates Zl, Z2 , Z3 and the R circumcenter origin of the complex plane. Then the circumradius equals IZI I = I Z2 1 = I Zin3 1 the = r and the side lengths are b I a = IZ2 - z3 1 , = Zl - Z3 1 , c = IZI - Z2 1 .
4.2. SOLUTIONS 175 The desired inequality is equivalent to 1 1 R12 -ab + -be + e-1a >- bc 48 4pr i. e. b a � a+ + e R2 = }f = ]f R � 2r,i which or ( D is Euler's inequality for a triangle. A d i or n n r ca, Revista Matematidi Timi§oara (RMT) , No. 2(1985), pp. 82, Problem 5720) AAA A 55. Let I, 2 , 3 and be the pointsAof complex coordinates ZI, Z2 , Z3 and let A A ) 2 2 3 ZA =A aZA + (1 - a z3, a E Hence point lies on the line and the triangle l 2 3 hasB its circumcenter in the origin ofA the complex planeA .A2 A 3 Al APoint � Al B ,issothe foot of the altitude from l in the triangle l . It follows that R
We have
I Z2 - z3 1 h I ZI - z2 11zl - z3 1 i A I ZI - z2 11zl - z3 1 · IZ2 2r- z3 1 8A 1 A 2 A 3 2 2 2 therefore h = IZI - z2 11zl - z3 1 ' 2r as desired. y _
_
s
n
l
_
(Dorin Andrica, Romanian Mathematical Olympiad - final round, 1984) I 1/ 1 56. Denote z + z by r. From the hypothesis,
4.
176
TRIGONOMETRY
3 ( )( 1 )2 :::; O . This implies Hence r3 :::; 2 + r, which by factorization gives r - 2 r + r 2,( Ti desired. 1987; Revista tu Andreescu, Romanian Mathematical Olympiad first round, 1 ( 1987) , pp. 75, Problem 6 191 ) Matematidl Timi§oara ( RMT) , No. � ? Zl 57. Denote t = 2 , t E
as
==
If
i
�
�
n
59. We will use the following auxiliary result:
177 4.2. SOLUTIONS 1) Let C be a semicircle of unit radius and (IMO 1973, Problem P P1 , PLemma. 2 , . . . , n points on C, where � 1 is an odd integer. Then I OP1 + OP2 + . . . + OPn l � 1 , 0 where is the center of C. idea is to show that the orthogonal projection of the vector sum f. The key OP OP OP1 Proo 2 +...+ + n onto some lme has length not less than 1 (see S . Savchev, T. Andreescu, " Mathematical Miniatures" , The Mathematical Association of America,l 1 2003, pp. 75) . Let 2k -0 P�. From the considerations of symmetry, the line �
--=t
�
--=-=-t
�
-::::-=+
•
n =
containing the middle vector the fact that n is odd!) .
A
n
is a natural candidate for such a line (here we use
o-------��--�
B
It is technically convenient to consider l as an axis with positive direction deter As is well known, the projection of the sum of several vectors is equal mined by to the sum of their projections. Hence it suffices to prove that the sum of the signed onto is lengths of the projections of and the orthogonal greater than or equal to Denote the diameter of C by and = and also projections of and onto by We have
OP� . OP1 , OP2 , . . . , OP2k -1 OP� , OP� , . . . , OP2k ! l AB 1. A B l A1 B1. OPk 1 OP1 + OP2 + . . . + OPk 1 � (k - 1) OA1' OPk+1 + OPk+2 + . . . + OP2k 1 � (k - 1) OB1. - 1 OP OB OA OP 1 This is because OA j � OB1 for j , . . . , k - and j � 1 for j 1 1 k + , . . . , 2k - . Since 1 + 1 0, the proof is complete. Consider the complex numbers 1 , 2, . . Zk cos O:: k + sin O::k , k PP P and the points 1, 2 , . . . , n with complex coordinates Zl , Z2 , . . . , Zn' =
=
i
=
0
=
.
,n
4. TRIGONOMETRY
178
Using the above Lemma we have . . . + l � 1 , or
Ikt=1 COSO:k itk=l O:k I
zn
+
It follows that as
I OP� + OP� + . . . + OP� I � 1, hence IZI + Z2 sin
desired .
� 1.
(Dorin Andrica, Revista Matematidi Timi§oara
Problem C:58)
+
(RMT) , No. 2(1983) ,
pp.
90,
60. Using the identities
sin nx cos ( n + l )-X · =--.--'------'S =� L..J cos 2 JX sm x
1 j= l
and
sin nx sin ( n + l ) x · =S2 = � --.---. 2JX L..J sm sm x
we obtain On the other hand ,
j= l S12 + S22
=
( sinsinnxx ) 2
S� + si = ( cos 2x + cos 4x + . . . + cos 2nx) 2 + + ( sin 2x + sin 4x + . . . + sin 2nx) 2 = =
n+
L:
(cos 2kx cos 2lx + sin 2kx sin 2lx) =
=x+2
hence
( sinsinnxx ) 2 = n + --
Set
l L: cos 2 ( k )x, 1 9< k�n -
l
cos 2(k - )x .
l �l
(Dorin Andrica, Romanian Mathematical Regional Contest " Grigore Moisil" , 1995)
Chapter 5 M ATHEMATICAL ANALYSIS
PROBLEMS
1
1 . Let :::; a < f3 be real numbers . Prove that there are integers m, n that a < ifiii < f3.
> 1 such
(an ) n�O and (bn ) n�o be the sequences of integers defined by (1 + . 1o3) 2n+1 = an + bn 3, n E ( ) (b ) Find a recursive relation for each of the sequences an n � O and n n � o , ( ) 3. Study the convergence of the sequence xn n � O satisfying the following prop erties:) 1 Xn > 1 , n = 0,1 1, 2, . . . 1 ) Xn + 1 ( 2) - Xn+1 - -- = -2 Xn+1 Xn - 1 , n = 0, 1 , 2, . . . ( ) 4. Study the convergence of the sequence Xn n � l defined by Xl E (0, 2) and � 1 = + V2xn - x X n+1 1 for n � . ( ) 5 . Consider the sequence of real numbers Xn n� l such that i + x� + . . . + X2n x = O. lim n�oo n Prove that Xl + X2 + . . + Xn = 0. 1m n�oo n Is the converse true? b ( ) (b ) 6 . Let an n � l and n n � l be sequences of positive numbers such that an > n n 1 for all n > . (an) n� l is increasing and (bn)n� l is unbounded, then the sequence Prove that if (Cn ) n� l ' given by Cn = an+1 - an , is also unbounded. ( ) 7. Let 0 < a < a be real numbers and let Xn n � l be defined by X l = a and 1) (a + Xn- 1 +10') 2 Xn = Xn- 1 + (a + , n � 2. 18 1 2. Let
. 10
v
1·
V
.
N.
182
5. MATHEMATICAL ANALYSIS
Prove that the sequence is convergent and find its limit. 8. Find a sequence
(an ) n2:l of positive real numbers such that ) ( lim an+l - an n-+oo (Jan+l - �) O . lim n-+oo = 00
and
=
(Xn )n2: l be an increasing sequence of positive real numbers such that X 1·1m n 0 n-+oo n h ( Prove that there is a sequence nk 2: l of positive integers such that 9. Let
2 =
.
(3 be real numbers and let (Xn ) n2: l , (Yn )n2: l , (Zn ) n2: l be real sequences such that � (3 } � max{ x + O'Yn , y + xn :::; Zn for all n ;::: l . 1 ( 2 (32 ) for all n ;::: 1 a) Prove that Zn ;::: - 8 0' + . (3 ) ( ) ( ) ( 2 2 1 b ) If n-+oo lim Zn - -8 0' + , prove that the sequences Xn n 2:b Yn n 2: l are convergent and find their limits. ( ) ( ) 1 1 . The sequences x n n 2:l and Yn n 2: l are defined by Xl 2, Yl = 1 and Xn+l x� + 1, Yn+l /= XnYn for all n ;::: 1. a) Prove that Xn Yn < V7 for ( )all n ;::: 1. / b ) Prove that the sequence Zn n ->l, Zn xn Yn , is convergent and lim Zn < V7. n-+oo ) ( ;::: 0 and a f= 0 be real numbers and let Xn n2:l be an increasing 1 2 . Let sequence of real numbers such that C¥ ( ) a. lim n x + x l n n n-+oo 10. Let
a,
=
=
=
a
=
Prove that the sequence is bounded if and only if a > 1 . 1 3. Evaluate
14. Evaluate
1n-+oom kl:n k((nk -+ k)!l) ! (+n (-k k)!+ 1) =O 1.
Ln ( -k2 ) � +l n-+oo k= l n lim
183
5.1. PROBLEMS
L -1 ' q > 1; ii (4 1l) +l ' q > 1 ( ) � n + qn . n=l nq( n ) 16. Let Xn n � l be an increasing sequence of positive integers such that X n+2 2 Xn > Xn+l for all n � 1. Prove that the number L... 101Xn n=l 15. Evaluate
(i)
00
00
+
00
(J - "'
is irrational.
An L (k!1) k=l n
17. Prove that
is irrational for all 18. Let
k,
s
=
n � 1.
00
be positive integers and let numbers such that
aI, a2 , . . . , ak , bl , b2 , . . . , bs be positive real
n � 2. k 21)) al a . . . ak bl b bs. 2 2 ) ( 19. Let X n n � l be a sequence with X l 1 and let X be a real number such that
for infinitely many integers Prove that =
s;
=
. • .
=
Prove that
20. Let
such that
IT (1 - �) Xn+l n=l
= e -x •
A f:. ±1 be a real number. Find all functions f : R R and ( f ln x + A ln y ) 9 (JX) + 9 (..fij) for all x, y E (0, 00). �
9
:
(0, 00)
�
R
=
21. Let f be a continuous real-valued function on the interval
[a, b] and let ml , m2 O be real numbers such that m l m 2 > . Prove that the equation m f (x) � a b-) x b -2x has at least a solution in the interval (a, . =
+
184
5. MATHEMATICAL ANALYSIS
b (0 , 1 /2) , and let 9 be a continuous 22. Let a and be real numbers in the interval ( ( )) ( ) b () real-valued function such that g g x = ag x + x for all real x. Prove that g x ex for some constant e. f [0,00) such that 23. Find all continuous functions f2 (X + y) - f2 (X - y) = 4f(x) f (y) for all real numbers x, y. ( -00, 0] and g : f Prove that if the continuous functions 24. (i) [0 , 00) have a fixed point, then f + 9 has a fixed point. [0 , 1] and 'ljJ [1 ,00) have (ii) Prove that if'ljJthe continuous functions rp a fixed point, then rp has a fixed point. 25. Let rp be a differentiable function at the origin and satisfying rp(O) = 0 . Evaluate () ;� � [rp x + rp (�) + ' " + rp (;)] , where n is a positive integer. =
:
�
�
:
:
:
�
�
�
�
�
�
�
:
�
�
�
�
26. Let a be a positive real number. Prove that there is a unique positive real number f-£ such that p? > for all >
f [a, b') 27. Let' b f( ) and f (a) = j (b) . :
�
�
XIL - aIL-
x
x 0.
be a twice differentiable function on
[a, b) such that f(a)
=
,\ the equation f" (x) - ,\(f' (X)) 2 = b) has at least a solution in the interval (a, . f [0, 2] (0 , 1] that are differentiable at the origin and 28. Find all functions satisfies f(2x) = 2f2 (X) - 1 , x E [0, 1] Prove that for any real number
°
:
�
29. Let ,\ be a positive integer. Prove that there is a unique positive real number
f) such that
x 0.
for all real number >
f
: � � � be a function continuous at the origin and let distinct positive real numbers.
30. Let
'\, f-£
be two
185
5. 1. PROBLEMS
Prove that the limit
f( >..x) - f(J-tx) 1�O -"----'-- X --'--"a: f exists and is finite if and only if is differentiable at the origin. ( ) 31 . The sequence Xn n � 1 is defined by 1. m
n�oo(0 nX] n = - 2. Vnxn . ) O 1 32. Let Xo E , and Xn+ l Xn - arcsin(sin 3 x n , n � . Evaluate lim n�oo f JR be a twice differentiable function with the second derivative 33. Let : JR nonnegati ve. Prove that f(x + I' (x)) � f(x) , x E b 34. Let a < be positive real numbers. Prove that the equation Y a( ; T+ aX bY ( b) has at least a solution in the interval a, . ) 35 (. Find with proof if there are differentiable functions cp : JR JR such that cp( x ) and cp' x are integers only if x is integer. f [ b] JR be a differentiable function. 36. Let : a, O O Prove that for any positive integer n there are numbers < 2 < . . . < n in the b ) ( interval a, such that f(b) - f(a) f' (OI ) + f' (02 ) + . . . + f' (On ) b-a n f 37. Let , 9 : JR JR be differentiable functions with continuous derivatives such that f ( x) + 9 ( x ) l ' ( x ) - g' ( x ) for all x E JR. f(x) - g (x) Prove that if Xl, Xf( real solutions of the equation 2 are) two( consecutive ) ( ) 0, then the equation x + g x = 0 has at least a solution in the interval X l , X2 . f' f 38. Let : [-�, �] (-1, 1) be a differentiable function whose derivative is continuous and nonnegative. Prove that there exists Xo in [-�, �] such that (f(XO ))2 + (1' (XO ))2 � 1. Prove that lim
=
�
R
=
�
�
fh
�
=
=
�
186
5.
MATHEMATICAL ANALYSIS
39. Prove that there are no positive real numbers
x and y such that Y 2 X x2 y - = x + y . (n+1) ( n 40. a) Prove that if x � y � n : 1 ) for some integer n � 2, then yX n+¢Y � ifY + n +V'x. b) Prove that 2n + 1 , n > 1 > n n +l� _ n+ 4 1 . Let Xl , X2 , . . . , Xn be positive real numbers such that Xl + X2 + . . . + Xn +
+
+ n'n V 'H
3.
Prove that
42. Let
f
:
=
1.
�
f(c) = 0 for someJR c E JRJR. be a function with a noninjective antiderivative. Prove that h f JR be continuous functions. Prove that 43. Let h , , . . . , n JR ( ) h( ) f ( ))d max( h x , x , . . . , n x x �
:
is a derivative and evaluate
44. Evaluate
p'
45 . Let be a polynomial of odd degree such that has no multiple zero and let : JR � JR be a function such that is a derivative.
f
p
f op f Prove that is a derivative. 1 f I I be a function with an antiderivative F that 46. Let = (0, 00) and let satisfies the condition F (x)f G) = x, 1 -+ 1R, g (x) = F (x)F (�) is a constant function and I for all x in . Prove that f then find . :
g
:
�
187
5. 1. PROBLEMS 47. Let n >
1 be an integer and let
that
f
:
[0, 1]
�
JR be a continuous function such
f(x) dx = 1 + -1 + . . . + -1 . 1 n 2 1 a
Prove that there is a real number
Xo E (0, 1), such that f(xo ) 1 - xg 1 - Xo f [a, b] 48. Consider the continuous functions , =
:
9
Prove that the equation
�
R
!(x) /." g(t)dt = g (x) f !(t) dt ( b) has at least a solution in the interval a, . [ b] JR be a continuous function such that 49. Let f a, { !(X) dx of O. b Prove that there are numbers a < a < f3 < such that { !(x) dx (b - a)!(p) . [ b] [c, dJ be a bijective 50. Let a, c be nonnegative real numbers and let f a, increasing function. ( b) Prove that there is a unique real number E a, such that { !(t)dt = (IJ - a)e + (b - lJ) d. 5 1 . Let
:
=
:
�
f-£
:
�
:
�
2
53. Let f : JR
�
5
2
JR be an injective and differentiable function.
5. MATHEMATICAL ANALYSIS
188
F (0 , ) lR, Prove that the function F (x) = -l a: f(t)dt x 1a :
00
�
is monotone.
54. Prove that
55. Prove that there are no Riemann integrable functions that for all real numbers
f [0 , 1] 56. Let :
that
J( Y f(t) dt = f(f(xy)) , a:
x =J y. �
fa [0, 1] :
lR � lR \ {O} such
' ( )] 2 d 1 [i l' x x = .
f [0, 1] lR such that J{Ia f (x)(x - f(x))dx = 112 ·
57. Find all continuous functions
58. Let defined by
:
lR be a differentiable function with continuous derivative such
I f ( l) - f(O)1 < 1 .
Prove that
f
�
:
�
lR be a continuous function and let the sequence
(fn) n�l be
fn (x) = { fn- l (t) dt, x E [0, 1] . Prove that if there is an integer m 'l fm (t)dt =such(m :that1)! ' f then the function a has a fixed point. [ 1 1] lR be a differentiable function with nondecreasing derivative. 5.9 . Let f : - , Prove that 1 ( I f( ) d f( I) f' (l ) 2J -1 x x � - + . f, g [a, b] lR be continuous functions. Prove that there is a real number 60. Let b ( ) C E a, such that [ f(x) dx + (e - a)g (e) = l g(x) dx + (b - e)f(e) . 2:: °
�
:
�
SOLUTIONS 1 . We prove that there is
an
> 1 such that n fJn - an > 1 .
integer
e fJ - a. Then fin - an = (a + e)n - an (�) an-I e + . . . + en > nan-I e > ne, 1 � fJn n because a > . Take an integer nfJ> . Then - a > 1. e The interval (an ,fJ n ) has length greater fJ than 1, hence there is an integer > 1 such that an < < n , or a < rm < , as desired. Let =
=
m
m
(Dorin Andriea, Revista Matematidi Timi§oara (RMT) , No. 1(1982) , pp. 90,
Problem 4955)
2. Note that
( 1 + V3) 2 (n+1 ) +1 ( 1 + V3) 2n+3 ( 1 + V3) 2n+1 ( 1 + va) 2 = 4an + 6bn + (2an + 4bn ) V3. b V3 4 = ( an + n ) ( + 2 V3) On the other hand, ( 1 + va) 2 (n+1) +1 an+1 + bn+1 V3, b and since an , n4 are integers we bderive that b 6 (i) an+1 an .+ n i (ii) bn+1 an2+1an-+44bann. . rel atlOn · (11.. ) Imp Ies · n · m 6 . SubStI·tutmg Fr om re1 at"lOn ( I) we btam 4 4 an+2 -6 an+1 2an + 4 an+1 6- an ' or 8 4 bna+nl+2 4bn an+1 - an . On the other hand, an = relation gives bn+2 - 4bn;+1 4, and bn+1the- 4bfirst 6b n + n· 2 2 189 =
=
=
=
=
=
0
.
=
=
=
=
r
190
5.
MATHEMATICAL ANALYSIS
Thus
bn+2 = 8bn+l - 4bn· ( ) (b ) It follows that the sequences an n 2:l, n n2:l are given by
for all n 2:: 1.
(Dorin Andrica, Revista Matematica Timi§oara (RMT) , No. 2(1981) ,
Problem 4648)
pp .
71,
Xn+l and taking into account condition 1) J2 (x; + 1) +1+ X n n = 0, 1, 2, . . . (1) X!(n+l ) Xn 1R is the function given That is Xn+l = xn , n = 0, 1, 2, . . . , where ! : (1, 00) 3. Solving the quadratic equation in
yields
=
by
'
l
�
It is not difficult to check that ! is decreasing and /(2 + V3) 2 + V3. We distinguish three cases: Case 1 . If = 2 + V3, then = 2 + V3 for all n. Case 2 . If E (1, 2 + V3), then from the monotonicity of function ! it follows that < . . < 2 + V3 < . . . < < < < <
Xn Xo Xo X5 X3 Xl · Xo X2 X4 Case 3. If Xo E (2 + V3, 00), then X4 X2 Xo. Xl X3 X5( ) 2 + V3 In all cases the sequence xn n>O lim Xn 2 + V3. is convergent and n�oo ( Titu Andreescu and Dorin Andrica, Romanian Mathematical Regional Contest .
<
<
< . . . <
< ... <
<
<
=
" Grigore Moisil" , 2003) 4.
Note that
Xn+2 = 1 + JXn+l (2 - Xn+l ) = =
1+
2:: 1 for all n 2:: 2. hence We study three cases.
Xn
l
J - 2 x n + x;
=
1+
I Xn
-
11,
5.2.
i) If
Xl < 1, then
Xl X n = { X2
191
SOLUTIONS
1 if if is even 1 if is odd and So the sequence converges if and only if = The equation
n= n X3 n
X2 X3. X2 = 1 + .j2X2 �
n>
- x
has only the solution 1, then ii) If
Xl =
X2 = 2 +2V2 . In all other cases the sequence is divergent. odd Xn = { 21 ifif nn isis even
and the sequence is divergent. 1, then iii) If
Xl >
if n is odd if is even.
n
2+ V2 . It follows that the sequence is convergent if and only if = 2 Le. 2 ( Titu Andreescu, Revista Matematidi Timi§oara (RMT) , No. 1-2(1979), pp. 56, Problem 3865)
X l X Xl =
5. We prove a more general statement: If p is a positive integer and
sequence such that
.
x
� P + x � P + . . . + X2 p
n = 0, n-+oo n then .n-+oo Xl + X2 + . . . + Xn = 0. 11m n For this, recall the inequality Xl + X2 + . . . + Xn ) 2p X2lp + X22p + . . . + X2np n n ( It follows that Xl + X2 : + Xn S " � + � : . . . + X;':' 11m
<
-
I
.
•
.
I
x
P
x
(Xn ) n�l is a (1) (2)
•
P
Using the squeeze theorem and the hypothesis (1), the conclusion (2) follows. For
p
= 1 we obtain the initial problem.
The converse is not true. Take
Xn = ( - l ) n and observe that if n is even if n is odd
5.
192
Hence
MATHEMATICAL ANALYSIS
2 · X l + X + . . . + Xn = 0 11m n�oo n I + X� + . . . + X2n X = 1. lim n�oo
but
n
(Dorin Andrica, Revista Matematidl Timi§oara (RMT) , No. 2 ( 1977 , pp . 47,
Problem 2570)
M
6. Assume by way of contradiction that there is > 0 such that for all n . Summing up these inequalities from 1 to n yields
)
an+ l - an
or (1) b From an 2:: n n it follows that a--;n+l 2:: -n +-1 bn+1 n b ) ( Since the sequence n n >l is not bounded from above we obtain that the sequence ( ann+ 1 ) n >l is not bou:ded from above, which contradicts (1). (Dorin Andrica, Revista Matematica Timi§oara (RMT) , No. 1(1978), pp. 91,
Problem 3441)
) + 1 Xl + a) 2 a . ' < a. The 1eft mequa 7 . N ote that 0 < X2 = and 1"1ty IS 0bV10US X + (a + 1 l the right inequality is equivalent to Xl < a . Since 0 < X2 < a we obtain - likewise o < X3 < a and then Xn E (0, a ) by inducting on n . On the other hand, l) ( a + Xn - l + a 2 Xn - Xn- l - Xn- l + (a + 1) - Xn- l - Xna-2l +- X(a�_+l 1 ) > 0, therefore the sequence is increasing and bounded. It follows that the sequence is 1 = convergent and let nlim�oo Xn . Then l) l + a2 (a + = 1 + (a + 1) 1 so = a. (
_
_
1
(Dorin Andrica, Revista Matematica Timi§oara
Problem 2567)
8. We prove that
(RMT) , No. 2(1976), pp. 53,
an = n In n, n � 1, satisfies the conditions.
First, so
5.2. SOLUTIONS
193
an+! - an = (n + 1) In (n + 1) - n ln n = In (n + 1) + In (1 + � r
Second,
v'an+l - va;; = J(n + 1) In(n + 1) - yIn In n = (n + 1) In(n + 1) - n ln n -r���7===���== = J(n + 1) In (n + 1) + v'n In n l n
( ) In ( 1 + � r
In 1 + In(n + 1) n + J(n + 1) In(n + 1) + v'n ln n J(n + 1) In(n + 1) + v'n ln n In(n + 1) n+1 Because
1 + n ln n J(n + 1) ln(n + 1) + v'n ln n ' / 1+ , V ( n + 1) In(n + 1)
n ( ) n-too In ( l + �r lim In 1 + .!. n
we have
= 1,
. 11m JnTil'n = O . n� oo v(n + 1) ln(n + 1) + n ln n n ln n . In(n + 1) = 0 and 11m . Because 11m = 1, I. t follows that n+1 (n + 1) In (n + 1) lim = 0,
n�oo
as
n�oo
desired.
(v'n�oo an+l va;;)
Remark. Another such sequence is given by = n..jii, n � 1. (Dorin Andrica, Revista Matematica Timi§oara (RMT) , No. 2(1977), pp. 70,
an
Problem 3087)
9. Assume by way of contradiction that there is no sequence (nkh� l with the desired property. Then there is a 0 such that 1-a 0 n hence � an, for all n � 1. It follows that (n - l)n � a(l + 2 + . . . + (n - 1)) = a 2 '
> xn+ xn > >
Xn+l - xn
Xn - X l
1 94
5 . MATHEMATICAL ANALYSIS n - 1 Xl a X - 0' --n2 > + -2 >
then
3 2n n n for all n � 1. This is in contradiction with · 2 = 0, 1 1m n and we are done. (Dorin Andrica, Romanian Mathematical Olympiad - final round, 1984)
X n-too n
x� + O'Yn ::; Zn and y� + fJxn ::; Zn we obtain 2 0<2 2 0 ::; (x n + �) + (Yn + % r ::; 2 (zn + � ,1 ) , for all n ;::: 1 .
10. a) Summing the inequalities
The conclusion follows immediately. b) Notice that
XI n + fJ"2 I ::; ...j2 (Zn + 0'2 +8 fJ2 )
�
From the squeeze theorem follows that
nlim-too Xn = - �2
YI n + "2a I ::; ...j2 (Zn + 0'2 +8 fJ2 )
and
and
�
a n1-1.mtoo Yn = - -2 .
(Dorin Andrica, Romanian Mathematical Regional Contest " Grigore Moisil" , 1997)
Yn = Xl X2 Xn-l and 1 1 Zn = Xl + -X1l + -Xl X2 + . . . + ---XlX2 " , Xn- l for all n � 1. Fr Since Xn+ l - Xn = x� - Xn + 1 > 0, the sequence (Xn ) n�l is increasing. om X2 = 5 we obtain 1 1 . By induction on n we obtain
hence It
1
follows that
•
1 < 2·5 -
-kl' Xl X2 . . . Xk
•
•
k E N*.
r-I G 1 1 1 Zn < 2 + -21 + + ... + 5n - 2 = 2 + . 1 2·5 2. 1-5 5 [ 1 n- l ] 5 21 =2+ 1 - ( 5) < 2 + = - < V7 1-
2
8
as
desired.
-8
8
'
5.2.
195
SOLUTIONS
On the other hand,
Xn+1 Zn+l = -YnX+l = Xn+lYn = Xn+1Yn -Zn Yn XnYn+l XnXnYn Xn�+l = X� + 1 = 1 + � > 1 ' x� ( ) x x� so (zn ) n2:1 is an increasing and bounded sequence. Then Zn n 2: 1 is convergent and 21 ..[7, < lim Zn :::; n-too 8 �
as
claimed.
(Dorin Andrica and $erban Buzeteanu, Romanian Mathematical Regional Contest " Grigore MoisH" , 1992)
c > 0 such that a - c > C¥ From ) nlim-too n (xn+l - xn = a it follows that there is integer n l such that 1 (1) �n1 (a - c) < Xn+l - Xn < (a + c) � n for all n � nl. Summing up inequality (1) from n = n l to n = n l + - 1, > 0 implies (a - c) p-l L ( 1 k) C¥ < xnl +p - xnl « a + c) p-Ll ( 1 k) a (2) n+ n+ k=O l k=O l L 1 k) converges if and only if a > 1, therefore applying the The series ( n + c¥ k=O l 12. Let
o.
an
p
p
00
squeeze theorem to the inequality (2) leads to the conclusion. (Dorin Andrica, Gazeta Matematidi ( GM-B ) , No. 1 1 (1979) , pp. 422, Problem 18011) 13. Note that
Ln k ((nk kl))!! (+ (k k+) !1) = Ln (k k I) ! + Ln k! ( 1 k) ! = + n+ k=O n k=O k=O n (k k I) ! I! Ln k! ( n! k) ! Ln ( k!1 (k 1 I ) ! ) 1! Ln (nk) L = k=O + + n k=O n - = k=O - + + n k=O = 2n 1 = 1 - (n + I) ! + -n! . Since 2n . ( 1 I) ! 1n-too 1m lim - = 0, n + n-too n. _
,
196
5. MATHEMATICAL ANALYSIS
it follows that
k (n - k)! + (k + 1 ) (k + 1)!(n - k)! = 1. k= O
� lim 6
n -+ oo
(Dorin Andrica, Revista Matematica Timi§oara (RMT) , No. 2 ( 1975 ) , pp . 52,
Problem 2281 )
14. Note that
lim ", -+ 0 ", > 0
Indeed, lim ", -+ 0 ," > 0
XX = lim
", -+ 0 ", > 0
ex 1n x
XX = 1.
=e
after applying L'Hospital rule. Then
!�O ¥ >O "'
'" = l� '" O = 1, e
>
(- x )
xx + 1
= 1. X O () () Let c > . There is an integer n c > 0 such that for any integer n � n c , we
lim ", -+ 0
--
", > 0
have
� +1
- c < (:2)k < 1 + c, k = 1, 2, . . . , n. 2 n k Summing up from k = 1 to = n and using algebraic manipulations yields Ln (�)2 �+1 n k < 1 + c, n � n (c) , 1 - c < k= 1 L 2 k=1 n or k ) �+1 1 1 ( 1 c ) 1 1 1 c) < ( -2 - -2 (c - n + -n L -n2 < -2 + -2 c + -n + -n , k=l () for any integer n � n c . Therefore n ( k ) �+ 1 1 lim � -n2 = -.2 1
n
n
n-+oo
(Dorin Andrica)
()
15. i We have
1
I -1 x , Ix l < 1. + x + x2 + . . . + x + . . = -n
.
5. 2.
SOLUTIONS
197
Hence
11/q (1 + x + x2 + . . . )dx = 10 1 /q -1dx- x, o so 1 q 1 ' q > l. In = � n q l nq n= I l 1 (ii) For any x < we have 1 + x4 + x 8 + . . . + x4 n + . . . = 1 1 4 . _x Hence dx 11 /q (1 + x4 + x8 + . . . ) dx = 11/q -1 - x4 ' 0 o so 1 1 1 (1 1 ) 1 1 4 ) ( 1 � n + q4n+ l = - q + 4 ln - q2 + "2 arctg q2(. 1977) 70 (Dorin Andrica, Revista Matematica Timi§oara (RMT) ,40No. ,86pp.08) , 309 98 ) ( 1 1 1 1 1 Problem ; Gazeta Matematica (GM-B), No. , pp. , Problem 00
_
00
16. The number () has the decimal representation
() = 0. 0 . . . 0 1 0 . . . 0 1 0 . . . 0 1 . . . 1 0 . . . 0 1 kl k2 '-.;--'
'-.;--'
'-.;--'
kn
where kl ' k2 , . . . , kn are the number of zeros between two consecutive ones. Because we have
kl < k2 < k3 < . . . < kn < . . . ,
hence () does not have a periodical decimal representation. It follows that () is irrational, as claimed. (Dorin Andrica, Revista Matematica Timi§oara (RMT) , No. Problem
4661 )
17. Note that
and that
An = - 1 + (k!)1 < - 1 + 1 = - 1 + < 2. � � n k! k O k = O = A 2 A 1 1 Hence < n < for all n � . So n is not an integer. 00
00
e
2(1981 ) , pp. 73,
5. MATHEMATICAL ANALYSIS
198
Assume by way of contradiction that there are positive integers n,p, with "# such that P. Then
q q 1 A q = n. 1( 2) 1 ( ) 1 n 1 pq q - = A + (q : l )n + (q + 1 )n q + n + . . . < 1 < A + (q +1 l) n + (q + l) n (q + l) n + . . . = 1 1 1 = A + (q + l )n (1 + (q + l) n + (q + 1 )2n + . . . ) = A + (q + l1)n - 1 , O for some integer A � . It follows that B = ( 1 l ) + ( l ) 1( 2) + . < 1 , q+ n q+ n q+ n . . B A so + is notA integer, which is false. 1 Therefore n is irrational for all n � . (Dorin Andrica) 1 ) Using the limit 18. 1 lim z:ra = n-+oo and taking the limits in both sides of the equality, we obtain 1+ 1+··· +1 = 1+1+···+1 so k 2)= Using the limit 1) = In a lim n ( z:ra n-+oo and the relation n ( \Ial - 1 ) + n ( y'(i2 - 1 ) + . . . + n ( ifiik - 1 ) = = n ( \Ilh - 1) +n ( \Ib; - l) + . . . + n ( v'b,; - l) 8.
� k times
�' s tim es
we obt�in after taking limits:
This implies as
desired .
) Andrica, Romanian Mathematical Regional Contest " Grigore Moisil" , 1999(Dorin
5. 2. SOLUTIONS
19.
Let
199
Pn = IT (1 - �) . Then k=1 Xk+l
hence
I) X Xn+2 xn+l ' - (n + Xn+l n+l Pn+11 P1n (nXn+2 + 1)1 --;;,y(n + 1)1 ( n + 1)1 It follows that Pn+11 = -P1I + -x2!2 + -x33! + . . + (nxn++lI ) ! x2 + . . . + xn+l ' = 1 + I!x + 2f (n + 1)1 Because 2 + . . . + ( xn+l ) eX , X x lim ( 1 + + n-+oo x . 2 .' n + 1) . P it follows lim n+1 D = e-i ,A ddesired. Andreescu n-+oo i ( Titu that and or n n r ca, Revista Matematica Timi§oara (RMT), No. _
_
-
.
--
=
I'
1
=
as
1(1977) , p p . 49, Problem 2843) 20.
Interchanging and y we obtain
x f (ln x + >. ln y) = 9 (y'X) + g (..;y) , x, y E (0, 00) so f (In y + >' In x) = 9 (y'X) + 9 (..;y) , x, y > O. Let b a = ln x + >' ln y and ln y + >' ln x. Then X = e:>.:>.2b--1a and y = e :>.2 - 1 , hence f(a) = f(b) = 9 (e2(:t_a1 ) ) + 9 (e 2(:'2-:,b1 ) ) , a, b E f f ) C. Then for X we have It follows that is a constant and let (x ( constant and g x ) �. g (V'X) D( =ori�,n Asond9riisca,aRevista Matematica Timi§oara (RMT) , No. 1-2(1979), 51, Problem 3827) F I lR, 2 1 . Consider the function F ( x ) ( x - a ) ( x - b ) f ( x ) + m l ( x b ) + m2 ( x a ) . =
:>' a - b
R
=
= Y
=
pp.
:
=
-t
-
-
5 . MATHEMATICAL ANALYSIS
20 0
Note that F is continuous and
hence there is c E (a, b) such that F c) = O. It follows that
( - aml- c + bm-2c f(c) --
and the solution is complete.
-
(Dorin Andrica)
g (x) = g (y) implies that g (g (x)) = g(g(y)) and hence x = y from the given equation. That is, 9 is injective. Since 9 is also continuous, 9 is either strictly increasing or strictly decreasing.( Moreover, 9 cannot tend to a finite limit L as x or else we'd have g (g (x)) - ag x) = bx, with the left side bounded and the right side 22. Note that
----+ 00 ,
unbounded. Similarly, 9 cannot tend to a finite limit as x ----+ - 00 . Together with monotonicity, this yields that 9 is also surjective. Pick arbitrary, and define for all n E recursively by + l = g (x ) for n 0, and for n < O. Let rl = 1 + Ja + 4b)j2 and = (a- Ja2 + 4b)j2 = be the roots of - ax - b = 0, so that rl 0 and 1 I rl l I r2 1 . Then there = clr� + c2 r� for all n E exist E lR such that Suppose 9 is strictly increasing. If f:. 0 for some choice of xo, then is dominated by r� for n sufficiently negative. But taking and Xn+2 for n sufficiently negative of the right parity, we get 0 < < but g (x ) g(Xn+2 ), contradiction. = Thus = O . Since = l and l Clrl , we have = l for all x. Analogously, if 9 is strictly decreasing, then = 0 or else is dominated by for n sufficiently positive. But taking and for n sufficiently positive of the right parity, we get o< contradiction. Thus in that case, g (x) = < for but < all x. ( Titu Andreescu, The " William Lowell Putnam" Mathematical Competition, 2001, Problem B-5)
Xo - l ( ) Xn Xn- l g 2 xn x Cl ,C2 Xn
Z
Xn 2 r > > r2 Z > 2 > . C2 Xn Xn Xn+2 ( ) n > C2 Xo C X gx rX � Xn r C2 Xn Xn+2 X n+ 2 Xn g ( Xn + 2 ) g ( x n ) , (
>
n
Xn
r2 X
x = = 0 yields f O) = O. For x = we obtain f2 (2x) = 4f2 (X) and then f(2;C) = 2f(x), since f(x) � O. 23. Setting
We prove that
(
y
y
(
f nx) = nf(x) , n � 1. Assume that f(kx) = k f (x) for all k = 1, 2 , . . . , n. We have
2(
f ( n + l )x) then
- f2 ((n - l )x) = 4f(nx)f(x),
201
5.2. SOLUTIONS hence as
f((n + l) x) = (n + l )f(x) ,
desired. It follows that if p, are positive integers then
q
qf (�) = f (P) = pf(l ) , so f (� ) = � f(l ) f( ) f(l ) for any positive rational r. and r = r Setting x = 0 in the initial condition gives then for all real
y, hence
f(y) = f( -y) , f(r) = Ir l f ( l ) ,
for all rational numbers We prove that = for all real numbers Let be an arbitrary real number and let be a sequence of rational numbers with lim = Because
f(x) r. I xlf( l ) (rn ) n>l -
x. x n--+oo rn x.
f is a continuous function, it follows that f(rn ) = lim I rn l f ( l ) = f ( lim rn) , lim n--+oo n--+oo n--+oo hence f(x) = f( l )lx l · f( l) � 0, therefore the desired functions are f(x) = al x l for some Note that a = a � O.
and
( Titu Andreescu, Revista Matematica Timi§oara ( RMT) , No. 2 ( 1977) , pp. 90,
Problem 3203)
f for and g, respectively. We have a and b be fixed points f(a) � 0, b = g (b) � 0 a b so a � . ( ) f( ) g(x) - x. Consider the function
:
-t
5. MATHEMATICAL ANALYSIS
202
and
cp(b) = I(b) + g (b) - b = I(b) O.( b) ( ) 0 By the Intermediate Value Theorem, there is an Xo E a, such that cp xo = , hence (I + g)(xo ) = Xo, desired. (3 'l/J (ii) Let a and be fixed points of the functions cp and , respectively. We have a = cp (a) 1 , (3 = 'l/J((3)( )'l/J(� 1) so a ::; (3. Consider the function w lR lR, w ( x ) = cp x x -'l/Jx . The function w is continuous because functions cp and are continuous. Moreover, a)'l/J(((3)a) - (3a = (3(a('l/J(((3)a) - 11)) � 0, w (((3)a) = cp(((3)'l/J w = cp [ (3] - = cp - 0 ::; O. ( 'l/J) ( ) Likewise, there is an ,0 E a, such that w (,o ) = , hence cp ,o = ,0, desired. ( Titu Andreescu, "1Asupra unor5 functii cu punct Revista Matematica 0 77 ) ) ( 9 1 1 , pp. Timi§oara (RMT), No. ::;
as
o ::;
:
::;
-+
as
fix" ,
25. We have
� (cp(x) + cp (�) +0 . . . + cp (�)) = () 0 ) ( 1
_
:
I-"
Hence
-+
I-"
for all x
> 0,
5.2. SOLUTIONS
as
desired. (Dorin Andrica, Revista Matematica Timi§oara (RMT) , No. 2(1978), Problem 3 547) 27. Consider the function
F : [a, b]
-t
203 pp.
54,
lR,
E F(x) = I (x) e- >'f(:r;) , ' F F ( a) = F ( b ) . I I Function is differentiable, since and are differentiable, and ' F b () ) By Rolle's theorem it follows that there is c E (a, such that c = O. On the other hand, F' (x) = e->.f(:r;) (I" (x) )..(1' (X))2 ) , I )..(I' (C))2 = 0, desired. hence (c) ).
-
as
"
R
(Dorin Andrica, Revista Matematica Timi§oara (RMT), No . 2(1981) ,
Problem 4677)
28. Let g : [0, 2]
-t
[0, �), g(x) = arccos l(x) . Then I(x) = cos g(x)
for all x E [0, 2] and the condition is equivalent to cos g(2x) = cos 2g(x) . It follows that g(2x) = 2g(x) + k(x)7r
Z.
for all x E [0, 1], where k : [0, 1] --t 7r On the other hand, 0 � g(x) < "2 for all x E [0, 2], hence k(x) = 0 and g(2x) = 2g(x),
x E [0, 1]
By induction on n we obtain Because 1(0) = 1, g(O) = arccos 1 = 0, so g(x) 9 = x
(�)x 2n
-
g(O)
for all x E (0, 2] . Since is differentiable at the origin 9 is differentiable at the origin and
I
for all x E (0, 2] .
(;)x - g(O) = g (0) X n-+oo 2n
g(x) . g - = 11m
,
_
pp .
76,
204
5 . MATHEMATICAL ANALYSIS It follows that g(x) = fJ;x, where fJ; = g'(O) E [O, �), because 0 � g (x) < i for
all x E [0, 2] . Therefore the desired functions are
IJ1. (x) = cOSfJ;X ,
I-"
E
[O , �) .
( Titu Andreescu, Revista Matematica Timi§oara (RMT) , No. 1(1977) , pp. 45,
Problem 2852)
29. Consider the function
Then
I
:
(0, 00)
-t
lR, I(x) =
ln x x
�.
II (x) = 1 - A+In x ' X>. l t ) and I' (X = 0 for x = e .l >' It follows that () = e l is the only maximum point of the function, so I(x) :::; I(e t ) , X > O. Hence
only for () =
et .
(Dorin Andrica, Gazeta Matematica (GM-B) , No. 3(1976), pp. 104, Problem 15768; Revista Matematica Timi§oara (RMT), No. 1-2(1979), pp. 57, Problem 3870)
> fJ; and let y = XfJ;. Then fl ) f( ) f Y - y G) flAx) - !-'x z-thm. O X = yhm.-tO y = A, so I(ay) - I(y) limO -t y = � = a, y A where a = - > 1. Il Let c > O. Then there is a 8 0 such that for any y < 8, we have I (ay) - I (y) < a-c . Y < a + c. Substituting y with Yk ' k = 1, 2, . . . , n, in the relation (1) we obtain a I(y) - I ( J!.. ) ) 1( -a a - c < y a < -a1 (a + c) , I ( J!..) - I ( JL2 ) 1 1 a2 (a - ) < a y a < a2 (a + c) , 30. a) Let A
_
I-"
fJ;
I-"
>
_
'-' c
-
(1)
2 05
5.2. SOLUTIONS 1( ( 1 ( a!- 1 ) 1 ( a )
-
1
Y
-
-
Y
a - I
�
_
Y
Y n
I
�
Y
-
a - I
-
Y
I -
=
- 1-£
=
=
=
(Dorin Andrica, Revista Matematid1 Timi§oara (RMT) , No. 2(1978),
Problem 3708)
pp .
76,
1= 1 x > x X n+ 1 - X n Xn > 0 ( Xn ) n � l n 0 n X n (X n ) n � l l l 1 n-+oo Xn � 1) z-+o (.!x 1 1 1 n-+oo (Xnn+1+ 1 ) -Xnn n-+oo 1 Cesaro-Stolz ' s Theorem implies 1 nlim-+oo nXn hence lim nXn = 2, as desired. n-+oo
eZ n - 1 Since eZ + for all real 0, we have so is increasing. By induction on we obtain < for all � 1, the sequence so the sequence is bounded. Therefore the sequence converges and let l be its limit. The equation el = + has the unique solution 0, hence lim = O. 1 = - , it follows that Using lim 2 eZ 1 1· 1 eZn ' = l1m 1m =
31 .
=
-
5. MATHEMATICAL ANALYSIS
206
(Dorin Andrica, Revista Matematid1 Timi§oara (RMT) , No. 2(1982), pp. 68, Problem 5004) 32. We first prove by induction that X n O. This is true for n = 0 and assuming Xk 0 for some positive integer k yields 0 sin Xk 1, hence sin3 Xk sin Xk, which implies Xk+l X k - arcsin(sin xk ) = O. It is not difficult to see that (X n ) n>O is convergent and lim X n = O. We have n�oo
>
<
>
>
(vn sin xn ) 2 = and
<
<
n
--=-1-
sin2 Xn
, 2 X sm sm n+1-n n+ l · 2 Xn ' m ---' -::1�----:<1;- = l1m 11 . n�oo SIn2 Xn - SIn2 X n+ l n�oo - 2 2 sin Xn+ l sin Xn . . 2 X sm . . 2 X sm sm sm n+ l 2 Xn n+l 2 Xn ' = l1m = 1.1m n�oo sin(xn - X n+1 ) sin(xn + xn+d n�oo X� sin(xn + xn+ d . X . X 2 sm Xn + Xn+ l . sm · n+1 2 l1m n ' ' 1m = 11m nl-+ oo sin(xn + xn+ d n-+oo Xn n�oo Xn+1 1 1 Xn ' l1m = 1.1m -n-+ oo Xn+1 n�oo 1 + xn+ l 2 Xn From Cesaro-Stolz Theorem we obtain •
(
( )
)
--
lim n-+oo
n 1
lim (vnx n ) 2 = limoo n-+ n-+oo
n 1
hence
sin2 Xn
( )2
� nlim -+oo sin Xn
=
!. 2
Thus lim foxn = J2 . n-+oo 2 Alt�rnative Solution. We have seen above that (xn ) n >O lim Xn = 0 - is convergent, n�oo and Xn 0 for all n = 0, 1 , 2 , . . . We will calculate 1 n+1-n = l (1) 1 1 . nlim n �� -+oo 1 - 1 - X2 x2n+1 (xn - arcsin(sin3 xn )) 2 x� n It is clear that the last limit is obtained from 1 1 lim = lim (arcsin t3 )(2 arcsin t - arcsin t3 ) = t-+o (arcsin t - arcsin t3 ) 2 arcsin 2 t t-+ O ( arcsin t - arcsin t3 ) 2 arcsin2 t
>
------�--
[
_
1
5.2. SOLUTIONS (2 arcsin t - t2 arcsin t3 ) ' t(2 arcsin t - arcsin t3) 1mo . t3 )2 - tl-+1m' o ( arcsint t t2 arcsint3 t3 ) 2 = 2. . t - arCSIn - tl-+ ( arCSIn
_
207
_
_
t
�
t3
It follows that the limit (1) is and the conclusion is obtained via Cesaro-Stoltz Theorem. ( Titu Andreescu, Gazeta Matematid1 (GM-B), No. 10(2002), pp. 409, Problem C : 255 7
)
33. Let x be a zero of 1'. Then J(x + f' (x) ) = J(x) and the conclusion follows. Let x be a real number such that J' (x) < O. Applying the Mean Value Theorem on the interval [x + J' (x) , x] we obtain
J(x) - J(x + J' (x)) = - !, (x)J' (c) ,
for some c E (x + J' (x) , x). Because the second derivative is nonnegative, J' is non decreasing, hence
f' (c) < !,(x) < 0,
and
J(x) - J(x + !'(x)) <
as desired. Let x be a real number such that f' (x)
0,
> O. Likewise,
J(x + ! ' (x) ) - J(x) = !'(x)! ' (c) ,
f ))
f' (c) >
> O.
for some c E (x, x + ' (x and J' (x) Hence J(x + J' (x)) � J(x) for all real numbers x, as claimed. (Dorin Andrica, Revista Matematica Timi§oara (RMT) , No. 1-2(1989), pp. 67, Problem 6 143 ) the Mean Value Theorem for the function J(t) = In t on the interval [a, a34.� bApplying 1 yields
a+b 2 b-a 2
In -- - In a
Hence
In
1 x'
xE
(a' a-+-b ) . 2
( a + b ) :Z; = b - a . 2a
2
(1)
2 08
5. MATHEMATICAL ANALYSIS [ a ; b , b] gives
Using the same argnment for the interval a+b ln b - In 2 b-a 2
1
--
y'
yE
(a+b ) -, b , 2
b-a ) (� Y In 2 . a+b
hence
=
(2)
( From the equalities (1) and 2) we obtain ( a + b ) Z (�)Y =
2a
a+b
Then as
desired.
( )
(Dorin Andrica, Revista Matematica Timi§oara RMT , No. 2(1978), pp. 54, Problem 3548) 35. We prove that no such function exists. Assume the contrary and let k be an integer. From the Mean Value Theorem we obtain =
(
36. Define
Xk
=
k
( )
)
a + - b - a , k = 0, 1, . . . , n and note that n
b-a -:;:;:k = 0, 1, . . . , n . Xk+l - Xk The Mean Value Theorem yields f(Xl ) - f(xo ) b - a f' ((h ) , (h E (xo , xd =
=
'
--
n
5.2. SOLUTIONS
209
Summing up these equalities implies
or
as
f(b) - f(a) t((}i ) , = � t b-a n i=l
desired.
(Dorin Andrica, 3878)
Revista Matematidl Timi§oara (RMT), No.
1-2(1979 ) , pp. 58,
Problem
F : lR lR, F(x) = f(x) - g (x) and note that F is differentiable. Since ' F F (xI ) = F (X2 ) = 0 from Rolle's ( ( ) ) c = O. is c E X l , X2 such that ' ( ) 'there ' ( ) Theorem F ( ) ) ( ( ) f On the other hand, x = f(x )- g (x) = f x + g x , and therefore c + g c = 0, 37. Let
as
--t
desired.
(Dorin Andrica, 3113 )
Revista Matematica Timi§oara (RMT) , No.
2(1977),
pp.
74,
Problem
(f(X)) 2 + (f' (X))2 > 1 for all X in [-i , i]. Then f' ( x) )1 - ( f ( X ) ) 2 > 1 for all X E [-i, i] . Integrating from - � to i yields arcsin f (i) - arcsin f (-i) > On the other hand, it is clear that arcsin f ( �) - arcsin f (-i) ::; i + i = 38.- Assume that
7r .
7r .
contradicting the previous inequality. ( Titu Andreescu, Mathematical Horizons, 2000) y
x f:.= = 0, which is impossible, because x and y are positive. x > 0 such that x . 2Y + . 2-3: = X + and let = X l - X2 , X = X2 - X3 for some X l > X2 > X3 > O. Then 23:1 - 3:2 - 1 X l - X2 1 - 23: 3 - 3:2 X2 - X3 , or 23:1 23:2 23:2 23:3 Xl - X2 (}X2 -( X32 I ) (}2 By the Mean Value Theorem there are l E X ' x and E (X3 7 X2 ) such that (} ( ) 23:1 23:2 ()1 In 2, l E X2 ,XI and 23:2 23:3 = 2()2 In 2, (}2 E (X3 , X2 ) 2 = X2 - X3 Xl - X2
39. If = y , then Assume that there are
x
y
y
y,
y
_
_
---
_
_
---
21 0
5. MATHEMATI(}CAL (}ANALYSIS
291 In 2 = 292 In 2, that implies = a contradiction. (Dorin Andrica, Revista Matematid1 Timi§oara (RMT) , No. 1-2(1980), Problem 4152)
1 2,
Hence
f
f (t) = \Ii - n+\It.(0, 00)
40. a) Consider the differentiable function
:
-+
!, (t)
so, if t
= .!. t n �l '
(
then
(t n(n\l) (t)
_
70 ,
lR,
n_) , -1 n n+1 f' > O. f � n: 1 ) n(n+ l) nn f(y) . nn _1 ) n(n+l ) ,00) � ) ( + l ), > [(_ > > y ( - - n+1 + We have
pp.
_
Hence
and for x
is increasing on
we have f (x)
There-
fore as
desired. b) We prove that
n � 3.
for all The inequality is equivalent to
(1 which is clearly true, because
n�
1. for all Setting x =
+ �r � n,
nn+ l and y = (n + l) n , x � y in the inequality from a) yields
n\fii + n + 1 � 2n + 1, n � 3 n +\Yn+l
as
desired.
(Dorin Andrica, Problem 4668)
Revista Matematid1 Timi§oara (RMT) , No.
41 . Recall Jensen ' s inequality for a concave function
f:
2(1981),
pp.
74,
5.2. SOLUTIONS 211 A f( ) Consider x = ln x, Yi = �, P = Y I Y2 . · . Yn and i = !!.. for i = 1, 2, . . . , n. We Xi Yi
have
so
Hence
.
and smce
Yi = X1i , i = 1, 2, . . . n, it follows that -
1 X � l X� 2
X�n <
--=----=---::-. . .
thus as
desired.
(Dorin Andrica, 3111)
( i=l Xi l n
n
:2:
zt ;
0=1
= n,
Revista Matematid1 Timi§oara (RMT) , No.
2(1977),
pp.
74,
Problem
F
f.
F
42. Let be an antiderivative of the function Since is noninjective, there are real numbers < such that = Applying Rolle's Theorem on the interval we obtain = = 0 for some c E as desired.
F( ) F( ) [X I ,X2] Xl X2 f(c) F' (c)xI X2 .
( Titu Andreescu)
43. Because
(XI ,X2 ) ,
h(x) + 1 /1 (x) ( )) ) h( ( x) + h max(h x , x = 2
-
h (x) 1
5 . MATHEMATICAL ANALYSIS and h , h are continuous, ma:x(h , h) is also continuous. Assume that if f h , 12 , · · . , k - l are continuous, then ma:x(h , . . . , fk - I ) is continuous. It follows that fk ) = ma:x(ma:x(h , · . . , fk- I ) , fk ) ma:x(h , f) and according to the 'first step, the function ma:x(h , . . . , k is continuous. Hence 212
12,
12 , · ·
.
{
ma:x(h , . . . , fn ) is continuous and furthermore a derivative function. Note that if n is even, then
1= : l
ma:x( I, x, . . . , x n ) and
Xn , 1, xn,
=
X E (-oo , - I ) x E [ - 1 , 1] X E (I, oo ) ,
-n
C, X E (-oo, -I) + J max(l , x, . . , xn )dx x n+�l ,1 C X E [-I , I] x + n + , x E (1, 00) . n+1 On the other hand, if n is odd, then xn - l , x E (-00, -1) ma:x(I, x , . , x n ) = X E [- I, I] xn+
.
.
{
.
and
J max(l, x, (Dorin Andrica, Problem 5185) 44. Denote
We have
hence
.
. .
, x n )dx
=
1
1,
xn ,
-n n
x E (1, 00) ,
+ 1 + C, C x+ , xn+ l + n C n+1 + , xn
----
X E (-oo, - I) x E [- 1 , 1] X E (I, oo).
Revista Matematidl Timi§oara (RMT), No. 2(1983) ,
pp.
62,
5. 2 .
21
SOLUTIONS
1 + x 2 d ( e arct gz ) = ! VI + x2 dx = ! xl +1 x+2 exarc2 tg:c dx = ! x V� = x VI + x2earctg:c - I1 - 2 ! xVI2 earc+ tg:cX2 dx' ! x2 earctg:c dx = xJ1+X2earctg:c - (II + 12 )
On the other hand, x earctg:c
12
so
VI + x2
= (x
-
2 tg:c arc 2e x 1)V1 +
2
3
=
=
+ C.
(Dorin Andrica, Romanian Mathematical Olympiad - final round, 1975; Revista Matematid1 Timi§oara (RMT) , No . 2(1978 ), pp. 35, Problem 2125) 45. We start with the following lemma .
lR lR
Let g, h --t be functions such that: 1) h is a derivative; 2) 9 is differentiable with a continuous derivative. Then 9 h is a derivative function. Proof. Let H be an antiderivative of h and define u Lemma.
:
.
Then
u
l
(x) l ' or g (x) h (x) = u (x) - g (x)H( x ).
=
g
:
lR
--t
lR,
u
(x)
H ,
( x ) h (x) + 9' (x) (x)
=
g(x) H(x) .
The function u l is a derivative and gl . is continuous, hence the function 9 . h is a derivative, as claimed. 0 Applying the lemma for h and g (x ) = xk for a nonnegative in f teger k, it follows that x k (f 0 ) ( x ) is a derivative, hence ' ( f 0 is a deriva tive. Since the degree of is odd, ( ) Assuming that lim p(x ) = -00, z--+-oo there are real numbers xi , x� , . . . , xm , x� such that p is increasing on each of the interval (-00, xd, [xi , x2 ], . . . , [X�_ 1 , Xm ] ' [x�, 00) and (x = p(xD M1 , = = M , . . , p(xm ) (x�) = m . Let F1 , H1, . . . , Hm be an anti 2 derivative of ( f 0 on the interval (-00, . [x�, 00), respectively. It follows that F1 0 0 . , Hm 0 are antiderivative of f on the intervals (-00, [Mm ' 00) , respectively, hence f is a derivative function on all
H
= op p lR lR p p) p p = . Xl , d p P(X2 ) p(x�) ) = p dM[ � ] p -pI H - I x- , x , X2 , " , I p MI ] , [MI , Mp2] , , ,I p , .
.
=
.
• • •
R
(Dorin Andrica, 2)
Revista Matematica Timi§oara (RMT) , No. 2(1985), pp. 76,
Problem
46. Substituting
x --t 1
-
x
in the given condition yields
F
G ) f(x) � =
,
(1)
21 4
5 . MATHEMATICAL ANALYSIS
x I. g (x) = F'(x)F G ) + F (x)F' G ) ( - :2 ) = f (x)F (D - :,1 G ) F (x) � = -X1 - -x12 · x = O F (x ) F ( � ) = x I, g >0 x I, (1 ) f (x) 1 F(x) ex ' x I. InF(x) = �e x + d, d > O. F(x) = dx�, x I. = d = ,;c. F(x)F (;�) = 1 1. - 1 x I, Xc , f(x) = -XF---;1 -�-;--) = .;c (
for all
in
for all
in
for all
in
We have
so is constant. Then there is a constant c
such that
c
we obtain
and from
E
Integrating gives
for all
in
ln
ln
where
It follows that c, so
c becomes cP
The relation
Finally,
E
where
e is any positive real constant.
( Titu Andreeseu, Romanian Mathematical Olympiad - final round, 1987; Revista Matematid1 Timi§oara (RMT ) , No. 2(1987), pp. 86, Problem 6307)
lR, g(x) = f (x) + x + . . . + xn - 1 )
47. Consider the function
and note that
9
9
:
[0, 1] -+ - (1
is continuous. We have
1
Jro 1 g (x)dx = Jor 1 g(x)dx - Jro (1 + x + . . . + xn-1 )dx = = l' f (x)dx - (l + � + " ' + D = 0. From the Mean Value Theorem there is Xo E (0, 1) such that g(xo) = l' g(x)dx = 0, hence 1 - x� , f (xo ) = l + xo + . . . + xno - l = -1 Xo
as
-
desired.
( Titu Andreeseu, Revista Matematidl Timi§oara (RMT) , 3444)
Problem
No.
1-2 ( 1979),
pp.
33,
5. 2 .
SOLUTIONS
215
F : [a, b] lR, F (x) = [ g (t)dt t f(t)dt. F is differentiable' and F(a) = F (b) = O. Applying Rolle's Theorem, Note that b) F ) we obtain c E (a, such that ( c = 0 hence ftc) [ g (t)dt = g (c) t f(t)dt, 48. Consider the function
--t
as desired.
(Dorin Andrica)
F : [a, b] lR, F (t) = l f(x)dx - t f(x)dx F is continuous and F (a)F (b) < O . Then there is a E (a, b) such and observe that F( ) that a = 0 so lQ f(x)dx = l f(x)dx ( b) From the Mean Value Theorem there is f3 E a, such that l f(x)dx = (b - a)f({3) , ( b) therefore there are numbers a, f3 E a, , a < f3, with { f(x)dx = (b - a)f({3) , 49. Consider the function
--t
as claimed.
(Dorin Andrica,
Revista Matematica Timi§oara (RMT), No. 1-2(1979) ,
Problem 3897) 50. Since Denote
pp.
61,
f is an increasing bijective function, f is continuous. ' ( )d r )d f( y y, 8, = I.' x x, 82 = f.d
and note from the diagram below that
(1)
5. MATHEMATICAL ANALYSIS
216 y
d . . . .. . . . . . . . . . . . . . . . . . . . . . . ---7J .
1-.1! 1
C
��
1-. .-.._"=' -;...., . -T .
•
•
•
•
,/
•
•
� 1. . . . . . .
b
a
o
x
( From the Mean Value Theorem there is e E c, d) s.llch that
{ f - l (y)dy
Observe that e is unique and let
as
desired.
=
(d - c)rl (e).
fJ; = 1- 1 (e) . The relation ( 1)
t f (t) dt = (" - a)e + (b - ,,)d, " E (a, b) ,
(Dorin Andrica, 3556)
gives
Revista Matematidi Timi§oara (RMT) , No.
2(1978),
pp.
56,
Problem
5 1 . Consider the function F : �
--+ �,
[ cp(t)dt, and note that F is differentiable. ) From the hypothesis we obtain F (x + y) - F (x) = F(x) - F (x - y , so F ( x + y) + F (x - y) = 2F(x), x, y E F( s ) =
�.
Differentiating with respect to y it follows that
F'(x y) = F'( x - y), x, y lit z �, we obtain F'(z) = F'( O), so c.p(z) = F' (O) for all z E lit
�, E
+
E
Setting x = y = Hence c.p is a constant function, as desired. ( Titu Andreescu, Revista Matematidi Timi§oara (RMT) , No. Problem 2865)
pp.
47,
/.8 f(t)dt. The condition is equiva /. f(t)dt + /. y f(t)dt :::: 2 /. f(t)dt,
52. Consider the function
lent to
1 ( 1977),
x
cp
:
JR
-4
JR,
cp(s) =
.=.±.ll 2
5.2.
SOLUTIONS
217
cp(x) + cp(y) -> cp ( x + y )
hence
(1)
2
2
for all E llt Since I is differentiable, is twice differentiable and moreover is concave up, from � 0 or � 0 for all so I is a nondecreasing function. relation (1) . Hence ( Titu Andreescu, Revista Matematidi Timi§oara (RMT) , No. 1 ( 1976), pp. 56, Problem 2349; Gazeta Matematidi (GM-B) , No. 2 ( 1980), pp. 68, Problem 18154)
x, y
"cp (x) cp f' (x)
x,
h [0 ,00 ) --+ lR, h(x) xf(x) - 1" f(t)dt. h Because I is differentiable, is differentiable and h(x) xf' (x) , x � O. (1) ) ( Since I is injective and continuous, I is either increasing or decreasing, so f' x () o for all x or I' x � 0 for all real numbers x. 0, x � 0, Case 1. If I ' (x) � 0 for all x, then hfrom (1)h we deduce that h (x ) O h ) ) ( ( 0, x � O. hence is nondecreasing. It follows that x '( h' (x) � 0, x � 0, and h is nonincreasing. It Case 2 . hIf f x )h �O 0 for all x, then ( ) ( ) 0, x � O. follows that x � Since F is differentiable and F' (x) xf(x) - 10(X f (t)dt h(x) x2 - --;2 ' ' we derive that F (x) � 0 for all x > 0 or F'(x) � 0 for all x > 0 , hence F i s a :
53. Consider the function
=
=
�
'
=
�
�
=
=
_
monotonic function, as desired. ( Titu Andreescu, Revista Matematidi Timi§oara (RMT) , No. 2(1978), pp. 76, Problem 3709; Gazeta Matematidi (GM-B) , No. 1 ( 1980), pp. 38, Problem 18115) 54. Recall from Problem
14 that
0 such that for all x < I xx - 11 < c. Then for n > � we obtain I n° l '!. (x'+ ! - X)dX I � nol';; Ix·+1 - xldx n2 l� xlxx - l ldx < cn2 l� xdx c2
and let c > O. There is 0 >
0,
=
=
0
0
= -.
21
5. MATHEMATICAL ANALYSIS
8
It follows that
1
lim n-+oo
hence
n2 {n (XX+ 1 - x)dx 10
lim
as
n-+oo
desired.
Alternative Solution .
n2
(�
10
xx+ 1 dx
Consider the function
= 0,
= -21 ,
F (t )
and we can write
= l' x·+1 dx. Then F(O)
=
0
� xx+ l dx = lim n2 F ( -1 ) = lim F(u) = lim F' (u) = l n-+oo n u-+O u2 u-+O 2u n-+oo 1 . uu+ l = -1 hm = hm -+u O 2u 2 u-+O. uu = -.2 lim
n2
--
0
--
--
( Dorin Andrica, Gazeta Matematidi (GM-B) , No. 1 1 (1979) , pp. 424, Problem 18025; Revista Matematidi Timi§oara (RMT) , No. 1-2 (1980) , pp. 71, Problem 4160)
x =f. y be real numbers. Then x f(x) and f (t) dt f(y)
55. Assume the contrary and let
(Y
Jx
f(t)dt
hence
=
0,
f(y) f( = x) ,
/.Y
f(y) f(x) f(y) - f(x) so f(x) = f(y)
It follows that f 2 (X) + f 2 (y) = 0, which is absurd since f(x) =f. 0 for all x. ( Titu Andreescu, Revista Matematidi Timi§oara (RMT) , No. 2 (1978) , p p . 35, Problem 3126)
=
56. It is clear that
[ (1 /,(x) l - I)'dx O. [ (f'(x))'dx [ 1/, (x) ldx + [ dx [ 1 /,(x) l dx I [ /' X)dx l ;:::
Hence
-2
and
as
1 ;:::
desired.
( Titu Andreescu,
Problem 3130)
;:::
(
;::: 0,
,
Revista Matematidi Timi§oara (RMT) , No. 2 (1977) ,
pp.
77,
5. 2. SOLUTIONS
2 19
57. The relation is equivalent to
Hence or
r 1 (xf(x) f2 (X) )dx = r 1 X2 dx . io io 4 (f2 (X) - xf(x) + x dx = 0,
l
l (f(x) - �r
:)
= O. Because f is continuous, f ( x) - � = 0, for all x E [0, 1], so f (x) = � , x E [0, 1].
( Titu Andreescu, Problem 3319 )
dx
Revista Matematidi Timi§oara (RMT) , No.
58. The relation
1(1978),
pp.
72,
l fm(t)dt = m : 1) 1 ' l (fm(t) - : ) dt = O. (
is equivalent to
Since
fm is continuous by the Mean Value Theorem there is Xo E (0, 1) such that
or
rco
io
(fm-1 (t) - mtm-1_ I)! ) dt = 0. (
Xl E (O, xo) such that m-1 fm-I (xd = (�1_ 1) 1 " Continuing this procedure, we obtain Xm E (0, X m-1 ) such that fo (xm ) = xm ,
Using the same argument, we obtain
as
desired.
( Titu Andreescu, Problem 33 2 0)
Revista Matematidi Timi§oara (RMT) , No.
1 (1978),
pp.
72,
the Mean Value Theorem we deduce that for any X E [-1, 1] there is xC E59.(-1,From x) such that f (x) - f ( - 1) = (x + 1 ) f' (cx ) . Since f' ( cx ) :::; f'(I), f (x) - f( - 1) :::; (x + 1)1' (1), hence
r 1 f (x) dx _ 2f ( - 1) :::; (x + 1 )2 1 1' (1) i-I 2 1 -1
220
5 . MATHEMATICAL ANALYSIS
and the conclusion follows. ( Titu Andreescu, Revista Matematidi Timi§oara (RMT) , No. 2(1981) , Problem 4686)
pp.
77,
h : [a, b] --+ lR, h (t) = (b - t) [ f (x)dx + (t - a) [ g(x)dx h and note that is differentiable and h' lt) = - [ f (x)dx + (b - t)f(t) + [ g (x)dx - (t - a)g (t) . h (a) = h (b)' = 0, from Rolle's Theorem it follows that there is a real number Since b c E (a, ) such that h (c) = O. Then - /." f(x)dx - (e - a)g (e) + (b - e)f (e) + t g(x)dx = 0, 60. Consider the function
and the conclusion follows. ( Titu Andreescu, Revista Matematidi Timi§oara (RMT ) , No. 2 (1985) , Problem 5628)
pp .
56,
C hapter 6 C O MP REHEN SIVE P RO B LEMS
P ROBLEMS 1 . Let A be a set with n elements and let B be a subset of A with m � 1 elements. Find the number of functions f : A --+ A such that f(B) � B.
Y
2. Let A be a set with n elements and let X, be subsets of A with p � 1 and q � 1 elements respectively. Find the number of functions f : A --+ A such that C f(X ) .
Y
3. Let A b e a set with n elements and let X b e a subset of A with k � 1 elements. Find the number of functions f : A --+ A such that f(X) X.
=
= {I, .
4. Consider the sets A 2, . . , n } , B {I, 2, . . . , m } and let k ::::; min{ n, m } . Find the number of functions f : A --+ B having exactly k fixed points.
=
aI, a2 , . . . , an be positive real numbers and let m � 1. Prove that (1 al )m ( a2 )m . . + (1 an ) � n . 2 . + a2 + 1 + a3 + . + al 6. Let a I , a2 , . . . ,a n be positive real numbers and let k � O. Prove that
5. Let
7. Let
m
m
a, b, c be positive real numbers such that abc = 1 . Prove that
8. Let numbers x ,
[a, b] be an interval. Find the y, z [a, b] {3(y - Z)2 + ,(z - X)2 E( x , y, z) = a( x - y ) 2 + a, (3"
be positive real numbers and let E such that
is maximum.
22 3
224
6. COMPREHENSIVE PROBLEMS 9. Find the maximum number of nonzero terms of the sum n i L If ( ) - f (j ) 1 i ,j = l where f : {I, 2, . . . , n } --+ { a , b , c } is one of the 3 n possible functions. b b 10. Let a I , a2 , . . . , an , l ::; 2 ::; . . . ::; bn be positive numbers such that al � bl , al + a2 ::; b1 + b2 , . . . ,al + a2 + . . . + an � bl + b2 + . . . + bn · Prove that
+ Viii. + . . . + ..;a;; ::; ..jb; + .jb; + . . . + V'bn. n i2 1 1 . Define S (n,p) = L (n + 1 - 2 ) p for all positive integers n and p. Prove that i= l for all positive real numbers a i , i = 1, n the following inequality holds ) ( 4P n 2 p rp.i� ai - aj ::; S ( n,p) L 1��
1 2 . Prove that
for all integers
n � k � 2.
13. a) Consider the real numbers and the determinants k 1, 2, . .
Ak, = 1 au
aij , i = 1, 2, . . . , n - 2, j = 1, 2, . . . , n, n � 3,
. ,n 1
1
an-2,k- 1 an -2,k+1 Prove that Al + A3 + A5 + . . . = A2 + A4 + A6 + . . . ' b) Let X l, X2 , ,Xn be distinct real numbers and let n - ( k+ l ) kl Pk = IIi=O (Xn-i - Xk ), qk = iII=-l (Xk - Xi ) , for k = 1, 2, . . . , n. • • •
Prove that
1
PROBLEMS
6.1. c) Prove that
for all integers
n � 3.
n'" ( - l) k k2 k=1 (n - k)!(n k) ! - 0,
P ( x ) = xn
am
+
�
+ a x n -I + 14. Let l real numbers. Prove that if there are m f:- p E
P( ) ( l) then x = x - n .
225
.. n { I , 2 , . . . , n}
= (_l)m (�)
.
+
a be a polynomial with all zeros positive such that
and lip =
(-1)"
(;) ,
Po , PI ' " ' ' Pn by Po (x) = 1 and Pk+ 1 (x) = (n - k x)Pk (x) + xPk(x) P P for all k = 0, 1, . . . , n - 1. Prove that deg k = k for all k and find the polynomial n . (P ) 16. Let n n � o be a sequence of polynomials defined by Pn+dx) = - 2xP�(x) Pn (x) and Po (x) = 1. P (O ) . Find n . 1 5 . Define the polynomials
+
+
17. Consider the polynomial
P (x) = Ln 1 x k . k=2O( n) 1 P ) ( 2 Prove that the equation x = p x has no real roots. P be a polynomial with real coefficients. Find all functions f 18. Let t such that there is a real number such that f(x + t) - f(x) = P (x) for all x E b [ 2 2] 19. Find the real numbers a, , c, d, e E - , such that +
k
+
R
a+b+c+d+e=O
and
:
1R -+ 1R
6. COMPREHENSIVE PROBLEMS
226
20. Let p be an odd prime. The sequence (an)n? O is defined as follows: ao 0, a1 = 1, . . . , ap- = p- 2 and, for all � p- 1, an is the least integer greater than an-l that does not form an arithmetic progression of length p with any of the preceding terms. Prove that, for all is the number obtained by writing n in base p - 1 and reading it in base p.
2
==
n
n, an
2 1 . i) Let c be nonnegative real numbers and let I : increasing function. Prove that
a,
L
[! ( k) ] +
L
[1- 1 ( k ) ] -
[a, b) -+ [c, d] be a bijective
n (GJ ) = [bJ [d] - a (a)a (c)
of points with nonnegative integer coordinates n (GJ ) is the number a 1R Z is defined by Z if x E 1R \ [x] a (x) 0 if x Z0 x - I if x E \ {O}
where k is integer, on the graph of I and
:
-+
=
{
=
ii) Evaluate
22. i) Let c be nonnegative real numbers and let I decreasing function. Prove that
a,
L where k is integer and ii) Prove that
for all integers
n � 1.
[f( k)] -
L
[f - 1 ( k ) ]
==
:
[a, b] -+ [c, d] be a bijective
[b]a (c) - [dJa (a),
a is the function defined in previous problem.
t [ n22 ] = f: [�]
k= l k
k= 1
VIi
23. Let 1 . . . be a sequence of integers such that no two are consecutive. Prove that for all positive integers m between n l n 2 . . . + nm and nl n2 there is a perfect square.
< nl < n2 < . . . < nk <
. . . + n m+ 1
24. Let (Xn)n ? l be a sequence defined by positive integers n.
+ +
+ +
Xl
= 3 and Xn+l = x�
-
2 for all
227 6. 1 . PROBLEMS ( ) ( d Taking into account that xn ' xn+ = 1 for all n 2:: 1, there are sequences Un n 2: 1 , (vn ) n2:1 of positive integers such that Prove that is a perfect square for all
n 2:: 1 .
25. Find the sum of the series
n) �
n=l
26. In a coordinate system xOy consider the points Ak (k, n - I), k 0, 1, . . , n- I for a given positive integer n. Find the number of open segments OAk that do not contain a point with integral coordinates.
=
27. Let
.
(an ) n2:1, (bn ) n2:1' (Cn ) n2:1 be sequences of positive integers defined by � .ij4) b� � (1 + + n = an + n + cn ' n 2:: 1.
Prove that
if if if
n =: O n =: 2 n =: 1
(mod (mod (mod
3) 3) 3)
(an) n2:1 , (bn )n2: 1' (Cn ) n2:1 , (dn ) n2: 1 defined by al = 0, an+ l = 2bn + 3cn , bn+1 = an + 3dn , Cn+l = an + 2dn ,
28. Consider the sequences 1, = 1, = 0 and
b1 = Cl d1 dn+1 = bn + Cn , n 2:: 1.
Find a closed formula for the general term of these sequences.
= 2,
f N* x N* --+ N* be function such that f(I, 1 ) f(m + I, n) = f(m, n) + m and f (m , n + 1 ) f(m , n) - n 2001. for all m, n E N* . Find all pairs (p , q ) such that f(p, 29. Let
:
a
30. Determine all functions f
for all integers
x, y, z.
:
Z --+ Z satisfying
)q = =
6. COMPREHENSIVE PROBLEMS
228
3 1 . 1981 points lie inside a cube of side 9. Prove that there are two points within a distance less than 1.
32. The squares of a chessboard are randomly labeled from 1 to 64. On the first 63 there is a knight. After some moves, the 64 ' s square, initially unoccupied, is also unoccupied. Let n k be the square number of the knight who was initially on the k's square. Prove that
63
=l
L i n k - k l � 1984. k
(Fn ) n2: 1 is given by FI = F2 = 1, Fn+2 = Fn+l + Fn ,
33. The Fibonacci sequence
Prove that for all
n
2::
n
2:: 1.
2.
34. Let n be a positive integer and let Nk be the number of increasing arithmetic progressions with k terms from the set {I, 2, . , n}. Prove that
where q =
[n - 11 ] . k
.. 1 2 ( n 1) + 1 -
Nk � - "2 q
+
+ "2 q
k,
_
FI F2 = Fn+l = Fn + Fn l P (k) = Fk forn= = Fl983 ( ) 3 6 . Let Xl , X2 , f3 b e real numbers and let the sequence xn n 2: 1 b e given by 1:- 0 for all m > 1, prove that there are real numbers AI , A2 If x� - X m+I Xm - 1 Fn
35. Let be the nth Fibonacci number (that is, 1, = for 2:: 2), and let P ( x) be the polynomial of degree 990 such that 1. k 992, 993, . . . , 1982. Prove that P(1983) A. ,
such that
and
for all
n > 2.
6. 1 .
PROBLEMS
22 9
37. Consider b E [0, 1) and the sequence (an )n� l defined by al ak - l = 0, k � 3, and 1 an+l = k ( b + an + an2 - l + . . . + ank-l -k+ 2 ) for all k.
Prove that the sequence is convergent. 38. Let (an )n� 1 and (bn ) n2:1 be sequences such that i) (b n } n > 1 is strictly monotonic and unbounded; an ii) there exists lim ; n�oo bn bn+1 . . . an+l - 2, n > 1 . 111 + ) bn an an = O. Prove that nlim �oo bn 39. Let (U n)n � l ' be a sequence defined by U l E 1R \ {O, I} and
U n+l = Ul
U2
U3
If the sequence converges, evaluate n 1 - II (1 nlim -+oo n k =2
°
+ Ul . . . Uk ) .
40. Let (an)n2: 1 and (bn )n2: 1 be sequences such that i) 0 b 1 b2 bn b 1 � k > 1, n � 1; ii) an iii) there exists lim . n �oo b n an an+ 1 - an . . Prove t hat hm eXIsts and is equa1 to nl'1m - . n�oo bn+l - bn �oo bn
< < < ... < < ...; �:
41. Let k be a positive integer and let
Prove that
42. Let f : 1R
�
1R f (x) =
n
L sin akx,
k =l
=
a2
= ... =
23 0
6. COMPREHENSIVE PROBLEMS
where are real numbers. Prove that if number Xo such that f (xo) -I O.
ak
43. Let that
f
:
l ai l -I l aj l for i -I j , then there is a real
lR lR be a differentiable function with continuous derivative such -t
lRf(x)lR=, lim t (x) = g (x) = sin f (x) ,
lim
x-too
Prove that the function
is not periodical.
9
:
x -too
-t
00 .
44. Let a be a real number and let f : N -t [0, 1 ) , f (n) part of the number an. i) Prove that f is injective if and only if is irrational. ii) If a is rational, find the number of elements of the set
= {an} i.e. the fractional
a
M = {f(n) 1 n E N}.
lR lR M =M
45. Let f : -t be a function such that i) f has a period T > 0; ii) f (x) � for all x; iii) f (x) if and only if x = kT, for some integer k. Prove that, for any irrational f) the function 9 : -+ lR,
lR
g (x) i s not periodical.
= f(x) + f (f)x) ,
lR lR be a continuous function with a period T > O. A a) Prove that if T is irrational, then for any E [min f(x) , max f (X)] there is a 46. Let f
-t
:
sequence (X n ) n � l of integers such that
xER
xER
= A. A b) , Prove that if T is rational, then for any E [min f(X) , max f(x) ] and for any lim f (x n ) n -too
x ER
xER
irrational number f), there is a sequence (X n ) n �l of integers such that
47. i) Let x, y, z, v be distinct positive integers such that that there is no > 1 such that
A
x
+y
=
z + v. Prove
6. 1 . PROBLEMS
23 1
a, b, c, d be distinct positive integers such that aP + bP = cP + at .
ii) Let p be a prime number and let
Prove that
l a - c l + I b - dl �
p
48. i) Prove that
for any x � y O. ii) Prove that
>
1) l) n (n + n+ 1 --en < n. < en , (n +
1
n�
49. Let c be nonnegative real numbers and let f function. i) If f is increasing, prove that
a,
1.
: [a, b) � [c, d), be a bijective
l f(t)dt + ld r' (t)dt = bd - ac.
ii) If f is decreasing, prove that
f.' f (t)dt - ld r' (t)dt = bc - ad. lR be a continuous function such that lim J-L(x) = o. 50. i) Let (0, 00) Prove that t lim e - (Z e J-L(t)dt = 0 lR be an n-time10differentiable function with the n-derivative ii) Let f : [0, 00) continuous such that there exists n � k J-L
�
:
z�oo
z
z-too
�
lim
x-t=
Prove that lim
z�=
=Ok C f ( ) (x) = �
f (x) exists and
lim
x-t=
5 1 . Let
f
:
A.
f(x) = A.
[a, b] [c, d) be a continuous function such that �
Prove that if c + d =f. 0 ::;
' f' (x)dx = cd. f. � a b
0, then
�
c d
l f(x)dx ::; b � a G� �r
232
6. 52. Let
COMPREHENSIVE PROBLEMS
I : [a, b] lR be a continuous monotonic function and let F : [a, b] F (x) = (x - ) l' f(t)dt + (x - b) [ f(t)dt. �
�
lR,
a
Prove that all values of F have the same sign.
53. a) Consider the functions : (0, 00) � lR and 9 : [1, 00 ) 1) 9 is differentiable with continuous derivative; 2) is continuous and the function h [1, 00) � lR, h(x) nonincreasing. Denote
I
I
:
n = kL= l h(x) . Prove that ( 1) an+ 1 - h( l ) ::; !.gg(l )n+ I(x)dx ::; an , b) Prove that n kI kl L lim n-+oo k=l 2 cot - = 00. I [0, 1] 54. Let be a positive real number and let
�
lR such that
= gl(X) - I(g(x)) is
an
:
a
function. Evaluate
( r)
�
lR+ be an integrable
In : lR lR, 2 , X -1- 0, x 1 k E '71 * s�n k7r ' sm X X = o-k7r1 ' k E Z* x= for all integers n � O. I 1) Find the numbers an , k such thatI n is continuous on lR* . 2) Find the number an such that n is a derivative function. 55. Consider the functions
56. Let p, q
�
-II
I
/U
� 0 be integers. Find the numbers Cp, q such that Ip , q : lR � lR, 1 cosq 1 sm - if x =f- 0 f if x 0 •.•
is a deri vative function.
(x) = {
•
p
_
X
.
x'
=
6. 1.
n X) -- { 'an ( X) , X
PROBLEMS
57. Let q be a positive integer. Find the number an (q) such that fn : lR n- l , =f. 0 cos q cos � cos !I
f(
.
. .
q is a derivative function for any integer n > O. 58. 1) Let
x
X
x
-+
lR,
233
n E N* ,
=0
f : lR -+ lR be a continuous function such that l
lim
-
I Y I -too Y
Prove that the function
g(x)
/. f(x)dx = M (f). 0 Y
= { f G) , M(f),
x ;l a x 0
=
is a derivative function. T 2) Let f : lR -+ lR be a continuous function with a period > O. Prove that
M(f)
=
lim
Itl -too
lt /.t f(x)dx = l /.T f(x)dx o o
-
T
59. If f : lR -+ lR is a derivative function, then is derivative function? 60. If h , h : lR derivative function?
-+
9
: lR -+ 1R,
lR are derivative functions then f
g(x)
= If(x) 1 also a
= max{h , h } is also a
SOL UTIONS
h
functions : B ---+ B. Each of them can be extended in n n - m 1 . There are ways to a function f : A ---+ A which satisfies f (B ) � B . Hence the required number is (Dorin Andrica, Revista Matematica Timi§oara (RMT) , No. 1-2 (1981), pp . 81, Problem C : )
mm
mm . n n - m .
ll
2. Let Xq be a subset with q elements of X . Since Y has q elements, it follows that there are q! bijective functions 9 : ---+ Y . Each of them can be extended in n n- q ways to a function f : A ---+ A which satisfies Y � f(X) .
Xq X X The number of subsets q of is (:) , hence the requested number is (:) nn -p. Remark. If q > p, consider (:) = O. (Dorin Andrica,
Romanian Mathematical Regional Contest " Grigore MoisH" ,
2000) 3. Since f (X ) X, it follows that f is bijective on X . There are k! such bijections and each of them can be extended in n n- k ways to a function f : A ---+ A. Hence the desired number is
= k!nn -k .
(Dorin Andrica)
4. We consider two cases. i) ::; m. Let A k be a subset with k elements of the set A. There is only one function : A k ---+ A k that has the property for all E A k . This function can be extended in (m ways to a function f : A ---+ B such that f(i) f:. for all
nh
i E A \ Ak •
h (i ) = i
- l )n- k
The number of the subsets Ak of A is
l ) n- k .
(�),
k
i
i
hence the desired number is
(�) m (
ii) m ::; n. Let Bk be a subset with elements of the set B. There is only one function B k ---+ Bk such that = for all E Bk• This function can be extended in ( m - l ) m - k ways to a function 9 : B ---+ B such that g (i) f:. for all i E B \ Bk •
h:
h (i ) i
235
i
i
6. COMPREHENSIVE PROBLEMS
23 6
mn-m ways to a function f : A
Moreover, each function 9 can be extended in that clearly has exactly fixed points .
k
The number of the subsets Bk of B is
(7) mn -m (m - W -k .
(7)
-4
B,
hence the desired number is
: A B with k fixed points is (�) (m l ) n-k if n :O; m and (7) mn-m (m _ l) m-k if m :O; n. Therefore the number of functions f
(Dorin Andrica,
-t
Romanian Mathematical Regional Contest "Marian 'farina,
2001) 5 . Using the inequality
x r + x� +
.
.
.
+ x��
�
for
� (Xl X2 n -l +
+ ... +
Xn ) m
we obtain
On the other hand,
an al a2 an al a2 a2 + a3 + · · · + -al >- n n a2 a3 . . . -al = n from the AM-GM inequality. Therefore (2n) m (1 aa2l ) m (1 aa32 ) m (1 aaln ) m � � m I n -
+
+
-
-
+ . . . +
+
'
-
+
as desired.
= n.
2m ,
( Titu Andreescu,
Revista Matematidi Timi§oara (RMT) , No. 1 (1974) , pp. 7, Problem 1564; Gazeta Matematidi (GM-B) , No. 2(1976), pp. 65) 6. We have
n k ) kn L kn-l L 2 kn-2 II( al al a i = l ai = +
+
Using the AM-GM inequality gives
+
+ ... +
ala2 · · .an·
237
6.2. SOLUTIONS From the previous relation we deduce
(n a;r kn-2 + n ( k + w.:) + . . + (�) . (nf k + � n a; ::; n (k + a;). 1
2
n ( k + a;) � kn + (�) (n ai r kn- 1 + (;) Thus
n
(1)
Using again the AM-GM inequality, we obtain
so
n
n
II
i=l
(k + ai) ::; k + -1 Ln ai. n i=l
(2)
(Titu Andreescu, Revista Matematidl Timi§oara (RMT) , No. 2 (1977), pp. 63,
Problem 3045)
First Solution.
7. Since abc = 1, this non-homogeneous inequality can be trans formed into a homogeneous one by a suitable change of variables. In fact, there exist positive real numbers such that
p, q, r
q c = -r . q p r Rewriting the inequality in terms of p, q, r, we obtain (1) (p - q + r) (q - r + p)(r -p + q) ::; pqr, where p, q, r > o. At most one of the numbers = p -q+r, = q -r+p, W = r-p+q is negative, because any two of them have a positive sum. If exactly one of the numbers is negative, then ::; 0 < pqr. If they are all nonnegative, then by the AM-GM inequality, 1 ..;uv ::; 2 ( + ) = p. Likewise, ..;vw ::; q and ..jWu r. Hence pqr, desired. Second Solution. Expanding [P(out the left-hand side of (1) gives (p-q+r)(q -r +p)(r -p+q) = p- r) + (r -q)(P-q) + q(r -q) + pq][r + (q -p)] = pr(p - r) + r(r - q)(p - r) + rq(r - q) + pqr + p(p - r)(q - p)+ +(r - q)(p - r)(q - p) + q(r - q)(q -p) + pq(q -p). a = !!. ,
b = -,
v
u
u, v, w
uvw
u
::;
v
uvw
::;
as
=
6. COMPREHENSIVE PROBLEMS
23 8 Note that
pr(p - r) + rq(r - q) + pq (q - p) + (r - q) (P - r) (q - p) = O . Thus (1) is equivalent to
o ::; p(p - q) (p - r) + q ( q - r) ( q - p) + r (r - p) (r - q ) , which is a special case of Schur's inequality. Third Solution. Denoting the left-hand side of the desired inequality by L, we have L = abcL = b a - 1 + c b-1+ = a c- 1 +
(
D(
D (
D
= (ab - b + l ) (be - e + l) (ea - a + 1) = L 1 • Also, since lib = ae, lie L=
(
= ab, 11a = be,
a- 1+
D(
b-1+
D(
C- 1+
D
=
= (a - 1 + ae) (b - 1 + ab) (e - 1 + be) = L2 . If u = a - I + lib ::; 0, then a < 1 and b > 1, implying that v = b - 1 + lie > 0 and W = e - 1 + 11a > o. Then L = uvw ::; 0, as desired. Similarly, either u ::; 0 or v 0 yields the same result. If u, v, W > 0, then all factors of Ll and L2 are positive. The AM-GM inequality ::;
gives
1 J(ab - b + l) (b - 1 + ab) ::; [ (ab - b + 1) + (b - 1 + ab)] = abo 2 Likewise, J(be - e + l) (e - 1 + be) ::; be,
J(ea - a + l) ( a - 1 + ae) ::; ca. Hence L = VL 1 L2 ::; (ab) (be) (ea) (abe) 2 = 1. Fourth Solution. Using the notations established in the third solution, it is easy to verify the equalities
=
beu + ve
= 2,
eav + aw
= 2,
abw + bu = 2.
As in the third solution, we only need to consider the case when u, v, w AM-GM inequality gives 2 2:: 2eVbuv,
2 2:: 2aVcvw,
> O. The
and 2 2:: 2vawu,
from which uvw ::; 1 . Fifth Solution. Let ab - b + 1, = be - e + 1 , = ca - a + 1; U2 = 1 - be + c, V2 = 1 - ea + a, and 2 = 1 - ab + b. As in the third solution, we only need to consider the case in which > 0 for i = 1, 2. Again, we have
Ul = W Ui , Vi , Wi
VI
WI
6.2. SOLUTIONS
239
Y Let X = a + b + e and = ab + be + ca. Then Y U l + VI + WI = - X + 3 and U + V + W 2
2
2
=
X - Y + 3.
Hence either U l + VI + WI :::; 3 or U2 + V2 + W2 :::; 3. In either case L :::; from the AM-G M inequality. ( Titu Andreescu, IMO 2000, Problem 2 )
1 follows
a �y (3 'Y. Let x, y, be three Then ( ) )(( ) y E (x, y, z - E ( , z, y = (1' - a z - X ) 2 - - x2 ) ) � 0, and (3( ( E( a, y, b) = a y - a) 2 + y - b) 2 + 'Y(b - a) 2 ( E (b , y, a) = a y - b) 2 + f3 (y - a? + 'Y( b - a) 2 . We need to find the maximal values of the functions ( ) a(y - a)2 + f3(y - b)2 fl y and (3( ( h (y) = a y - b) 2 + y - a) 2 on the interval [a , b] . Since h (a) = f3(b - a) 2 � a ( b - a) 2 II (b) and the coefficient y a. 2 of y in h is a + f3 � 0, it follows that the maximum value of II is obtained for y b. Likewise, h (b) � h (a) and the maximum value of h is obtained for 8. Without loss of generality we can assume that arbitrary numbers from the interval [a, b] such that x :::; x
:::;
z
:::;
z.
=
=
=
Therefore
=
y, b) = E(a, a, b) = (f3 + 'Y) (b - a)2 , and ({3 + 'Y) (b - a)2 . max E (b , y, a) = E (b , b , a) It follows that the maximum value of E is ( f3 + 'Y) (b - a) 2 and is obtained for a , y = a , z = b or = b, y = b, z a. max E (a,
yE[ a , b]
=
yE[ a , b]
x =
(Dorin Andrica
1 (1983) ,
pp.
x
and loan 66, Problem C5:2)
Ra§a,
=
Revista Matematica Timi§oara (RMT) , No.
9. For a function f : { 1, 2, . . . , n} --+ {a , b, e} let Ma = f - l ({a}) , Mb = f- 1 ({b}), Me f - 1 ({ e}) and let p, q, r be the number of elements of sets Ma, Mb, Me, respec tively. Obviously p + q + r n and without loss of generality we may assume that p � q � r. A term If(i) - f(j) 1 is different from ° if the pair (i, j) is in one of the sets Ma x Mb , Mb X Ma , Ma x Me, Me X Ma , Mb X Me or Me X Mb . Hence the number
=
=
n
of nonzero terms in the sum
L I f (i) - f(j) 1 is 2(pq + qr + rp). i,j=l
6. COMPREHENSIVE PROBLEMS
24 0
The problem reduces to finding the maximal value of 2 (pq+qT+TP ) when p+q+T == n and p, q , T 2:: 0 are integers. Note that if (Po, qo, TO) is a triplet that maximizes 2(pq+ qT+Tp) then the absolute value of any difference of two numbers from this triplet is at most 1. Indeed, assume that Po - TO 2:: 2 and define
PI Po - 1, ql Then PI
+ ql + TI
=
= n and
= qo , T = TO + 1. I
= Poqo + POTO + qo ro + Po - ro - 1 > Poqo + POTO + qoTo,
which contradicts the maximality of 2(Poqo + qoro + ropo). We have the following cases . 1) n 3k. Then Po + qo + TO 3k, Po 2:: qo 2:: ro 2:: Po - 1, hence Po
= qo = ro = k.
=
= k and then
In this case the maximal value is
2 k k ) 6 2 2 2 n + = = 3 . 2) n = 3k + 1. Then Po + qo + ro = 3k + 1, Po 2:: qo 2:: TO 2:: Po - 1, so 3po 2:: 3k + 1 2:: 3po - 2. Hence Po = k + 1 and then qo = TO = k. In this case the maximal value is k) 2((k + 1)k + (k + 1) k + 2 = 2(3k2 + 2k) = 32 (n2 - 1). 3) = 3k + 2. Then Po + qo + ro = 3k + 2, Po 2:: qo 2:: TO Po - 1, so 3Po 3k + 1 2:: 3po - 2. It follows that Po = k + 1 and qo + ro = 2k + 1. Because k + 1 qo TO k, qo = k + 1 and ro = k. The maximal value is in this case n2 --1 2[(k + l )(k + 1) + (k + l)k + (k + l)k] = 2(k + 1 )(3k + 1) = 2 · 3 2 2 n n -1 Therefore the requested number is 2 3 if 3 divides n and 2-- otherwise. 2 ( k2
+ k2
n
2:=
2::
2:=
3
Remark.
2:=
2:=
The problem can be reformulated as follows: Suppose that n points in space are colored by three different colors. Find the maximum number of segments AB such that A and B are different colors. (DoTin Andrica and Pal Dalyai, Romanian IMO Selection Test, 1982; Revista Matematica Timi§oara ( RMT ) , No. 1(1982) , pp. 83, Problem 4917) 10. From the inequality
4 fb 2 4 fb2 2 . . . ( .fiil ) + + (va; l ) + ( f02 �_ � � Y UI
it follows that
_
Y
U
_
'b)
4 Un Y
2
> - 0,
6. 2. SOLUTIONS
241
We have
hence
va;; � .jb; + Jb; + . . . + A,
va; + .Ja2 + . . . +
as claimed.
( Titu Andreescu, Revista Matematidi Timi§oara (RMT) , No. 2 (1977) , 63, 3046) Problem 1 11 . By mUltiplying the numbers aI, a2 , . . . , an with a suitable factor '" ai2p i=l n ;P 1 L we may reduce the problem to the case when a . i l = Assume without loss of generality that a al � a2 � . an· Let a = � and suppose by way of contradiction that S ( n , p) pp.
M =
-n
L-.t
=
=
.
.
�
2
Then
hence
n
n L a;P L[ 1 )] 2P . + a(i a i= l (0 i= l Consider the function
>
-+
=
-
00 ,
n
242
6. COMPREHENSIVE PROBLEMS Then
n 1 )]2 = 2p I) x + a(i - p - l i=l and n 1 )] O. i=l Because
0 for all real x, it follows that
a contradiction. Hence
4P n 2p ) 2 p rp.l� (a� aJ. <- S(n,p) i=l ai , l��
as desired.
.
__
_
'" L-.t
Remark. For p 1 we obtain the MitrinoviC 's inequality � � (n - 1) (Dorin Andrica)
=
(ai - aj )2 21 2 � ai2 ' ��f l �n n •
1 2. Note that
for all positive real numbers x. Setting x yfii 1 0, implies
then
= - > ( vn) n > 1 + (�) ( vn _ 1 ) k n - 1 > ( vn _ 1 ) k ,
It follows that
and then
as desired.
n-1
( �)
(� )
> ( vn - l) k , n
J� > ifrl - l,
� (�)
6.2.
243
SOLUTIONS
(Dorin Andrica, Romanian Winter Camp, 1984; Revista Matematidi Timi§oara
(RMT) , No. 1 (1985), pp.
72, Problem 1)
1 3. a) Clearly,
1 1
1 1
1 1
1 1
au a2 l
al2 a22
a l,n-l a2,n-l
al ,n a2,n
an - 2,l an-2,2
=0
an-2,n - l an-2,n
Expanding the determinant after the first row yields
hence
as desired. b) Consider the determinants
Ak
=
1
Xl X nl - 2
From equality a) we obtain
1
Xk -l
1
X k+ l
Xkn--l2 Xnk+-l2
1
Xn X�-2
n (- l)k k = l Pk qk
L - = O. c) Set Xk = k 2 , k = 1 , 2, . . . , n, in the previous equality. After some algebraic
manipulations we obtain
as desired.
n ( - l ) k k2 "'"' � (n - k)! (n + k)! = 0, n � 3,
(Dorin Andrica, Romanian Mathematical Regional Contest " Grigore MoisH" ,
1995)
244
6. COMPREHENSIVE PROBLEMS
14. Let X l , X2 , . . . , X n > 0 be the zeros of polynomial P. The relations between zeros and coefficients yield
2: Xl X2 ' Xm = • .
(:)
and 2: XlX2
•
•
.
xp =
�).
Then we have the equality case in the Generalized Mac Laurin 's Inequality:
L XlX . . . X > C�
m ------:-:-2---
rn
p
L Xl X2 . . . xp ' m�p Ch
= Xn - From 2: XlX2 . ' ' Xm = (:} it follows that Xi = 1,
hence X , = X2 = . . . i = 1 , 2, . . . , n, hence
P(x) = (x - l) n ,
as claimed.
( Titu Andreescu, Revista Matematidl Timi§oara ( RMT ) , No. 1 (1977) , pp.
Problem 2300)
24 ,
1 5 . It is obvious that degpo = 0 and deg pl 1. Assuming that degpk = k, from the given relation we obtain deg pk+l = k + 1 , hence by induction degpi = i for all
=
i = 0 , 1 , . . . , n.
Consider the function f : 1R -+
lR, f(x) = xn ex . It is easy to prove that
f C k) (x) = X n -k Q k (x)e X ,
where Q k (X) is a polynomial with real coefficients of degree k. We prove that Q k (X) = Pk (X) for all k. Note that Qo (x) 1 - Po (x) and
=
hence
xn - Ck+ l ) Q k+ l (x) eX f C k+ l ) (x) = (f C k) (x)) ' = (Xn - k Q k (x) eX ) ' = = Xn -C k+ l ) [ (n - k + X)Q k (x) + xQ � (x)] ,
=
Q k+ l (x) = (n - k + X)Q k (X) + xQ � (x). Since (Pk ) k=O , n and (Q kh= o ,n satisfy the same recursive relation and Po = Qo it follows that Pk = Q k for all k. So
and, on the other hand,
f C n) (x) It follows that
=
(x n eX ) Cn)
=
n L C� (xn ) C k) e x . k=O
6. 2 .
245
SOLUTIONS
(Dorin Andrica, Revista Matematidi Timi§oara ( RMT ) , No. 1(1978 ), pp. 67, Problem 3293) 16.
We prove that
P� (x) = -2nPn - dx), n � O.
Note that and assume that Then
P; (x) = -2 = -2 · 1 · Po (x) P� - l (x) = -2(n - 1)Pn -2 (x) , n � 2.
Differentiating we obtain
P� (x) = -2Pn - 1 (x) - 2XP� _1 (x) - 2(n - 1)P�_ 2 (x) = = -2Pn - 1 (x) 4(n - 1)xPn -2 (x) - 2(n - 1)P�_ 2 (x) = = -2Pn - 1 (x) - 2(n - 1)[-2xPn -2 (x) P�_ 2 (X)] = = -2Pn - (x) - 2(n - 1)Pn - 1 (x) = -2nPn - 1 (x),
+
+
1
as needed. The initial relation becomes so Hence
Pn (O) = -2(n - 1)Pn -2 (0), n � 2. Pn (O) =
{ �- l) � (�)!
if n is odd if n is even
Alternative solution . Note that ( e -X2 ) ( n ) = Q n (x) e -X2 for a polynomial Q n . From ( e-X2 ) ( n+ l ) = [(e -X2 ) ( n) ] , = (Q n (x) e -X2 ) , = = [Q � (x) - 2xQ n (x)] e -X2 = Q n + l (x) e -X2 , we obtain
Q n+ l (x) = -2xQ n (x) - Q � (x), n � O . Since Qo (x) = 1 = Po (x) , we note that Q n (x) = Pn (X) for all n � 0,
6. COMPREHENSIVE PROBLEMS
246 hence On the other hand,
e
_x2
=
X2 X214 X3!6
1 - 11
-
+-- -+...+
(-1)
n X2n
-
nl
+...
(1)
If n is odd, then by differentiating the series (1) for an odd number of times, we deduce that (0) o. If n is even, set n 2m. Differentiating (1) n times yields
Pn = (e-X' ) (2m) = (_l ) m (:) x [ ( - W+1 (�:+;) ! . J ' =
+
hence
Therefore
Pn (O)
=
{ �- l) " (%) !
+ . .
if n is odd if n is even
(Dorin Andrica, Revista Matematidl Timi§oara (RMT), No. 2(1978 ), pp. 76, Problem 3706)
nL a xk be a polynomial with nonnegative real coefficients. By P(x) k=O k the Cauchy-Schwarz Inequality we derive that P(x) (�va.. va.xkr � (�ak) (�akx2k ) P(1)P(x2 ). It suffices to prove that P(I) < 1. Indeed n 1 <1 L k=O l 1 as needed. Therefore p2 (x) < P(x2 ) for all x, so the equation has no real roots. 8 17. Let
=
=
=
n+ +
(Dorin Andrica, Revista Matematica Timi§oara (RMT) , No. 1-2(19 9), pp. 107, Problem C9: 8 ) 18. If
Let
P = 0, then for t
=
0 and for any function f
P(x) Lk=On akxk =
:
1R
--+
1R
the claim holds.
247
6.2. SOLUTIONS f:. with an 0, and let t E JR* . We search for a polynomial n+ l
Q t (x) = I: bk X k k=l
such that
Q t (x + t) - Q t (x) = P(x), x E JR.
(n ; )
Identifying the coefficients from both sides yields
1 tb = a n n+1
( +2 1) t2 n
bn+1
()
n + 1 t b n = an - l
: �) + . . . + G) t (� Note that the system has unique solution, hence there is a unique polynomial Q t
such that
n+1 b
b, =
n+ l
1Io
t
Q t (x + t) - Q t (x) = P(x), x E JR Q t (O) = 0,
and
deg Q t = 1 + deg P. Set g(x) = f(x) - Q t (x) . Then f satisfies the claim if and only if g(x+t) -g(x) = 0 for all real x, i.e. 9 has period t. Therefore
f(x) = Q t (x) + g(x), where 9 is a function of period t. (Dorin Andrica, Revista Matematidl Timi§oara (RMT), No. 2(1984), pp. 103, Problem C6:10) 1 1 1 1 1 1 9 . Because a, b, c, d, e E [-1, 1], there are real numbers x, y, t, u such 2 2 2 2 2
that
a = 2 cos x,
b
= 2 cos y,
c = 2 cos z,
d = 2 cos t,
e = 2 cos u.
U sing the identity
2 cos 5a = (2 cos a) 5 - 5 (2 cos a) 3 + 5 (2 cos a) ,
we obtain
2 cos 5x = a5 - 5a3 + 5a and the analogous relations. Summing them up yields
Z,
I: 2 cos 5x = I: a5 - 5 I: a3 + 5 I: a = 10
6. COMPREHENSIVE PROBLEMS
248 so
L cos 5x = 5. Hence
{
cos 5x = cos 5y = cos 5z = cos 5t = cos 5u = 1,
}
V5 - 1 V5 + 1 and therefore a, b, c, d, e E 2, --- ' - --- .
2 2 From the relation a + b + c + d + e = 0, it follows that one of the numbers is 2,
V5 + 1 V5 - 1 . easy to check that for these two of them --- and the other two - --- . It 1S
2
numbers L a3 = O .
2
( Titu Andreescu, Romanian Mathematical Olympiad - final round, 2002)
20. We will say that a subset of
N is p-progression-free if it does not con
tain an arithmetic progression of length p . Denote by bn the number obtained by writing n in base p - 1 and reading it in base p. One can easily prove that an = bn for all n = 0 , 1 , 2, . . . by induction, using the following properties of the set B = {bo, b1 , . . . , bn , . . . } (whose proofs we postpone) : 1 ° B is p-progression-free; 2° If bn - 1 < a < bn for some n � 1, then the set {bo, b1 , . . . , bn - 1 , a} is not p-progression-free. Indeed, assume 1° and 2° hold. By the definition of a k and bk , we have a k = bk for k = 0, 1, . . . , p - 2. Let al = bk for all k � n - 1, where n � p - 1. By 1°, the set is p-progression-free, so an � bn. Also, the inequality an < bn is impossible, in view of 2° . Hence an = bn and we are done. So it suffices to prove 1° and 2°. Let us note first that B consists of all numbers whose base p representation does not contain the digit p - 1. Hence 1° follows from the fact that if a, a + d, . . . , a + (p - l)d is any arithmetic progression of length p, then all base p digits occur in the base p representation of its terms. To see this, represent d in the form d = pm k , where gcd(k, p) = 1. Then d ends in m zeros, and the digit o preceding them is nonzero. It is easy to see that if a is the ( m + l)st digit of a (from right to left) , then the corresponding digits of a, a + d, . . . , a + (p - l)d are the remainders of a, a + 0, . . . , a + (p - 1)0 modulo p, respectively. It remains to note that a, a + 0, . . . , a + (p - 1 ) 0 is a complete set of residues modulo p, because 0 is relatively prime to p. This finishes the proof of 1 ° . We start proving 2° by the remark that bn - 1 < a < bn implies a � B . Since B consists precisely of the numbers whose base p representations do not contain the digit p - 1, this very digit must occur in the base p representation of a. Let d be the
6. 2 .
2 49
SOLUTIONS
number obtained from a be replacing each of its digits by 0 if the digit is not p - 1, and by 1 if it is p - 1 . Consider the progression
a (p - l)d, a - (p - 2)d, . . . , a - d, a. As the definition of d implies, the first p - 1 terms do not contain p - 1 in their base p representation. Hence, being less than a, they must belong to {bo, b 1 , , bn - I }. Therefore the set {bo, b1 , . . . , bn - 1 , a} is not p-progression-free, and the proof is fin -
• • •
ished.
0
( Titu Andreescu, USA Mathematical Olympiad, 1995)
21. i) For a bounded region M of the plane we denote by n(M) the number of
points with nonnegative integral coordinates in M . Function f is increasing and bijective, hence continuous. Consider the sets
= { (x, y ) E 1I�? I a :S x :S b, 0 :S y :S f(x) }, 1 M2 = {(x, y ) E 1I�? 1 c :S y :S d, O :S x :S f - ( y )}, M3 = { (x, y ) E ]R2 1 0 :S x :S b, O :S y :S d}, M4 = { (x, y ) E ]R2 1 O :S x :S a, O :S y :S c} . M1
a
o
Then
n (M1 ) =
L [f(k)] , n (M2 ) = L [1 - 1 (k)],
a�k�b
n (M3 ) =
We have hence and the conclusion follows.
b c�k�d
[b] [dJ , n(M4) = a(a)a(c).
6. COMPREHENSIVE PROBLEMS
250
nJ [1, n(n 2+ 1) ] , + 1) f(x) = x(x 2 .
il) Consider the function f : [1,
--t
Function f is increasing and bijective. Note that ' £ormu 1a 1' ) we 0btam . A pp Iymg '
- 1 + VI + 8x 2 t, [ k(k ; I ) ] + hence
n ( n +l)
t.
t.
n(Gf)
n and f - X
l( )
[ -1 + �] _ n = n2(n2+ 1) ,
n (n + l )
[ - 1 + � ] = n2 (n2+ 1) + n - � t, k(k + l) = 2 + 1) = n(n2 + 2) = n (n2+ 1) + n - n(n 4+ 1) - n(n + 1)(2n 12 3 ( Titu Andreescu and Dorin Andrica, " Asupra unor clase de identitati" , Gazeta Matematica (GM-B), No. 11(1978), pp. 472-475) 22. i) Function f is decreasing and bijective, hence continuous. Consider the sets Nl = {(x, y ) E ]R2 1 a ::; x ::; b, c ::; y ::; f(x)}, N2 = { (x, y ) E ]R2 1 c ::; y ::; d, a ::; x ::; f - 1 (y )}, N3 = { (x, y ) E ]R2 a ::; x ::; b, 0 ::; y ::; c}, N4 = {(x, y ) E ]R2 1 0 ::; x ::; a, c ::; y ::; d}.
I
d
c
a
o
Then
L [f(k)] = n(N1 ) + n(N3 ), L (f - l (k)] = n(N2 ) + n(N4 ),
c�k�d
b
6. 2. SOLUTIONS
25 1
=
n(N1) n (N2 ), and n(N3 ) ([b] - a(a))a(c), n(N4 ) ([d] - a(c) )a(a) =
=
It follows that
L
a�k�b
as
[f ( k )] -
desired. ii) Consider the function
Note that
[j - 1 ( k )] n(N3 ) - n(N4 ) [b]a(c) - [d] a(a), =
L
c�k�d =
=
f : [1, n] [1 , n2 ], f (x) = n2x2 --+
f is decreasing and bijective and f - 1 (x) Vxn · =
Using formula i), we obtain
hence
n2
{; [�:] - (; [�] na(l) - n2a(1) n
=
=
0,
as desired.
(Dorin Andrica and Titu Andreescu, Gazeta Matematidl (GM-B), No. 6(1979), pp. 254, Problem 0.48)
a b � 0 such that Va - Vb ([Vb] 1)2.
23. It is easy to prove that between numbers there is a perfect square - take for example + It suffices to prove that
>
Jn 1 + . . . + nm+ 1 - Jn1 + . . . + nm
>
m
1,
�
>
1
1.
This is equivalent to
n1 + . . . + nm + nm+ 1 and then
>
(1 + Jn1 + n2 + . . + nm )2 .
1 + 2Jn1 + n2 + . . . + n m , m � 1 . We induct on m. For m = 1 we have to prove that n 2 > 1 + 2J1i1. Indeed, n2 � n1 + 2 = 1 + (1 + n1) > 1 + 2.Jril. Assume that the claim holds for some m � 1 .
nm+ 1
Then
>
252
6. COMPREHENSIVE PROBLEMS so (n m+l - 1 ) 2 4(nl nm ) hence (nm+l 1)2 4(nl + nm+d. This implies n m+l 1 2y'nl nm+l, and since nm+2 - nm+l � 2, it follows that + . . . +
>
+
+
+
>
.
.
.
+ . . . +
>
as desired.
( Titu Andreescu, Gazeta Matematidi (GM-B), No. 1(1980) , pp. 41, Problem
0. 113) 24.
Substituting
=
Xn+l x� - 2 in the relation
yields
+
=
UnX� - VnXn - (2un 1) 0, n � 1. (1) For a given n � 1 the relation (1) is a quadratic equation with integral coefficient and with an integer root Xn - Hence the discriminant � v� 8u� 4u n t� +
=
+
=
,
is a square, as desired.
(Dorin Andrica)
(an ) n2:n1 be a sequence of real numbers. From the equality x n xn x2n . . . xkn . . , I x i < 1, n � 1 1-x we derive n - L An Xn , L � 1 - xn n= l l n= where An La ndi d . Using Gauss' formula L
Let
--- =
+
+
+
00
+
.
00
=
=
=
=
26.
-
L...J
-
=
We start with a useful lemma.
=
253
6.2. SOLUTIONS Lemma.
There are gcd(k, n) - 1 integers among 1.n 2·n (k - l)n -k- ' -k- ' " ' ' k
Proof. Let gcd(k, n) = d, k = kId, n = ni d and note that gcd(k l , n l )
numbers are
1 · ni 2 · ni
= 1 . The
(k - l)n l ki T T The number of multiples of ki in the set 1 , 2, . . . , k - 1 is d - 1, hence among the above numbers there are d - 1 = gcd(k, n) - 1 integers, as desired. '
The line OAk has the equation
"
' "
n Y = k · x.
C
== A n - I B == Ao '-��----'---�
D
o
From the lemma it follows that among numbers 1.n 2·n (k - l)n
-k- ' -k- ' " ' '
k
there are gcd(k, n) - 1 integers. Hence the open segment OAk does not contain points with integral coordinates if and only if gcd(k, n) = 1. There are
(Dorin Andrica)
27.
We have
{12 + {/4) ) n an + bn {12 + cn {14 = (1 + 312 + {I4) n = ( {I2(l +({I2) = n V L,
= 2 - i ( ij2 + {14 + 2 ) n = 2 - i (1 + ( 1 + ij2 + {I4) ) n =
6. COMPREHENSIVE PROBLEMS
254 hence
an + bn � + cn ift! = 2 - t
f ( � ) ak + k=l (1)
We study three cases. i) If n == 0 (mod 3) , then 2 - t
2- t
f ( � ) a k = an , k=o
E
2- t
ii) If n == 2 (mod 3), then 2 we obtain
-
±2
Q, hence
f (�) bk = bn , k=O
�
(I)
E Q. Multiplying the relation
(1 ) by 2� = �,
(2) then
2 Hence
2- �
� (� ) a k = b n � '
Ti
iii) if n == 1 (mod 3) , then 2 {!2, we obtain
� (�) bk = cn �'
-;±1
E
- n ±2 3
2- i
n L k=O
( ) Ck = 2an · n k
� (�) Ck = �.
(II)
Q. By mUltiplying the relation (1) by 2� =
(3) then
2 Hence
.=.!!±l 3
n
L k=O
( ) ak = 2cn, n k
6. 2 .
2 55
SOLUTIONS
a n, tu (�)a. {
Relations (I) , (II) , (III) imply TF
=
2- F
tu (�)
and 2- �
n=O n=2 n=l n=O n=O n=l
bn �'
Cn .ij4, bn, Cn {12,
1a
{ � (�)c. f' b. =
n
{12 '
(mod 3) (mod 3) (mod 3) (mod 3) (mod 3) (mod 3)
Cn ,
n = O (mod 3) n = O (mod 3)
{12 '
n = l (mod 3)
=
( Titu Andreescu and Dorin Andrica, Revista Matematica Timi§oara (RMT), No. 1 (1984) , pp. 83, Problem C6:3) 28. Note that (\.1"2 + V3y� =
an +
bn\.1"2 +
cn v'3 + dn V6, n � 1. Let n
�[(5 + 2v6) k + (5 - 2v6) k ] , Yk = � v6 [ ( 5 + 2 v6) k - (5 - 2v6) k ) . Xk =
Then hence
an
1
bn = Cn = 0 and dn = Y .!!. . 2
= - x .!!. ,
2 2
= 2k and
Let n = 2k + 1. Then (v0 + -/3) 2k+ l = (v0 + v'3) (v0 + v'3) 2k = ( X k + 3 Yk ) v0 + ( X k + 2 Y k )v'3 , for all k � 1, hence
an
b n = X n-l 2
+ 3Yn-l , Cn = X n-l + 2Y n-l , dn = 0 = 0,
2
(Dorin Andrica)
2
2
29. We have
f(P, q) = f(p - 1, q) + p - 1 p (p ; 1) = f(P - 2, q) + (P - 2 ) + (p - 1) . . . = f(l, q) + p(p - 1) = f(l, q - 1) + (q - 1) +
=
=
2
= f (l, 1)
_
q(q ; 1) + p(p ; 1)
=
.
.
.
=
= 2 00 1 .
=
256
6.
COMPREHENSIVE PROBLEMS
Therefore
p (p - 1) _ q (q - 1) = 1999 2 2 (p - q) (p + q - 1) 2 · 1999. Note that 1999 is a prime number and that p - q < p + q - 1 for p, q E N* . We
=
have the following two cases: ( a) p - q = 1 and p + q - 1 = 3998. Hence p = 2000 and q = 1999. ( b ) p - q = 2 and p + q - 1 = 1999. Hence p = 1001 and q = 999. Therefore (p, q) = (2000, 1999) or (1001, 999). ( Titu Andreescu, Korean Mathematics Competition, 2001)
30. The only solutions are I(x) = 0, I(x) x, and I(x) = -x. First, it is clear that these three are solutions. Next, setting x = y = z = 0, we find 1(0) = 3(/(0))3 , the only integer solution of which is 1(0) = O. Next, with y = -x and z = 0, we obtain 1(0) = (/(X))3 + (/( -X))3 + (/(0))3 . This yields I(-x) = -I(x), so 1 is an odd function. With (x, y, z) = (1, 0, 0), we obtain 1(1) = (/(1))3 + 2(/(0))3 = 1(1)3; thus 1(1) E {-1, 0, 1}. Continuing with (x, y, z) = (1, 1, 0) and (x, y, z) = (1, 1, 1) yields 1(2) = 2(/(1))3 = 21(1) and 1(3) = 3(/(1))3 = 31(1). To continue, we need a lemma.
=
Lemma. If x
is an integer greater than 3, then x3 can be written as the sum 01 five cubes that are smaller in magnitude than x3 . Proof. We have 43 = 33 + 33 + 23 + 13 + 13 , 53 = 43 + 43 + ( _ 1)3 + ( _ 1)3 + ( _ 1)3, 63 = 53 + 43 + 33 + 03 + 03, and 73 = 63 + 53 + 13 + 13 + 03. If x = 2k + 1 with k > 3, then
x3 = (2k + 1)3 = (2k _ 1)3 + (k + 4)3 + (4 - k)3 + ( _ 5) 3 + ( _ 1)3, and all of {2k - 1, k + 3, 14 - kl, 5, I} are less than 2k + 1. If x > 3 is an arbitrary integer, then write x = my, where y is 4 or 6 or an odd number greater than 3 and m is a natural number. Express y3 as yr + y� + y� + y� + y�. The number x3 can then be expressed as (mYl )3 + (mY2 )3 + (mY3 )3 + (mY4 )3 + (mY5 )3 . 0 Since 1 is an odd function and 1(1) E {-I, 0, 1}, it suffices to prove that I(x) = xl(l) for every integer x. We have proved this for Ixl ::; 3. For x � 4, suppose that the claim is true for all values with magnitude smaller than x. By the lemma, x3 = xr +x�+x�+x� +x�, where IXi l < x for all i. After writing x3 + ( -X4 )3 + ( -X5 )3 = xr + x� + x� , we apply 1 to both sides. By the stated condition of 1 and the oddness of I, we have Therefore, the inductive hypothesis yields
5 5 (/(X))3 = I:(xi /(1))3 = (/(1))3 I: xr = (/(1))3x3. i= l i=l
257
6.2. SOLUTIONS
Thus f(x) = xf(l), and the result follows by induction. ( Titu Andreescu, The American Mathematical Monthly, Volume 108, No. 4(2001), pp. 372, Problem 10728)
31. Assume by way of contradiction that the distance between any two points is greater than or equal to 1. Then the spheres of radius 1/ 2 with centers at these 1981 points have disjoint interiors and are included in the cube of side 10 determined by the six parallel planes to the given cube's faces and situated in the exterior at a distance of 1/2. It follows that the sum of the volumes of the 1981 spheres is less than the volume of the cube of side 10, hence
11
4" . 2 � 1981 ·
3
=
1981 .
�
>
1000,
3 a contradiction. The proof is complete. Remark. The pigeonhole principle does not help us here. Indeed, dividing each side of the cube in [ {!1981] = 12 congruent segments we obtain 1 2 1728 small cubes of side � = � . In such a cube there will be two points from the initial 1981 points.
12
4
3=
�
�
The distance between them is less than J3 which is not enough, since J3 > 1 . ( Titu Andreescu, Revista Matematica Timi§oara (RMT) , No. 2(1981), pp. 68, Problem 4627)
32. Note that
2:63 I nk - kl = 2:63 ck (nk - k) , where Ck { - I , I } , k=l k=l 2:63 I nk - kl = ±63 ± 63 ± 62 ± 62 ± . . . ± 2 ± 2 ± 1 ± 1, S= k=l E
hence
with 63 signs of + and 63 signs of -. Then
S ::; (63 + 63 + 62 + 62 + . . . + 33 + 33 + 32) - (32 + 31 + 31 + . . . + 1 + 1) = 1984,
as desired.
Remark. We prove that, for some labeling, S = 1984. It is known that a knight
can pass through all the 64 squares of the board only once and then come back to the initial square. Now label the squares from 1 to 64 in the order given by these knight moves. The free position can be made successively 64 1, 2, . . . , 63, 64, . . . so we can = 1, 4 = 2, . . . = 31. = 63, reach the situation = 32, = 33, . . For this diagram we have S = 1984. ( Titu Andreescu, Revista Matematica Timi§oara (RMT) , No. 2(1984), pp. 103, Problem C6:6)
nl
n2
.
, n32
,
n33 n3
, n63
258
6. COMPREHENSIVE PROBLEMS 33. Note that
hence
(1)
a 3F2n , b = -F2n+2' and e = -F2n-2 in the algebraic identity a3 + b3 + e3 - 3abe = (a + b + e)(a2 + b2 + e2 - ab - be - ea) 27F:tn - F:tn+2 - F:tn - 2 - 9F2n+2F2nF2n-2 = O.
for all n � 2. Setting = gives
Applying ( 1 ) twice gives
F2n (3F2n-2 - F2n) - F:jn-2 = F2nF2n-4 - D�n-2 = = = F6F2 - Fi = - l . The desired result follows from 9F2n+2F2nF2n-2 - 9F:tn = 9F2n(F2n+2F2n-2 - Fin ) = -9F2n· =
.
.
.
( Titu Andreescu, Korean Mathematics Competition, 2000)
34. Let r be the ratio of an arithmetic progression. We have 1 r k - 1 ) :S n, so r ::; = It follows that the maximum ratio is q = = Let r E 2, . . . , q } and let E 2 , . . . , n } b e the greatest first term of a
� � {I , progression with ratio _
r.
Then
ar {I , ar + ( - 1) k :S n,
1 � � I-
+(
(1)
r
r
to the number of arithmetic progressions of ratio from the set { I , ar2, . .is. , equal n}. Hence N(n, k) = al + a2 + . . . + aq• Because al = n - k + 1, using ( 1 ) gives N :S al + n - k + n - 2k + . . . + n - (q - l)k = = n q - k + 1 - k ( 1 + 2 + . . . + (q - 1 )) = nq - k - (q - l)qk + 1 = 2 2 ( k q = - 2 k + n + '2 ) q + 1 - k .
so
It suffices to prove that
� k + (n + � ) q + 1 - k ::; - � q2 + (n + � ) q
6. 2. SOLUTIONS 259 k � 1 q ::; k � 1 q2 . This inequality is clearly true, so we are which is equivalent to done.
Alternative proof. Note that n � k � 2. The arithmetic progressions with k terms and ratio r = 1 are
1, 2, . . . , k 2, 3, . . . , k + 1
n - (k - 1), . . . , n + such progressions. so there are The arithmetic progressions with terms and ratio r = are
n-k 1
k 1, 3, . . . , 2k - 1 2,4, . . . , 2k
2
n - 2(k - 1), . . . , n so a total of It follows that
n - 2(k - 1) progressions. q
Nk = I: [n - d(k - 1)], d=l
where
q is the greatest ratio of an arithmetic progression with k terms from the set {I, 2, . . . , n} . We have proved that q = [ � = � ] , hence + 1) Nk = t (n - d(k - 1)) = nq q(q l�(k - . _
d= l
It suffices to show that
(k - 2)q2 + kq + 2 - 2k � O .
(1)
The roots of the quadratic polynomial from the left-hand side of the inequality are
-2(k - 1) and q2 = l . k 2 Note that ql < 0 for k 2. Since q = [ � = � ] :2: 1, the inequality (1) is true for all k 2 . If k = 2, then is easy to check that the claim holds as well. ( Titu Andreescu and Dorin Andrica, Revista Matematidi Timi§oara (RMT) , No. 1(1982), pp. 104, Problem C4:2) 3 5 . Denote by Pn (x) the ( unique) polynomial of degree n such that Pn (k) = Fk for k = n + 2, n + 3, . . . , 2n + 2. (1) ql =
>
>
_
260
6.
COMPREHENSIVE PROBLEMS
We are going to show that Pn (2n + 3) = F2 n+ 3 - 1 for all n � O. Clearly, Po (x) = 1 and the claim is true for n = O. Suppose it holds for Pn - I (x), and consider Pn (x) . The polynomial has degree at most n - 1. In view of (1), for each k = n+1, n+2, . . . , 2n. Therefore Q (x) and Pn - I (x) agree at n distinct points, and hence Q (x) = Pn - 1 (x) for all x . In other words, Pn(x + 2) = Pn (x + 1) + Pn - I (x) for all x . Combined with the inductive hypothesis P2n - 1 (2n + 1) = F2 n+ 1 - 1, this implies Pn (2n + 3) = F2 n+2 + F2 n + 1 - 1 = F2 n+ 3 - l. ( Titu Andreescu, IMO 1983 Shortlist) 36.
For an arbitrary integer n we have
Xn+ 1 = aX n + (3Xn - l , (1) Xn = aX n - 1 + (3Xn -2 , n � 2. This is a system of linear equations with unknowns a and (3. The solutions are �a = XnXn-1 - Xn+IXn -2 a= 2 - XnXn -2 � Xn-1 and
_ X� - Xn - IXn+ 1 -(3 - �(3 � - X�_ I - XnXn - 2 _
Since a and -(3 are constant, the conclusion follows. (Dorin Andrica) 37.
Let � n = an - an - l for n � 2. Because 2 + . . . + ank--kI ) an+ 1 = k1 + an + an-I +2
(b
and we have
(1) where
Al = an-I + an- 2 A2 = a�_ 2 + an- 2 an - 3 + a�_ 3 k -2 k-3 Ak- I = ank-2 - k+2 + . . . + an -k+2 an _ k+ 1 + an - k+ l · \
6. 2 .
261
SOLUTIONS
Note that a 1 = a2 = = a k - 1 = 0 implies an > 0 for � k, so A1 , A2 , . . . , Ak- 1 � O. On the other hand, 6.1 = 6. 2 = . . . , 6. k -1 = 0 and from relation (1) it follows that 6.n > 0 for all � k. Hence the sequence ( an ) n� k is increasing. We prove that an < 1 for all � 1 . Assume that .
n
.
n
.
n
so
l.
since b < Therefore the sequence is upper bounded, so is convergent. Let x = nlim an. Then
-+oo
l
(2) Xk- 1 + X k-2 + . . . + x2 - (k - ) x + b = 0 If b = 0, then an = 0 for all � 1, hence x = O. If b E (0, 1), we prove that the equation (2) has a unique solution in the interval (0, 1). Let I [0, 1] -+ JR, I(x) = X k - 1 + X k-2 + . . . + x2 - (k - l ) x + b. Then 1(0) = b, 1(1) = b - 1, 1(0)/(1) = b ( b - 1) < 0 hence the equation (2) has an odd number of solutions in the interval (0, 1) . The
n
:
function I is twice differentiable and since I " (x) - (k - l) (k - 2)Xk - 3 + 2 > 0,
x E (0, 1),
I is concave up on (0, 1) . It follows that equation (2) has at most two solution in (0 , 1) therefore the conclusion follows. Remark. The claim that equation (2) has a unique positive solution it follows from 1(0)/(1) = b ( b - 1) < 0 and from the fact that I(x) = xk- 1 + X k-2 + . . . + x2 - (k - l ) x + b has a unique variation of sign (Descartes). (Dorin Andrica, Revista Matematica Timi§oara (RMT) , No. 1-2(1979), pp. 56, Problem 3 8 66) 38.
From the relation iii) we obtain
an + 1 - a n an
+
b n+ 1 - b n - 0 bn
, n � 1,
6. COMPREHENSIVE PROBLEMS
262 then
' an+l - an 1·1m ( - -ab n ) = nl-+oo 1m - bn b n-+oo n+l n On the other hand, by the Stolz-Cesaro theorem we have ·1m abn+l -- anbn = nl-+oo ' m an l -bn n1-+oo n+l It is easy to see that relations (1) and (2) imply l· m -anbn = 0, n1-+oo
Using ii) yields
(1) (2)
as desired.
( Titu Andreescu, Revista Matematidi Timi§oara (RMT) , No. 1 (1978 ), pp. 69 ,
Problem 39.
3304)
Consider the determinant
0 X l X2 Xn X l 0 X2 Xn � (Xl, X2 , . . . , Xn ) = Xl X2 0 Xn X l X 2 X3 0 Note that �(0,X2, . . . ,Xn) = �(XI,0,X3 , . .. , Xn ) = . . . =�(Xl,X2 , . . . ,Xn-I ' 0) = O. Moreover, if Xl + X 2 + . . . + X n = 0 , then �(XI ' X 2 , . . . , x n ) = 0 , therefore �(XI ' X2 ,·· . , xn ) = aXI X2 . . . Xn (XI X2 + . . . xn), for some real number a. By identifying the coefficients of XIX2 Xn from both sides we obtain a = +
+
. • .
l.
Hence
We have
---'-. . . + Uk UIUI++U2U2++. .. .. +. +Uk+l UI . . . Uk = 1 + UI +Uk+l Uk then n + U 2 . . . + Un+l UI ----II (1 + UI . . . Uk) = UI + U2 ----'-k=2 Since U 2 = -uI , we obtain UI + . . . + Un+l 1 n -n II (1 + UI . . . Uk) = nU ( 1 - UI l ) k=2 1+
--
-
+
263
6.2. SOLUTIONS U = nlim-+oo Un and note that + . . . + Un+l = U. U l 1m n-+oo It follows that 1 IIn (1 + Ul . . . Uk ) = U lim n-+oo k=2 Ul ( 1 - Ul ) Let
1.
n
-
n
.
( Titu Andreescu, Revista Matematica Timi§oara (RMT), No. Problem 3533)
40. Let
a = nlim-+oo anbn and let
bn
>
O . There is an integer n (c)
>
0 such that
k+1 < an k + 1 k - 1 bn < a + c k - 1 for n - n (c) 0 it follows that abn - c kk +_ 11 bn < an < abn + c kk +_ 11 bn for all n ;::: n (c) . + 1 n+1 + bn ) < an+l - an < a(bn+1 - bn ) - c kk -(b -1 k + 1 (bn+1 + bn ) , < a(bn+ 1 - b n ) + c k 1 a-c
Since
c
2(1978), pp. 52,
>
--
--
>
Then
-
_
and, dividing by
bn+1 - bn 0, we obtain k + 1 bn+1 + bn an+l - an k - 1 bn+ bn bn+1 - bn >
a - c -- ·
1
-
<
< a+c
k + 1 - bn+ 1 + bn , k + 1 bn+ 1 - bn
--
for all n ;::: n(c). From relation ii) we deduce
bbn+11 + bbn - kk +- 11 , 1, n+ - n n+l -- anbn a + for all ( a - abn+l -'-------::- <
hence
c<
Therefore
--
<
c
n ;:::
n ;::: n c)
as desired.
(Dorin Andrica, " 0 reciproca a teoremei Stolz-Cesaro §i aplicatii ale acesteia" , Revista Matematica Timi§oara (RMT) , No. pp.
2(1978), 6-12) 41. Let X l = k + Jk2 + 1 and X2 = k - Jk2 + 1. We have 1 < -1 < -1 I X2 1 = Xl 2k - 2' -
2 64 so
6.
COMPREHENSIVE PROBLEMS
- 0) n � x� � 0) n Hence ( l ) n - 1 < an < Xnl - ( l ) n + 1 <. Xln + X2n + 1 Xnl + X2n - 1 < X nl + 2"
2"
for all
n � 1. The identities
X�+ l + X�+ l = (X l + X2 ) (Xr + x�) - XIX2 (X � - 1 + X� - l ) = �1 = 2k(xr + x�) + (X� - l + X� - l ), show that xr + x� is an integer for all and since an is an integer, it follows that an = xr + x� for all � 0, and that an+ l = 2kan + an - l for all � 1.
n
n,
n
Then
n
and
Therefore
( Titu Andreescu)
42. Assume by way of contradiction that f(xo) =I- 0 for all X E (p) f (x) = 0 for all integers p > O. For p = 0, 2, 4, . . . 2 ( - 1), we obtain
,n
sin a lX + sin a2 x + . . . + sin an x = 0 ai sin al x + a§ sin a2 x + . . . + a� sin an x = 0
R
Clearly,
(1)
. an x = 0 , . a lX + a2( n - l ) sm . a2 X + . . . + an2( n - l ) sm a2(l n - l) sm 2 for all X e R Consider a number X such that at least one of sin al x, sin a2 X, . . . , sin an x is not zero. Then the homogeneous system of linear equation (1) has a nontrivial solution, hence the determinant is zero: 1
�8 =
a§
a2(l n - l ) a22( n - l )
= an2( n-l )
II (a; - a;) = O.
i , i =l i>i
6.2. SOLUTIONS
265
= = It follows that for some k =I- l i.e. which is a contradiction. The solution is complete. (Dorin Andrica, Revista Matematidi Timi§oara (RMT) , No. pp. Problem
a� aI
l ak I lad,
1-2(1980),
69,
4148)
43. Assume by way of contradiction that g JR -+ JR, g(x) = sin f(x) , is periodical. We have g' (x) f ' (x) cos f(x) , and since f and f' are continuous then g' is also :
=
continuous. Note that if g is periodical, then g' is periodical. Moreover, g' is continuous, so it is bounded. Consider the sequence Yn = (4 � Function f is continuous and = nlim -+oo Xn 00 , hence f(xn) = Yn for sufficiently large. Then g' (xn ) = f' (xn) so
n + 1) �, n 1. n
nlim -+oo g'(x n) = nlim -+oo f'(x n ) = 00 which is a contradiction, since g' is bounded. This concludes the proof. (Dorin Andrica, Revista Matematidi Timi§oara (RMT) , No. 2(1978), pp. 54, Problem 3544)
44. i) Let f be injective and assume by way of contradiction that is rational. Hence there are integers with = P. and gcd(p, = = 0 a contradiction. Therefore is irrational. Then f(q) = = Conversely, let be irrational and assume by way of contradiction that f = = f(n) for some integers =I- � O. Then so We obtain
f (2q) a
p, q
q) 1.
a q
a
a
(m) m n {am} {an} am - [am] an - [an]. a = [am]m -- n[an] Q, which is a contradiction, and the conclusion follows. ii) Let p, q be relatively prime integers such that a = P. . We have q f (n) = {an } = { r; } . E
q
By the division algorithm, there are integers and r such that
n = tq + r,
Then
r E {O, 1, 2,
. . . , q - I}.
{ P(tqq+ r) } = {pt + r: } = r : } = f(r) . We prove that f(O), f(l), . . . , f(q - 1) are all distinct. Indeed, if f(i) = f(j), then { i: } = { j;} f(n)
=
6.
266 It follows that i - j 0, so i j . Therefore
=
=
COMPREHENSIVE PROBLEMS
(i - j )p is an integer. Note that (p, q) q
M = {f(O), f(I), . . . , f(q - I)}
= 1 and Ii - j l < q, hence
= { 0 , qI , q2 , . . . -
-
q-l q
, -
}
since M has q elements. (Dorin Andrica, Romanian Winter Camp, 1984; Revista Matematica Timi§oara ( RMT ) , No. 1 ( 1985 ) , pp. 67, Problem 3)
45. Assume by way of contradiction that there is a number t > 0 such that g(x + t) g(x) , x E R
=
Then
f( x + t) + f(xO + to) = f(x ) + f(xO ), x E JR,
hence
f(t) + f(tO)
From the relation iii) it follows that
f(t) and then
t
= kI T
= 2f(0) = 2M.
= f(tO) = M
and to = k2 T,
for some integers k l ' k2 -:f. O. This gives
0 = kik2 E Q,
a contradiction.
(Dorin Andrica)
46. We start with an useful lemma. Lemma. If is an irrational number, then the set M {mO + nl m, n integers} is dense in JR. Proof. We prove that in any open bounded interval J � JR\ {O} there is an element of M, i.e. I n M -:f. 0 . Let J be such an interval and without loss of generality consider J C (0 , 00). There is an integer n (J) such that
0
=
1
n (J) J C (0, 1 ) . We consider two cases:
267
6. 2. SOLUTIONS 1 J = (O, e) , wI. th 0 < e < 1. n(J) Let N be an integer such that < e and consider the numbers 1 . J1
=
�
{B}, {2B}, . . . , {NB}. There are p, q E {I, 2, . . . , N, N + I} such that 0 < {pB} - { qB} :::; 1 N On the other hand,
M, M,
{pB} - { qB} = [qB] - [PB] + (p - q)B E hence JI n 0. It follows that n(J) ( {pB} - { qB}) E J n as desired. 1 . 2. J1 = n(J) J = ( a, b) wIth 0 < a < b < 1. Then 0 < b - a < 1 and by case 1), there is c E such that 0 < c < b - a.
M -:f.
Let
M
= [�]
no + 1. n J1 . Likewise, J n
M
M -:f.
Then a < noc < b and no c E 0, as desired. The lemma is now proved. a) Let A E min f(x), max f(X) . Hence there is Xo E 1R such that f(xo) = A. x E IR x EIR From the lemma we deduce that there are sequences (Xn ) n 2: 1 and (Yn) n 2: 1 such that
[
]
nlim -+oo (xn + YnT) = Xo .
The function f is continuous, so
nlim -+oo f(xn + YnT) = f(xo) = A. Note that f(xn + YnT) = f(xn ), therefore lim f(x n ) = A, n-+oo as desired. b) Let B be an irrational number and consider the function g(x) = f(xB) , x E R The number is irrational and a period for the function g. Using the result from a) , there is a sequence (xn ) n 2: 1 of integers such that
�
A = nlim -+oo f(Bxn), -+oo g(xn ) = nlim as desired.
(Dorin Andrica, " Asupra unor §iruri care au multimea punctelor limita intervale" , Gazeta Matematica (GM-B) , No. 11 (1979) , pp. 404-406)
47. i) Let = x + Y = z + v and assume that x < Y and z < v . Then we have x < -2 ' z < -2 ' Y = - x and v = - z . u
u
u
u
u
268
6. COMPREHENSIVE PROBLEMS Suppose, by way of contradiction, that there is a number A > 1 such that
xA + y A Consider the function f : (0, u )
-+
f (t)
=
=
Z A + VA .
(0, (0) ,
tA + ( - t) >.., u
and note that f is differentiable. We have l' (t) = A [t A -1 - ( u - t) A - I J ,
t E (0,
( �).
u ,
)
( �)
and since A > 1, it follows that f is increasing on 0, Both x, z are in 0, , so f(x) -:f. f(z) , because x -:f. z. This implies XA + yA -:f. Z A + v\ which is a contradiction. ii) Because p is a prime, by Little Fermat ' s Theorem we have
aP - a == bP - b == cP - c ==
hence
rF
- d == ° (mod p),
- ( aP - ) + (bP - b) - (cP - c) + (dP - d) == ° (mod p).
From P + bP
a
=
a
cP + dP , we deduce that
a - c + b - d ==
°
(mod p)
(1)
By i) we note that + b -:f. c + d, therefore
a
and then as desired.
l - c + b - d l � p,
a l a - c l + I b - d l � p,
( Titu Andreescu, Revista Matematica Timi§oara (RMT) , No. 2(1978 ) , pp. 55,
Problem 3550)
48. i) Consider the function <.p : (0, 00) -+ JR, <.p (t) t ln t - t . =
The Mean Value Theorem yields for some (J
<.p(x) - <.p (y ) x-y E
=
<.pI ((J)
(y, x), then
It follows that
x In x - y In y - x + Y x-y
=
In (J.
6. 2. and so
XX yY
269
SOLUTIONS =
(eB) X - Y .
The inequalities y < B < x, imply as desired. ii) Setting x
XX :::; ( ex) X - Y , ( ey ) X - Y :::; y y
k + 1 and y k yields ek < (k + l) k < e, k > O. k+1 kk Multiplying these inequalities from k 1 to k n gives en 2 . 32 (n + l) n < en < n+1 1 . 22 3 . . n n and then en 1 < en n . < (n + 1) n+ 1 n! (n + l) Hence (n +-- l)-'-n - < n , < (n + 1) n+ 1 , -'. en en =
=
=
=
. 43 3 •
--
.
•
•
'
.
----
as claimed.
8 ( 1977), pp. 327, Problem 16820; Revista Matematica Timi§oara (RMT) , No. 1-2 ( 1980), pp. 70, Problem 4153) (Dorin Andrica, Gazeta Matematidi (GM-B), No.
49. i) Note that f is bijective and increasing,' therefore f is continuous. In this diagram, we have
S,
=
hence
I.' f ( )dt, S2 = ld t
Sl + S2
as desired.
y
=
bd -
r' (t)dt,
ac,
c o
a
b
x
6. COMPREHENSIVE PROBLEMS
270
ii) Again, f is continuous and the diagram shows that
S1 - S2 = (b - a) e - (d - e) a = be - ad,
as desired.
y d
e
a
o
b
x
(Dorin Andrica, Revista Matematica Timi§oara (RMT) , No. Problem 4363)
i) Let c > O . There is <52 > 0 such that 50.
(>r
>
e-x < c
0 such that I I1(t) 1 < c for all t °
1
1 1 et ll1(t) l dt
'
x
>
1 (198 1), >
(>r
pp.
62,
and there is
U�2 .
For x > <5 = max (<51 , <52 ) , we have
I 1" e -x
l
e' l' (t)dt :'0 e- X
1" e' ll'(t) l dt + e-X 1� e' ll'(t) l dt
<
< c + c e-X ( eX - e 01 ) < 2c,
and the conclusion follows. ii) We start with the following lemma. that
Lemma. If <.p : [0, 00) is a differentiable function with continuous derivative such
lim (<.p(x) + <.pI (x)) = a,
x -+oo
then the limit xlim <.p(x) exists and equals -+oo
a.
Proof. Without loss of generality we may assume that <.p(0) = O. Define w : [0, 00) --+ JR, w (t) = <.p(t) + <.pl (t) - a. Function w is continuous and
lim w (t) = O .
Note that
t-+oo
6. 2 . then by integrating on [0, x]. It follows that and from i) we obtain
271
SOLUTIONS
e"
1" etw (t) dt,
a X t.p(x) = a - eX + e -
lo0 x etw (t)dt,
lim t.p(x) = a.
x-+oo
Denote and note that
fm (x) = fm - l (X) + f:n - l (X) , Using the lemma, x-+oo lim fn(x) = A implies lim (X) = A. X-+OO fn - l
m >
O.
Applying the same argument, we finally obtain
lim fo (x) = x-+oo lim f(x) = A,
x-+oo
as desired. (Dorin Andrica) 51.
Note that
(f(x) - c) ( f(x) - d) :::; 0
so
f2 (X) + cd :::; (c + d)f(x) , x E [a, b] . Then
l' f2 (X)dx + (b - a)cd ::; (c + d) l' f(x)dx,
and, since the left-hand side is 0, by hypothesis we obtain o
::; (c + d)
and then 0 :::;
(f(x) - C +2 d ) 2
On the other hand, --
Then
=
t f(x)dx,
1 l b f(x)dx -. c+d
(1)
a
2 f2 (X) - (c + d)f(x) + (C +4 d) � 0, x E [a, b]
l b f2 (X)dx + (b - a) (C +4 d) 2 � (c + d) l b f(x)dx , a
a
272 so
6. COMPREHENSIVE PROBLEMS [ rb (c + d) 2 ] � (b a) cd + (b a) 2 (C + d) c + d } f (x )dx. b - a (c - d) 2 � l b f(x)dx . 1
1
a
4
Hence
-
4
1
c+d
c+d
-
-
(2)
a
From (1) and (2) the conclusion follows. ( Titu Andreescu, Revista Matematidi Timi§oara (RMT ) , No. 2 ( 1986) , Problem 6004)
52. By the Mean Value Theorem there are numbers such that
Cx
E
(x, b) and c�
pp.
E
76,
(a, x)
F(x) = (x - a) (b - x)f(cx ) + (x - b) (x - a)f(c� ) = = (x - a) (b - x) [J(cx ) - f(c�)]. We have c� < for all x E [a, b]. Since f is monotonic f(cx ) - f(c�) has constant sign on [a, b] . Moreover, (x - a) (b - x) 0 for all x E [a, b] and the conclusion follows. ( Titu Andreescu, Revista Matematica Timi§oara ( RMT ) , No. 1 (1985) , pp. 63,
Cx
�
Problem 5505)
h
53. a) The function is continuous, hence it is a derivative. Let F be an anti
h. By the Mean Value Theorem we have h( ) F (k + 1 ) - F (k) = Ck for some Ck h Since is nonincreasing, h( ) h(k + 1 ) ::; Ck ::; h(k)
derivative of
so
E
(k, k + 1 ) .
h
h(k + 1 ) � F (k + 1) - F (k) � (k) . Summing these inequalities from k = 1 to k = yields � F (n + 1) - F(l) � an,
n h ( ) an+1 l hence + an+1 - h(l ) � ;:1 n l g'(x)f(g(x))dx � an , n � l . Because ;: n+1 g' (x)f(g(x))dx = 19 (n+1) f(x)dx, 1 the conclusion follows. b ) Setting f : (O, �) JR, g(x) = �, we JR, f(x) = - cot x and g : [ 1, 00 ) h g(l)
obtain (x) =
--+
--+
-; x cot �x , which is decreasing on the interval [ 1, 00 )
.
6.2.
SOLUTIONS
273
!.g (n+1 ) I(x)dx = - ;: � cot xdx = g (l) 1 . 1 ) . � = - In (sm -+ In(sm 1) . = - In(sm x) I1 n+1 Hence !.g (n+1) I(x)dx = lim [- In (Sin _1_ ) + In (sin 1)] = +00 lim n-+oo g(l ) n-+oo n+1 Using the left-hand side inequality from a) it follows that n I cot -l = 00. lim a = lim n n-+oo n-+oo k=l We have
n l
n l
•
� k2 L....t
k
(Dorin Andrica)
54. Recall that For c
>
at - 1 = ln a. t-+limo -t 0 there is 8 0 such that at - 1 In a - c -- In a + c , >
<
t
M Function 1 is integrable on [0, 1] hence is bounded. Let
for all x E [0, 1] . There is an integer no such that
�f (�)
The inequality (1) gives
a
In - c < then
>
0 such that I(x)
for all n � n and k = 0, 1, . . . , n .
o
a*f(�) -) 1 k 1 I n n
(
<
a
In + c ,
k
= 0, 1, . . . , n
n L (a*f(�) - l) k =l In a - c In a + c. � � kt=l n ( n ) n L (a*f(�) - 1) k l = = In a . lim __ n-+oo �� kt=l n ( n ) <
It follows that
(1)
ItI < 8
<
<
______
::; M
274
6. COMPREHENSIVE PROBLEMS On the other hand
n 1 (k) 1 L lim - f - = 1 f(x)dx,
n-+oo k=l n
therefore lm n �oo
°
n
(t, a� f(�) - n) = ([ f(X)dx) In a.
(Dorin Andrica, Romanian Mathematical Regional Contest " Grigore Moisil" ,
1997)
55. 1) We have
. ) - (t-+l --- ) )
2 1' sin nk7rt sin nk7rt 2 1 1 m fn ( X) - l' m m x -+ k� sin k7rt sm k7rt nk7r cos nk7rt 2 = lim = n 2 [( _ 1 ) n k -k ] 2 = n2 . k7r cos k7rt Hence fn is continuous on JR* if and only if an , k = n2 for all k E Z * . 2) It is known that ha JR -+ JR, cos - x -:f. 0 ha (x) = 0, x=0
( t-+ll
1.
(
t-+l
:
is a derivative. Recall the identity
_
a { a'
L cos 2 k - l)x, ( i� nx ) 2 = n + 2 1 9
(
sm x
Then
{a
x E JR \ {m7r, m E Z } .
x -:f. 0 + 2 h2( k - l ) (x), x=o 19
n,
n,
(Dorin Andrica, Romanian Mathematical Regional Contest " Grigore Moisil" ,
1995)
56. We have fo , o (x) = so fo , o is a derivative if cO, o = Functions u
(x )
=
are derivatives, so
{
.
1 x
l.
sm - , x -:f. 0 0,
Cl ,O = CO, l
x=0 = o.
{O
1, if x -:f. 0 c , o , if x = 0
and v(x) =
{ 1 x
cos , x -:f. o 0,
x=o
The function
", , (xl =
6. 2.
{
SOLUTIONS
{ = � sin �,
1 0 1 SI. n - cos - , x #
X X
Cl,l ,
x=
= 21 ( 2x ) + -12 -
u
{
0 0
0
x# 0 2Cl , l , =
X
,
2 Cl, l ,
is a derivative if Cl,l = O . For p, q > 1 consider the differentiable function G : JR --+ JR, 0 . 1 1 x 2 smP - cosq - x #
{
G(x) =
with
0
x
,
x'
x=
0
. P - 1 -1 cosq+ l 1 + 2x sinP -x1 cosq -x1 -p sm -x x
G' (x) = 0
Function
+q sinP+ 1 .!. cosq- l .!.x , x
,
x 1--+
{
{
0
x# x=o
2x sinP .!. cosq .!. x # O x x' 0 , x=O
is continuous, hence a derivative. Therefore
g(x) =
{
-p s inp - l
0
,
.!. cosq+ l .!. + q sinp+l .!. cosq- l .!. , x x x x
is a derivative. Using the fact that sin2 t = 1 - cos2 t and cos2 t = 1 - sin2 t, we 0 1 1 1 1 . . -p smP - l cosq - I + (p + q) smP+ l cosq - I x ' x #
g(x) =
Hence
0
X
,
{
X
X
x=
g(x) = -P!p - l, q- l (x) + (p + q) !p+ l ,q - l (x)+ 0 0 , x # + 0 PCp - l ,q - l - (p + q) Cp+ l, q- l , # = Therefore
{
0
X
0
x#0 = (p + q) Cp- l, q+ l - qCp- l ,q- l , ,
X
PCp - l ,q - l = (p + q) Cp+ l ,q- l qCp- l, q- l = (p + q) Cp - l ,q+ l ,
0
6.
276
COMPREHENSIVE PROBLEMS
and so Cp+l , q - l
=
Cp - l , q+ l
=
For k , l 2: 1 we obtain 2k - 1 C2k , 2 l = 2k + 2l C2k - 2 , 2 l
A,
P
--
p
+q
Cp - l , q- l
q
--
p
Cp , q · + q -l -l
2k - 1 2k - 3 . C l =...= 2k + 2l 2k + 2l 2 2k - 4,2 2k - 3 1 2k - 1 = . CO, l = . 2k + 2l 2k + 2l - 2 . . 2l + 2 2 2l - 1 1 2k - 1 2k - 3 = . . CO, 2 l - 2 = . . . = . . . � 2l + 2 2k + 2l 2k + 2l - 2 1 2l - 1 2l - 3 1 2k - 1 2k - 3 . = co,o = ' " "2 . . 2k + 2l 2k + 2l - 2 ' " 2l + 2 � 2l - 2 1 · 3 . . . (2l - 1) . 1 · 3 . . . (2k - 1) 2 k+ 1 (k + l) ! Note that C2k,2l+ 1 = CO,I , C2k +I , 2 l = B . CI,O and C2k +I ,2l+ 1 = C · CI ,I , where B, C are rational numbers and CO,1 = CI ,O = CI ,1 = 0, therefore =
_
A·
C2k,2 l+1
To conclude,
=
C2k+ I , 2 l
=
C2k+ I , 2 l+1
=
0, for all k, l
{
2: o.
(2k - 1) !! (2l - I ) ! ! -- , if p = 2k and q = 2l ---'-1 -'--2 k+ (k + l)! 0, otherwise (Dorin Andrica, Revista Matematidi Timi§oara (RMT) , No. 1 (1986) , pp. 78, Problem 5773)
Cp,q
=
57. We start with a useful lemma.
Lemma. For any real numbers X l , X 2 ,
cos Xl . COS X2 . · · COS Xn =
1
•
•
•
, Xn , we have
2n L.t COS(±X I ± X2 ± . . . ± Xn )
-
�
where the sum is taken over all 2n possible choices of signs .
n.
The proof can be made by induction on On the other hand recall that function ga : JR -+ JR, ga (x)
=
{ cos a
-
0,
x'
X =I- 0 X=0
is a derivative. We have two cases. q 1 qn - l Case 1 . If q 2: 2, set X l = - , X 2 = - , . . . , Xn = -- , X =I- From the lemma we X X X obtain n 1 q q -l 1 1 cos - . cos - . . . cos -- = cos(±l ± q ± . . . ± q n - l ) _ X X X 2n x
O.
L
6.2.
SOLUTIONS
hence
277
{
� O! (q) X =1= 0 n , 2n O!n (q) , X=0 where O!n (q) is the number of choices of signs +, - such that ± 1 ±q±q2 ± . . . ±qn - l 0, because in the sum are considered only the choices of signs such that ± 1 ± q ± q2 ± . . . ± q n- l =1= O. Note that if ±1 ± q ± q2 ± . . . ± q n - l 0, then q < 1 , which is false. Hence O!n (q) = 0 and therefore fn is derivative if and only if a n (q) = O. Case 2. If q = 1 then for odd we have O!n ( l ) = 0 because ±1 ± 1 ± . . . ± 1 cannot 2m be 0, having an odd number of terms. If is even, let 2m . There are m choices of m signs - and m signs + so fn (x)
= 21n � L.t
g± l ± q ±" ' ± q n - l
(x) +
=
=
n
To conclude, we have O!n (q)
=
1
n
0,
if q � 2 if q = 1 and
n odd q = 1 and n even
(n) n
°i
( )
n=
if ' 2 (Dorin A ndrica, Romanian Mathematical Regional Contest " Grigore MoisH" , 1999) 2
!!
58. 1) Function f is continuous, so it is a derivative. Let F be an antiderivative
of f . For x =1= 0 we have
therefore f
(x2 F G ) ) ' = 2xF G ) - f G)
G) = G ) ( G))' for all - X2 F
2xF
{
x # O.
Consider the function h : JR -+ JR, h(x) =
and note that
2XF(1 / X) , x =1= 0 x O. 2 M (f) ,
= lim 2x F ( � ) x �;:g
=
lim � rY f (s)ds = 2 M (f) . io Hence h is continuous so it is a derivative function. Let H be an antiderivative of h . We have 2x F (1/x) - (x 2 F( l /x))' , x =1= 0 g (x) = x 0 M (f) , lim h(x) �;:g
{
=
y-HX) Y
=
278
6. COMPREHENSIVE PROBLEMS h(x) =
The function u : JR -+ JR, u(x) =
is a deri vative since U : JR -+ JR, U(x)
{ { { ��
(X 2 F ( l / x)) " M (f) ,
X =1= 0 X = o.
(x2 F ( l / x)) " M (f) ,
X =1= 0 x=0
F( l / x) ,
: � �.
=
is differentiable and U' = u. It follows that function G = H 2) We prove that
-
U is an antiderivative of g , as desired.
t
lim � r f(x ) dx = M (f) . t-* oo t io For t > 0 there is an integer n = n(t) and a number a = a (t) t = nT + a. Then '
E
[0, T] such that
j t f(x)dx = inT f (x)dx + it f(x)dx o
On the other hand
(1)
nT
0
i nT f(x)dx = n�- l /. (k+ l) T f(x)dx.
Setting x = B + kT yields
/. (k+ l ) T f(x)dx = kT
so relation (2) gives
(2)
k=O kT
o
rT f (B + kT)dB = r T f(B)dB
io
io
r nT f (x)dx = n rT f (x)dx
io Setting B = x - nT yields
io
it f(x)dx = it - nT f(B + nT)dB = ia (t) f(B)dB nT
0
0
The relation (1) becomes
r t f(x)dx = n r T f (x)dx + r a ( t) f(x)dx
io
and hence
i
io
i
(3)
io
i
t 1 a ( t) f(x)dx for all t > n(t ) T l f (x)dx + f (x)dx = t o -t o t 0
O.
279
6. 2. SOLUTIONS We have
o
hence
1 aCt) 1 aCt) 1 T � t i( f(x)dx � t i( I f(x) l dx � t i( I f(x) l dx t 4°o 0 o o o
I
1
Moreover
i 0
1 a ( t)
tlim -+-oo -t
f(x)dx = O.
n(t) n (t) tlim -+-oo t = tlim -+-oo n (t)T + ( t) 1 1 = tlim -+-oo T + ( t) T '
a
a
n(t)
i
therefore
i
l t f(x)dx = -l T M(f) = tlim f(x)dx, -+-oo t T
o as claimed. The proof is similar for the case t -+
o
-
(Dorin Andrica)
00 .
59. The answer is negative. Indeed, consider the derivative function f : JR -+ JR, f (x) =
{
cos
{I
0,
Using the previous problem, the function
g(x) =
2cos :;; , /I
�,
x =/- 0 x=0
�x I '
x =/- O x=O
is also a derivative. Therefore the function
x =/- 0 x=O is not a deri vative and we are done. 2(Dorin Andrica, Romanian Mathematical Regional Contest " Grigore Moisil" ,
1 99 )
JR
60.
-+
JR,
The answer is negative. For example, consider the derivative functions iI , h :
iI (x) =
{
1 x
- cos - , x =/- 0
0,
x=O
h (x) =
{
1 x ' x =/- O 0, x=O
cos -
6. COMPREHENSIVE PROBLEMS
280
Then f(x) = ma:x ( h (x) , 12 (x)) =
{ I �I ' cos
0, which is not a derivative function (see problem 59). Alternative proof. Consider the derivative functions
{ �, { � cos 0,
h (x) = 12 (x) =
h (x) =
{
x#0 x=0
x =I- 0 x=0
cos , x =I- 0 0, x=0 -
COS 2 � ' x =I- 0 x _1 x=O 2'
Assume for the sake of contradiction that the statement is valid. Then 9
= ma:x ( h , 12 , h)
-
ma:x ( h , h )
is a derivative function, which is a contradiction, since g(x) =
{
0, �
2'
x =l- O x=O
Therefore the answer is negative. (Dorin A ndrica, Romanian Mathematical Regional Contest " Grigore Moh 1997)
"I take great pleasure in recommending to all readers Romanians or from abroad - the book of professors Titu Andreescu and Dorin Andrica. " "All featured problems are interesting, with an increased level of difficulty; some of them are real gems that will give great satisfaction to any math lover ... " "One more time, I strongly express my belief that the 360 mathematics problems featured in this book will reveal the beauty of mathematics to all students and it will be a guide to their teachers and professors. "
Professor loan Tomescu Department of Mathematics and Computer Science University of Bucharest Associate member of the Romanian Academy
I.S.B.N. 973-9417-12-4